Physiology 1, Physiology 2, Physiology 3, Physiology 4, Physiology 5, Physiology 6 Cardio 1, Physiology 7 Cardio 2, Physiology 8 cardio 3, Physiology 9 Respiration 1, Physiology 10 respiration 2, Physiology 11, Physio 12, Physio 13, Physio 14 and 15...

¡Supera tus tareas y exámenes ahora con Quizwiz!

In a given capillary the following set of conditions exists: Blood hydrostatic pressure = 12 mmHg Blood osmotic pressure = 28 mmHg Interstitial hydrostatic pressure = -3 mmHg Interstitial osmotic pressure = 1 mmHg What is the net filtration pressure? -18 mmHg -12 mmHg -14 mmHg +38 mmHg

(12 + 1) - (28 + -3) = 13 - 25 = -12 mmHg, re-absorption.

A 30 ml injection of 2H2O is injected as an isotonic solution (see later why) into a 60-kg woman. After a two hour equilibration, a venous sample is obtain and analyzed for 2H2O. A concentration of 0.001 ml of 2H2O/ ml of plasma was measured. During the equilibrium phase a total of 0.12 ml of 2H2O was lost. Determine total body water?

(30 ml - 0.12 ml)/ .001 ml ~ 30 L TBW ECF= 40% TBW = 12 L ICF= 60% TBW = 18 L

If B means increase and C means decrease, how would these factors influence sarcomere length (preload)?: Decrease total blood volume. Increased heart rate. Increased CVP Decreased ventricular compliance Venocontriction Active skeletal muscle pump Atrial fibrillation

(C is low side on black line, A is in the middle on black line, and B is at the top of black line. Decrease total blood volume. (Goes to C). Increased heart rate. (Goes to C) Increased CVP (Goes to B) Decreased ventricular compliance (Goes to C) Venocontriction (Goes to B) Active skeletal muscle pump (Goes to B) Atrial fibrillation (Goes to C)

Pacemaker Cell Action Potential

(Goes phase 4 -> Phase 0 -> phase 3 -> phase 4) Phase 4 (pacemaker potential): Funny channels are opening as cell is hyperpolarizing allowing slow Na+ leak (increase Na+ permeability) and closing K+ channels. Transient Ca2+ (T-type) channels open, pushing the membrane potential to threshold. Phase 0: (Action potential) Long-lasting Ca2+ (L-type) channels open, giving rise to the action potential. (Permeability to Ca2+ increase and Ca2+ influx) Phase 3: Opening of K+ channels (increasing K+ permeability) and closing of Ca2+ (L-type) channels hyperpolarizes the cell. *(L-type calcium channels cause depolarizaiton)*

GI Neural Control: ANS

(Left) Parasympathetic division of the autonomic nervous system: Signals from parasympathetic centers in the central nervous system are transmitted to the enteric nervous system by the vagus and pelvic nerves. These signals result in either contraction (+) or relaxation (−) of the digestive musculature. (Right) Sympathetic innervation: Preganglionic neurons of the sympathetic division of the autonomic nervous system project to the gut from thoracic and upper lumbar segments of the spinal cord. Efferent sympathetic fibers leave the spinal cord in the ventral roots to make their first synaptic connections with neurons in the prevertebral ganglia in the abdomen. Cell bodies in the prevertebral ganglia are postganglionic neurons, which project to the digestive tract where they synapse with neurons of the enteric nervous system in addition to innervating the blood vessels, mucosa, and specialized regions of the musculature.

Relationship of EKG and Cardiac APs

(This is in ventricle muscle) Phase 4: before P wave or after T wave. Phase 0: QR of QRS complex Phase 1: S of QRS complex Phase 2: ST interval Phase 3: T wave

Regulation of Cortisol/Androgen synthesis

*ACTH regulates production of cortisol and weak androgens* - ACTH binds to Gas receptors - Increases number of LDL receptors (cholesterol uptake) - Promotes conversion of cholesterol to pregnenolone - Promotes adrenal growth and blood flow.

Changes in alveolar pressure relevant to pleural pressure

*Alveolar pressure*: The pressure of the air inside the lung alveoli. *Transpulmonary pressure*: The difference between the alveolar pressure and the pleural pressure *Lung compliance*: The extent to which the lungs will expand for each unit increase in transpulmonary pressure (if enough time is allowed to reach equilibrium) Beginning of inspiration = 0 pressure in alveoli (equal to atmospheric pressure) During inspiration = -1 in alveoli Goes back to 0 at start of expiration. Goes up to +1 air pressure in alveoli during expiration. During this time, the intrapleural pressure gets more negative during inspiration and goes back to baseline during expiration.

ADH hypothalamus-posterior pituitary stimulus

*Arginine vasopressin (ADH)* Stimuli for release: - Increased plasma osmolarity - Decrease blood volume - Others: Ang II, catecholamines Physiological action: - Increase water reabsorption from the distal nephron.

GI Motility: Migrating Motor Complexes

*Basal Electrical Rate*: electrical activity of the stomach set by ICC (Intestitial cells of Cajal). *Migrating motor complex (MMC's)*: Contraction waves in the stomach and on through the intestine. MMC's are interdigestive housekeepers. Hunger Pangs - fasting, migrating myoelectric complexes, using motilin contracts stomach (phase III).

Physiological actions of T3 (4)

*Calorigenesis* - Includes cardiovascular, respiratory, and metabolic actions in support of the *increase in basal metabolic rate*. - Increases O2 consumption and heat production - Increases mitochondrial activity. - Increases carbohydrate, protein, and lipid metabolism *Cardiovascular effects* - Increases cardiac contactile and electrical activity (enhanced O2 delivery to tissues) *Trophic/Growth Effects* - Bone growth - Stimulates GH and IGF-1 production *Nervous system defects* - Normal development - Myelination - Neuronal differentiation - *Thus, severe mental retardation can occur if T3 is absent or low during fetal and early neonatal development.*

Cells of the heart (6)

*Cardiomyocytes* - small, striated, short, thick, branched cells, one centrally located nucleus, involuntary control, contractile *Pacemaker cells/Purkinje fibers* - specialized cardiomyocytes that generate and conduct electrical impulses. Pacemaker cells- SA and AV nodes. Perkinje fibers branch off from pacemaker and line both atriums and ventricles. *Endothelial cells* - line blood vessels and valves *SMCs* (smooth muscle cells) - coronary arteries and inflow/outflow vasculature *Fibroblasts* - more than 50% of heart cells, synthesis and remodeling of ECM *Cardiac Progenitor Cells/Immune Cells (macrophages)*

Amino Acid (tyrosine) derived hormones names and characteristics

*Catecholamines* (Epinephrine (80%) and Norepinephrine (20%) from adrenal medulla) - *Hydrophillic* (water soluble) - *Storage*: Secretory granules, released by exocytosis. - *Transport*: Circulate in blood in free form. - *Half-life*: Short - *Receptors*: Bind to membrane receptors at target cell. ------------------------------------ *Thyroid hormones* ( T3, T4 (thyroxine) from thyroid) - Lipid soluble, water insoluble - *Storage*: In thyroid follicles or released immediately after synthesis by diffusion through plasma membrane. - *Transport*: Circulate in blood coupled to a binding protein - *Half-life*: Long - *Receptors*: Intracellular receptors at target cell.

ANS regulation of the heart (4)

*Chronotropy*: Change in heart rate. Pacemaker potential (SA node) *Inotropy*: Change in contractility. Alterations of intracellular Ca2+ levels. *Dromotropy*: Change in conduction velocity. AV node. Lusitropy: Rate of relaxation (increases lusitropy = increase rate of relaxation).

Types of capillaries (3)

*Continuous*- Small pores, allow glucose and small molecules. *Tight* *Fenestrated*- Larger clefts, less selective - no protein. (Found in kidneys and small intestines, filtering. Have large pores in them) *Leaky* *Sinusoids*- Discontinuous endothelium, pass blood cells and protein. (found in liver or bone marrow. Has incomplete basement membranes, intracellular gaps). *Leakiest*

Medullary control of respiration

*DRG* - Dorsal Respiratory Group located in the nucleus of the tractus solitarii *Medulla*. Has output to diaphragm, involved with establishing the respiratory rhythm. Predominantly contains cells that are active during inspiration. Basically responsible for normal inhalation. *VRG* - Ventral Respiratory Group located in the nucleus of the tractus solitarii located near the nucleus ambiguus *Medulla*. Has both inspiratory and expiratory neurons. Controls forced inhalation or forced expiration *PRG* - Pontine Respiratory Group, consists of the *pneumotaxic* and *apneustic* centers in the *Pons*. - *Aspneustic*- stimulates the inspiratory neurons of the DRG and VRG. Stimulates the VRG to take big breaths in (or shallow breaths if not stimulated). If you lose the aspneustic center you lose ability to take deep breaths, only take short gasps. (Sets length of diaphragm electromyogram) - *Pneumotaxic center* - Sends inhibitory signals to the inspiratory center of the medulla (Stops inspiration). Removal of this center does not inhibit normal rhythmic breathing. This center likely plays a role in fine tuning the respiratory effort. Overstimulation causes fast respiratory rate- hyperventilation. (Responsible for the drop on the electromyogram).

GI Motility: Defecation

*Defecation Reflex* Control: - Intrinsic (enteric) rectosphincteric reflex - contraction of the rectum and relaxation of the internal sphincter (parasympathetic) - Voluntary- relaxation of the external sphincter. Fecal incontinence: maintained by rectosigmoid, anal canal, and pelvic floor musculature.

Hormone clearance

*Degradation*: Signal must dissipate once response is adequate. Location of degradation: - Liver (steroid hormones) - Kidney - Target cell (peptide hormones) *Mechanisms*: Specific enzymes degrade hormone then urinary excretion of degradation products. - Diseases that affect these organs will alter hormone concentrations. - Drugs that alter rate of degradation will affect concentrations.

Diabetes mellitus (insulin pathophysiology)

*Diabetes mellitus*: Lack of insulin release (type 1) or lack of sensitivity to insulin (type 2). Feedback changes: without glucose to breakdown, fats and proteins are activated.

GI Motility: Stomach: Emptying

*Enterogastric Reflexes* Stimulation of emptying: - Neural: increased distension - neural reflexes which increase activity of pump. - Hormonal: gastrin- released due to distension and meat in the stomach causes enhanced activity of pump to promote emptying. Inhibition of Emptying: Neural: Distension, acidity, osmolarity, and products of digestion in the small intestine. Hormonal: CCK, GIP, and secretin released from small intestine. General reflex rules: Entero-Gastric Reflex: Entero: where the food is (intestinal) Gastric: what will be affected (stomach) Affecting ahead: turn it on Affecting behind: turn it off

Pancreas anatomy

*Exocrine Function*- secrete digestive enzymes *Endocrine function*- Endocrine hormone production (Islet of Langerhans): Insulin and glucagon.

Types of filtration (4)

*Filtration only*: Substance is filtered into the tubular nephron and simply excreted in the urine. *Filtration, Partial Reabsorption*: Substance is filtered, but partially reabsorbed. Most of it is excreted in the urine. *Filtration, Complete Reabsorption*: Substance is filtered, but all of the substance is reabsorbed and none is lost in the urine. *Filtration, Secretion*: Substance is filtered, but that which is not is secreted into the tubular nephron and excreted in the urine.

Single unit smooth muscle cells

*Found in stomach and intestines* Smooth muscles are stimulated to contract through a number of factors including nerve stimulation or circulating chemical agents. Smooth muscles, however, can be divided into two groups, based on the electrical characteristics of their plasma membrane: Single-unit smooth muscle: Single-unit smooth muscles are termed "single-unit" because they have synchronized electrical mechanical activity and thus contract as a single unit. They undergo spontaneous rhythmic contractions in the absence of nerve or hormonal input. The electric event that initiates contraction of single-unit smooth muscle is a spontaneous depolarization of the membrane, known as the pacemaker potential (similar to that in cardiac muscle cells). The pacemaker potential is a relatively slow depolarization of the Em due to a gradual increase in Ca2+ conductance. As with other excitable cells, an action potential is triggered when Em reaches threshold. Within a population of single-unit smooth muscle cells, not all the cells have pacemaker potentials. However, non-pacemaker cells do fire action potentials at the same rate as the pacemaker cells since local currents are readily passed through the gap junctions between cells. The gap junctions provide low-resistance pathways for local circuit currents between cells, which allow rapid conduction of action potentials from one cell to the next and virtually simultaneous contraction of the cells. Therefore, the pacemaker cell(s) determine the frequency at which all the cells of single-unit smooth muscle fire action potentials. The activity of single-unit smooth muscles do not require nerve or hormonal input, but can be modified by nerve activity, circulating hormones, mechanical stretch and a variety of drugs. Each of these acts by either depolarizing or hyperpolarizing the Em. When the membrane is depolarized nearer to threshold the frequency of spontaneous action potentials increases and the resulting mechanical activity of the muscle increases. For example: stretching a single-unit smooth muscle depolarizes the membrane by opening stretch-sensitive ion channels, thereby increasing the frequency of action potentials and producing a contraction that opposes further stretch. On the other hand, agents that hyperpolarize the membrane tend to decrease the frequency of spontaneous action potentials, thereby reducing the mechanical response of the muscle.

Multi-Unit smooth muscle

*Found in: Esophagus and Gallbladder* Multi-unit smooth muscles have no or few gap junctions. Each cell responds independently, and the muscle behaves as multiple units. Multi-unit smooth muscles are richly innervated by branches of the parasympathetic and sympathetic autonomic nervous system. The autonomic nerve fibers release neurotransmitters at swelled regions within the nerve axon, called varicosities, along the terminal branches of the nerves. Membrane-bound vesicles in the varicosities contain specific chemical transmitters that are released to the extracellular space when an action potential passes through the varicosity. Since an axon releases transmitter from several varicosities along its length, a single autonomic nerve fiber may influence the activity of a number of smooth muscle cells even though it does not form discrete junction with any one cell. Binding of transmitter to membrane receptors located anywhere on the smooth muscle membrane alters the ionic permeability of the smooth muscle membrane, giving rise to changes in Em. Excitatory or inhibitory potentials may be produced, depending on the transmitter released. Just as with synaptic junctions, a single action potential in the nerve fiber releases enough neurotransmitters to produce only a small sub-threshold change in the Em of a multi-unit smooth muscle cell. Multiple action potentials in the nerve are required to depolarize the smooth muscle membrane to threshold, to trigger an action potential, and subsequently elicit an increase in myoplasmic [Ca2+]. The contractile response of the whole muscle, therefore, depends on the number of muscle cells that are activated and on the frequency of nerve stimulation. In addition to stimulation by autonomic nerves, multi-unit smooth muscle may also be induced to contract by certain hormones which are carried to the cells via circulation. The hormones bind to receptors on the membrane surface and increase intracellular [Ca2+] by G-protein activated intracellular signaling mechanisms. For example, norepinephrine binds to α1-adrenergic receptors causing a signaling cascade that increases myoplasmic [Ca2+] from the release of Ca2+ through IP3 receptors in the SR.

Bile function, composition, and production and secretions.

*Function:* facilitates the digestion of lipids in the small intestine by emulsifying fat and forming aggregates around fat droplets called micelles *Composition:* - Bile Salts: Amphipathic molecules which solubilize dietary lipids - Phospholipids (lecithin): Aid in solubilizing fats - Cholesterol: Aid in fat digestion and are removed as waste - Bilirubin: Breakdown of hemoglobin from red blood cells, removed as waste - Water and Electrolytes *Production and Secretions:* - Linked to Na/K-ATPase activity - Regulated by amount of bile salts returning

GI secretions function (2), Types of secretion (4), regulation of secretion (3..) and secretion mechanisms (2)

*Functions*: 1. Aid in digestion 2. Provide lubrication and protection *Types of secretion*: 1. Saliva 2. Gastric juice 3. Pancreatic juice 4. Intestinal secretions *Regulation of secretion* Local stimulation Autonomics GI Hormones - Gastrin - Cholecystokinin (CCK) - Secretin - Glucose-dependent insulin tropic polypeptide *Secretion mechanisms*: - Organics - mucus and proteins - Water/electrolytes

Sodium balance, glomerulotubular balance.

*Glomerulotubular balance*: Na+ excretion increases and decreases with the increases in GFR. An increase in GFR is associated with higher Glomerular pressures due to excess fluid, a sign to reduce fluid levels in the body. Renin-Angiotensin-Aldosteron system (RAAS) is heavily modified by NaCl balances in the Macula Densa. The majority of sodium is reabsorbed by the proximal tubule. By the end of the Distal Convoluted Tubule nearly 96% of filtered Na+ has been reabsorbed. The collecting duct works to actively reabsorb the remaining 3%. Very little sodium is lost to the urine. When water levels change in the body excess Na+ can be lost in the urine by increasing GFR and tubular flow to reduce the time spent near transporters. Though even at higher GFR's, Na+ reabsorption increases to keep retention of Na+ high. Since Na+ is the most abundant ion in the plasma, we need to maintain our plasma Na+ levels fairly tightly.

Receptors for cortical steroids

*Glucocorticoid Receptor (GR) or Mineralocorticoid Receptor (MR)* - Translocate to nucleus - Bind to glucocorticoid response elements (GRE) - Increase or repress protein expression.

Glycolysis, gluconeogenesis, glycogenesis, and glycogenolysis

*Glycolysis*: ATP- generating process in which carbohydrates are converted to pyruvic acid. *Gluconeogenesis*: Energy-dependent production of glucose from non-carbohydrate sources (fat, amino acids) (mainly in liver and kidneys) *Glycogenesis*: Process by which glycogen is synthesized from circulating glucose (occurs mainly in liver) *Glycogenolysis*: Process by which glycogen is broken down to glucose-6-phosphate and then to glucose for subsequent release into circulation (specific to the liver).

GI Neural Control: Pathophysiology. Hirschsprung Disease (congenital angaglionic megacolon).

*Hirschsprung Disease (congenital anganglionic megacolon).* Cause: Failure of enteric neural cells to migrate properly during development. Signs and symptoms: - Absence of ganglionic innervation to the muscles of a segment of the bowel (usually in the lower portion of the sigmoid colon) - Lack's normal peristalsis, results in constipation. - Stools are typically ribbon-like due to passing of feces through the narrow segment of colon. - Portion of bowel nearest the obstruction dilates, causing abdominal distention. Treatment: - Resection surgery of affected area.

Peptide/polypeptide hormone characteristics

*Hydrophilic* (water soluble) *Synthesis*: Intracellular, many as preprohormones. *Transport*: Usually circulate in blood in free form, dissolve in plasma *Half-life*: short (rapid metabolism and excretion). *Receptors: Extracellular/membrane receptors (Exception: certain peptides are not synthesized via this intracellular mechanism as preprohormones such as Angiotensin II).

Mechanism of insulin effects

*Insulin receptor* - Tyrosine Kinase Receptor (RTK) - Heterotetramer - Activation of Beta subunit results in autophosphorylation - Activity terminated by dephosphorylation and internalization.

Physiological actions of PTH

*Key Actions of PTH*: 1. *Increase plasma Ca2+* - Promote Ca2+ resorption from bone. - Increase Ca2+ resorption from kidney. - Increase active vitamin D (calcitriol) levels by activating renal *1a-hydroxylase*. - Calcitriol promotes Ca2+ absorption in gut. 2. *Decrease plasma phosphate* - Decrease PO4- reabsorption from the kidney

Thyroid hormone actions at target cells key points (3)

*Key Points*: 1. T3 is biologically active hormone 2. T3 binds to a nuclear receptor 3. T3 has multiple cellular actions.

Steroid and steroid-like hormones characteristics

*Lipophilic* (lipid soluble, binds to intracellular receptors) water insoluble. *Synthesis*: Intracellular, derived from cholesterol via enzymatic conversions *Storage*: Not stored, released by diffusion through plasma membrane *Transport*: Mostly circulate in blood coupled to a binding protein. *Half-life*: Long (slow metabolism and excretion) *Receptors*: Intracellular (usually cytosolic or nuclear) can be membrane-bound.

GI Motility: Esophagus- Deglutition (lower esophageal sphincter)

*Lower esophageal sphincter (LES)*- relaxes prior to bolus near sphincter-enteric response (VIP). - Normally tonic constricted; decrease in constriction leads to GERD. - Non-relaxation results in achalasia (rare disorder that makes it difficult for food and liquid to pass into your stomach).

Juxtaglomerular apparatus

*Macula densa cells*: Monitor the composition of fluid in the tubule lumen. Detect the flow rate of the filtrate in the *thick ascending limb* and release mediators. *Juxtaglomerular cells (granular cells)*: Surround the *afferent arteriole*. Synthesize and release *renin* in response to the mediators sent from the macula densa cells. The Afferent and Efferent arteriole pass the macula densa. Afferent arteriole is lined by granular cells/ juxtaglomerular cells that are somewhat *leaky*. When blood pressure drops, the flow rate within the thick ascending limb decreases which stimulates Macula densa cells to release mediators which signal the granular/juxtaglomerular cells to secrete Renin which leads to the Renin-Angiotensin-System response.

Juxtaglomerular Apparatus

*Macula densa cells*: Monitor the composition of the fluid in the tubule lumen. Detect the flow rate of the filtrate in the *thick ascending limb* and release mediators. *Juxtaglomerular cells (granular cells)*: Surround the *afferent arteriole*. Synthesize and release renin in response to the mediators sent from the macula densa cells. Afferent and Efferent arteriole pass the macula densa. Afferent arteriole is lined by ganular/juxtaglomerular cells that are somewhat leaky. When pressure drops they secrete Renin into the blood supply leading to the Renin-Angiotensin-System response.

Adrenal cortex anatomy

*Medulla*: Inner region which produces epinephrine and norepinephrine. *Cortex*: Outer region which produces mineralocorticoids (aldosterone), glucocorticoids (cortisol), and androgens (DHEA and androstenedione).

What are the *oxyntic gland* cells and what functions do they have?

*Mucous Neck Cells (surface cells)*: - Viscid alkaline coating *Parietal cells*: - Secrete HCL and intrinsic factor - Under ANS/hormone regulation - Secretion requires high amounts of ATP. *Peptic (Chief) Cells*: - Secrete Pepsinogen (zymogen) - Stimulated by ACh and HCL

Enteric/Intrinsic Nervous System

*Myenteric (Auerbach's) plexus* controls smooth muscle layer contraction and is stimulated by acetylcholine which increases tone and movement and inhibited by vasoactive intestinal peptide (VIP) that relaxes sphincters. *Submucosal (Meissner's) plexus* controls gastrointestinal secretions and blood flow and responds to both local and extrinsic signals. Overall neurotransmitter input to the enteric nervous system involves acetylcholine as an excitatory neurotransmitter and norepinephrine as an inhibitory neurotransmitter. Others that modulate the processes of the GI system include ATP, serotonin, VIP, somatostatin, and bombesin.

Hallmarks of the EKG recording

*P wave*: Atrial depolarization *QRS complex*: Ventricular depolarization *T wave*: Ventricular repolarization *R-R interval*: One cardiac cycle (one heartbeat) *PR interval*: This interval measures the time from the initial depolarization of the atria to the initial depolarization of the ventricles and reflects a physiological delay in AV conduction imposed by the AV node (travels slowly in AV node but fast in bundle of his). *QRS interval*: Time for wave of depolarization to spread through ventricles (Q wave first negative deflection from the baseline following p wave) R first positive deflection from baseline following Q wave. S first negative deflection that extends below the baseline following the R wave) *ST interval*: Time between ventricular depolarization and repolarization *QT interval*: Length of ventricular depolarization and repolarization. (Don't see atrial repolarization wave because it happens in a lot of different directions)

Transit time of gas molecules

*Perfusion Limited* - Transfer of the molecule into the capillary can only be increased by increasing blood flow. It rapidly reaches equilibrium. *Diffusion Limited* - Since the molecule reaches equilibrium in solution slowly, diffusion is the limiting factor of transport across the membrane. (Special case- carbon monoxide never reaches saturation and is diffusion limited) In the blood this can be complicated because molecules like Hemoglobin (and buffers) readily bind to gases, thus reducing their partial pressures and causing diffusion limitations. *Gases like O2 and CO2 are perfusion limited*

GI Motility: Small Intestine: Propulsion

*Peristalsis* - propulsive movements of chyme (0.5 - 2.0 cm/sec) Integrated control - Gastroenteric (peristaltic) reflex: distension of intestine causes propulsion, ENS mediated. - Hormones: -- Gastrin, CCK, Insulin, and Motilin enhance motility. -- Secretin and Glucagon- decrease motility. -Gastroileal Reflex- distention of the stomach intensifies peristalsis at the ileum forcing chyme into the cecum. Power Propulsion- irritation causes powerful contraction all the way to the colon. - Giant migrating contractions - Enteric and brainstem control

Calcium and Phosphate regulation

*Physiological Actions of Calcium* - Major constituent of bones and teeth. - Synaptic transmission (neurotransmitter release) - Maintenance of sodium permeability in nerves - EC coupling in muscle cells - Intracellular signaling - Calcium-dependent enzymes. *Physiological actions of phosphate* - Major constituent of bones and teeth - Intracellular buffering - Constituent of many macromolecules (phospholipids, phophoproteins, nucleic acids) - Enzyme activation/inactivation via phosphorylation/dephosphorylation. - Component of metabolic intermediates (NADPH) - Component of ATP

Proximal tubule cell transporters in secretion.

*Primary Active Transporters*: - Multidrug Resistant-associated protein (MRP) - Na/K ATPase *Secondary Active Transporters*: - Organic Cation Transporter (OCT) - Na/H Exchanger - Organic Anion Transporter (OAT) - H/Organic Cation Exchanger - Na/aKG Exchanger - PAH/Anion Exchanger Proximal tubule secretes large variet of organic anions (carboxylates and sulfonates like penicillin and PAH) and organic cations (amine and ammonium), many of which are endogenous compounds, drugs, or toxins. Proximal tubule take up PAH from the blood by antiport with a-ketoglutarate via organic anion transporter (OAT). Alpha-ketogluterate accumulates within the tubule cell due to metabolism -> a-KG or through the Na+/a-KG dicarboxylate symporters. Then PAH starts accumulates in the cells getting this a-KG out via OAT. PAH then moves out of the tubule cell into the tubule urine for secretion via exchange with an inorganic anion (Cl-) or an organic anion via another OAT (high PAH concentration in cell drives this). PAH and other organic anions can also be transported into the tubule urine via active transport (uses ATP) through MRP (multidrug resistance-associated protein). At high blood/plasma organic anion concentration, the anions compete with each other for secretion. Organic cations (mainly amine and ammonia) move from the blood into the tubule cell by facilitated diffusion via a cation transporter thanks to the -70 mV resting membrane potential inside the cell. To get from inside the tubule cells to the tubule urine the organic cations are transported via organic cation/H+ antiporter (exchanger). This antiporter is driven by the concentration gradient of H+ between the tubule urine and the tubule cell (H+ high in tubule urine allows it to go down concentration gradient into tubule cell and exchange organic cation which goes from cell to urine). This H+ gradient is established by the Na/H+ exchange.

Glucagon actions in liver

*Primary target of glucagon is the liver*

Blood flow and gravity

*Q - flow* Blood flow is highest at the base of the lungs due to gravity. At the apex blood flow is actually very low.

Mechanisms of inotropy (contractility)

*Regulate intracellular calcium, Inotropy has nothing to do with changing the length of the sarcomere* -Increasing Ca2+ influx - Increasing Ca2+ release from the SR - Increasing troponin sensitivity to Ca2+ More cross-bridges!

GI Motility: Small Intestine: Segmentation

*Segmentation*- distension causes localized constriction of smooth muscle. BER slow waves vary in different regions of the small intestine.

CRH- short-term stress vs prolonged stress.

*Short term stress*: Stress (stimulates) -> nerve impulses -> spinal cord -> preganglionic sympathetic fibers -> adrenal medulla -> secretes amino acid-based hormones epinephrine and norepinephrine (catecholamines). Short term stress response: - Increase heart rate - Increase blood pressure - Bronchioles dilate - Liver converts glycogen to glucose and releases glucose to blood - Blood flow changes, reducing digestive system activity and urine output. - Metabolic rate increases --------------------------------- *Prolonged stress* Stress (stimulates) -> hypothalamus -> CRH (corticotropin releasing hormone) -> Corticotropic cells of anterior pituitary -> ACTH -> Adrenal cortex -> Secretes Mineralcorticoids (aldosterone) and Glucocorticoids (cortisol). Long-term stress response: Mineralcorticoid: - Kidneys retain sodium and water - Blood volume and blood pressure rise. Glucocorticoids: - Proteins and fats converted to glucose or broken down for energy - Blood glucose increases - Immune system suppressed.

Factors that influence vascular compliance

*Sympathetic stimulation*- decreases compliance. - Important for regulation of venous pressure and cardiac preload. - Reduced aortic compliance with age or disease. Dotted line symbolizes sympathetic stimulation, for the same pressure there is a decrease in volume in vein due to venoconstriction (major effect in veins). Also, for the same pressure there is a decrease in volume in artery due to vasoconstriction (smaller effect than in veins).

TSH Regulation of the Thyroid Gland

*TSH binds to Gas and Gaq protein-coupled receptors.* TSH binding promotes: *Synthesis/secretion of thyroid hormone*: 1. Increases iodide uptake. 2. Activates thyroid peroxidase - Iodination - Coupling 3. Increases thyroglobulin synthesis 4. Increases secretion of T3/T4 - Endocytosis of colloid - Proteolysis of thyroglobulin *Growth of thyroid* - Increased cell size/number

Synthesis and storage of thyroid hormones

*Thyroid peroxidase*: 1. Converts I -> I2 2. Iodinates tyrosines of thyroglobulin 3. Coupling of iodotyrosines Pendrin = CI/I exchanger Iodine from plasma gets the follicular epithelium (cuboidal) cell via NIS ("iodide trap") transporter (symport Na+/I-). Then crosses over into the colloid via a pendrin channel (Cl-/I- antiport). Then Thyroid peroxidase acts on I- and turns it into I2. Then I2 binds with Tyrosine complex under thyroid peroxidases in process called Iodination. New MIT + DIT complex undergoes another round of thyroid peroxidase in a process called Coupling and turns into T3, T4, rT3, MIT, and DIT. This complex is then taken into storage. Basal level of thyroid hormone synthesis is maintained by the sympathetic nervous system. *TG is thyroglobulin*

Vasoconstrictors vs Vasodilators types

*Vasoconstrictors*: Catecholamines (Epinephrine, Norepinephrine) Angiotensin II Vasopressin (ADH) *Vasodilators*: Bradykinin Histamine Atrial Natriuetic Peptide (ANP)

Venous return

*Venous Return*: the flow of blood back to the heart. Over time, venous return must equal cardiac output. This balance is largely achieved by the Frank-Starling mechanism. If venous return does not equal cardiac output then that means blood is backing up somewhere, either in periphery or lungs.

Venules (collection vessels)

- Collect blood from capillary bed - Low resistance

Hypothalamic-Putuitary-Gonadal axis in females

- GnRH (gonadotropin-releasing hormone) - LH (luteinizing hormone) - FSH (follicle-stimulating hormone) - Estrogen (estradiol) - Progesterone - Androgens - Inhibin Functions in females: - Growth and maturation of follicles - Estradiol and progesterone - Prepares endometrium The most biologically potent naturally occurring estrogen - 17B-estradiol.

Veins (Capacitance vessels)

- Thin- walled (little smooth muscle) - Low pressure (0-2 mmHg) - High compliance - Blood reservoir (most blood stored here, Capacitance = storage). *Compliance* is the ability of a hollow organ (vessel) to distend and increase volume with increasing transmural pressure.

What is the average resting membrane potential

-70 mV

Structure of the kidneys

-About the size of a fist -20% of cardiac output -8-10 pyramidal lobes inside -1 million nephrons Outer Cortex Inner Medula Renal Blood Supply - Afferent arterioles - Glomerulus - Efferent arterioles - Peritubular capillaries - Veins The pyramidal lobes end at the renal papilla which supplies urine to the Renal Pelvis.

Capillaries (Exchange Vessels)

-Endothelium (one cell thick) and basement membrane. - Pores which allow for exchange of nutrients. - Areas with high metabolic requirements have extensive capillary networks (muscles, liver, kidneys, nervous system). - Areas with very low metabolic requirements lack capillaries. (cornea and lens of eye, nails, hair follices, cuticles, cartilage)

12 Lead EKG

-Gives more angles - View of various coronary distributions. Lead I Lead II Lead III aVR aVL aVF V1-V6

Collecting duct

-Monitors K+ levels - *Principle site of K+ secretion in the body.* - Tight membranes prevent passive water movement. - ADH/Arginine-Vasopressin works to increase water movement through aquaporins in the collecting duct. -Aldosterone increases Na+ permeability through the ENaC (epithelial Na+ channel) and upregulates the number of Na+/K+ ATPases in the basolateral membrane. Under normal circumstances, the cortical collecting ducts secrete most of the excreted K+. With great K+ excess, the cortical collecting ducts may secrete so much K+ that more K+ is excreted than was filtered. With severe K+ depletion, the cortical collecting ducts reabsorb K+ (via alpha intercalated cells). K+ secretion appears to be a function primarily of the *collecting duct principal cell.* K+ secretion involves active uptake by a Na-K-ATPase in the basolateral cell membrane, followed by diffusion of K+ though luminal membrane K+ channels. The hormone aldosterone promotes K+ secretion by several actions.

Alveolar gas exchange

-Most of the volume of the lungs is filled with air. - Capillaries surround the alveolar sac (air sacs) to maximize surface area for diffusion of gases. *Of note* The lungs have very little interstitial space, therefore, any additional fluid volume would typically go into the alveolar sac as there's no additional space that it can occupy outside of the capillary.

Capillary gas exchange

-The main point of all this is gas exchange (CO2 and O2) It's important to recognize the names and terms for the spaces involved. Note the very limited interstitial space of the lungs. Air sacs are completely pressed against capillaries.

What is the tidal volume for males? Females?

0.5 L (500 mL) for both.

Phases of action potential

1 - resting state 2- Graded potentials add up and hit threshold Phase 1: (threshold hit) Depolarization phase (0 mV = completely depolorized, Na+ channels open at this time, K+ not open yet) (observed in both graded potentials (subthreshold) and phase 1 of an action potential.) Overshoot: Inside of cell temporarily becomes more positive than outside (late phase 1, early phase 2 of action potential) Phase 2- Repolorization phase. (Na+ channel close (inactivation gate closed), K+ channel open) (Observed in both graded potential (subthreshold) and phase 2 of an action potential. Phase 3- Hyperpolarization phase. (observed in both graded potentials and late phase 2/ early phase 3 of an action potential). (K+ channels take a bit too long to close)

1. Given: Pc = 40, πi = 3, Pi = 2, and πc = 25, what is the net filtration rate? 2. Given: Pc = 10, πi = 3, Pi = 2, and πc = 25, what is the net filtration rate?

1. (40 + 3) - (2 + 25) = +16, filtration is favored. (arteriolar end) 2. (10 + 3) - (2+ 25) = -14, re-absorption is favored. (venous end)

Steroid Biosynthesis

1. *Aldosterone* Pregnenolone + *3b- hydroxysteroid dehydrogenase* -> progesterone + *21a- hydroxylase* -> 11-deoxycorticosterone + *11b- hydroxylase* -> corticosterone + *aldosterone synthase* -> aldosterone. 2. *Androgens* Pregnenolone or progesterone + *17 a-hydroxylase* -> 17 a-hydroxy-pregnenolone + *17,20 desmolase* -> DHEA + *3b hydroxylsteroid Dehydrogenase* -> androstenedione -> testosterone. Testosterone + *5a reductase* -> dihydrotestosterone (DHT) which is important in males for secondary sex characteristics. Testosterone + *Aromatase* -> Estradiol (estrogen). 3. *Cortisol* - Pregnenolone + *3b- hydroxysteroid dehydrogenase* -> progesterone + *21a- hydroxylase* -> 11-deoxy corticosterone + *11 b-hydroxylase* -> corticosterone + *17 alpha hydroxylase* -> cortisol. (cortisone made by 11b HSD) ------------------------------- Need to have *3 beta HSD, 21 hydroxylase, and 11 beta hydroxylase* to make aldosterone and cortisol (cortisol also needs *17 alpha hydroxylase*) Mutation in any of these and you won't get cortisol or aldosterone. Need 17 alpha hydroxylase to make cortisol and androgens. Mutation in any of these and you won't get cortisol or androgens. Will cause increased levels of the precursors and subsequently increase in aldosterone. Aldosterone synthase important in corticosterone to aldosterone. Aromatase important in testosterone to estrogen convertion. 5a reductase turns testosterone to DHT- especially important in males for secondary sex characteristics.

Aside from glucose, what else can trigger insulin release from beta cell?

1. *CCK and ACh* coupled to Gaq -> PLC -> IP3/DAG pathway. 2. Glucagon and B-adrenergic agonist Gas -> cAMP -> PKA pathway. (Glucagon triggers insulin to make sure that glucagon itself doesn't release too much glucose into bloodstream) Stimulators: 1. CCK (cholecystokinin) 2. ACh 3. Glucagon 4. B-adrenergic agonist Act by increasing cAMP or Ca2+

What are the 3 gastric phases

1. *Cephalic phase*: Via vagus nerve. Parasympathetics excite pepsin and acid production. Accounts for 30% of acid secretion. (stimulated by Smelling, chewing, and swallowing food (or merely the thought of food)) 2. *Gastric phase*: Accounts for 60% of acid secretion. 1. Local nervous secretory reflexes 2. Vagal reflexes 3. Gastrin-histamine stimulation 3. *Intestinal phase*: Accounts for 10% of acid secretion. 1. Nervous mechanisms 2. Hormonal mechanisms

What are the actions of insulin?

1. *Decreases blood glucose - carbohydrate metabolism.* - Increases glucose transport into target cells: muscle, adipose - Increases glucokinase activity (glycolysis) - Promotes the formation of glycogen, inhibits glycogenolysis - Inhibits gluconeogenesis - inhibits liver enzyme 2. *Decreases fatty acid and ketoacids in the blood* - Excess glucose from the liver converted to fatty acids then fats and transported to adipose tissue. - Fatty acids taken up by adipose tissue and converted to fats. 3. *Decreases blood amino acid concentration* - Promotes protein anabolic processes in muscle - Stimulates transport of amino acids into cells.

Nutrient status and pancreas endocrine release

1. *Nutrient abundance* (fed state) - Pancreas secretes *insulin* (hormone of nutrient abundance) - Promotes nutrient uptake and storage. 2. *Nutrient deficiency* (fasting) - Pancreas secretes *glucagon* - Promotes nutrient usage (stimulate nutrient formation and breakdown of nutrient stores).

Calcium regulation

1. *Parathyroid hormone (peptide)*: Increase plasma Ca2+ 2. *Vitamin D metabolite [1,25 (OH)2D3] (Calcitriol)*: Short-term: Supports the actions of PTH to help increase plasma Ca2+. Longer-term: Negative feedback over PTH secretion. 3. *Calcitonin (peptide)*: Decrease plasma Ca2+.

GI motility

1. *Propulsive: which is Peristalsis* - Contractile ring and relaxation - Coordinated ENS long and circulatory muscle reflex - Moves food Orad (located next to oral opening) to Caudad (towards the posterior). (Propulsion/Peristalsis is the controlled movement of ingested food, liquids, GI secretions, and sloughed cells from the mucosa through the digestive tract. It moves the food from the stomach into the small intestine and along the small intestine with appropriate timing for efficient digestion and absorption. Propulsive forces move undigested/unabsorbed material into the large intestine and eliminate waste through defecation. 2. *Mixing: segmentation* - Trituration (decreases particle size, thereby increasing the surface area for action by digestive enzymes in the small intestine) - Mixing motility pattern (blends pancreatic, biliary, and intestinal secretions with nutrients in the stomach and bring products of digestion into contact with the absorptive surfaces of the mucosa.) - Haustrations (Ring-like contractions of the circular muscle divide the colon into pockets called haustra. The haustration motility pattern is reminiscent of mixing (segmentation) movements in the small intestine. Nevertheless, haustral formation differs from small intestinal segmentation in that the contracting and the receiving segments on either side remain in their respective states for extended periods of time.)

Hormone transport via what 2 ways

1. *Water-soluble*: No special mechanisms for transport. - Amino acid-derived catecholamines (epi and norepi) - Peptide 2. *Lipid-soluble*: Bound to plasma proteins. - Steroid - Amino acid-derived thyroid hormones (T3 and T4). ------------------------------------ Only 1-10% of total hormone present in plasma exists in free form Free hormone and carrier-bound hormone are in dynamic equilibrium with each other. Only free hormone is biologically active Must be unbound from carrier protein before being able to cross plasma membrane.

Reabsorption and generation of HCO3- throughout the nephron

1. 80% reabsorbed in the proximal convoluted tubule. 2. 10% reabsorbed in the thick ascending limb. 3. 6% reabsorbed in the distal convoluted tubule 4. 4% reabsorbed in the outer medullary collecting duct. -------------------------- 40 mmol/day H+ secreted coupled to NH3 in the proximal tubule. 15 mmol/day H+ secreted coupled with titrated acid in the proximal tubule. 5 mmol/day H+ secreted coupled with titrated acid in the distal convoluted tubule. 10 mmol/day H+ secreted coupled with titrated acid in the inner medullary collecting duct. Total = 70 mmol/day H+ excreted. When H+ gets excreted, it makes the equivalent of HCO3- (because they now don't have the H+ to pair with). *So 70 mmol/day of HCO3- is made this way. *

What 2 things are needed for movement to occur across membranes?

1. A driving force- that which is initiating the movement of molecules. 2. A pathway

1. A-band 2. H band 3. I band 4. M band 5. Z disk or z line 6. Titin

1. A-band (dark band): Everything that includes all the myosin. 2. H band: Contains myosin but no actin. 3. I band (light band): Contains actin but no myosin. 4. M band: Bunch of proteins that hold myosin in place. 5. Z disk or z line: Helps hold actin in place. 1 sarcomere is Z line to z line. 6. Titin: Attaches from z line to M band, holds half a sarcomere.

excitation-contraction coupling steps.

1. ACh released at synaptic terminal diffuses across synaptic cleft and binds to receptor proteins on muscle fiber's plasma membrane, triggering an action potential in the muscle fiber. 2. Action potential is propagated along the plasma membrane and down T tubules. 3. Action potential triggers Ca2+ release from the SR. 4. Calcium ions bind to troponin in the thin filament; myosin binding sites are exposed. 5. Cycles of myosin cross-bridge formation and breakdown, coupled with ATP hydrolysis, slide the thin filament towards the center of the sarcomere. 6. After the action potential ends, cytosolic Ca2+ is removed by active transport into the SR. 7. Once cytosolic Ca2+ is removed, tropomyosin again blocks the myosin binding sites. Contraction ends and the muscle fiber relaxes.

ADH cell pathway

1. ADH from plasma binds to the ADH receptor on the principal cell lining the late distal tubule or collecting duct. 2. Stimulates Gas -> cAMP -> PKA 3. PKA transcribes for Aquaporin channels to be placed at tubular side to allow more water reabsorption. Placed at basolateral membrane for water reabsorption into plasma.

Excitation-contraction coupling steps

1. Acetylcholine (ACh) released at the synaptic terminal diffuses across the synaptic cleft and binds to receptor proteins on the muscle fiber's plasma membrane (, triggering an action potential in the muscle fiber. 2. Action potential is propagated along the plasma membrane down T tubules. 3. Action potential triggers Ca2+ release from the SR. 4. Calcium ions bind to troponin in the thin filament; myosin-binding sites are exposed. 5. Cycles of myosin cross-bridge formation and breakdown, coupled with ATP hydrolysis, slide the thin filament toward the center of the sarcomere. 6. After the action potential ends, cytosolic Ca2+ is removed by active transport into the SR. 7. Once cytosolic Ca2+ is removed, tropomyosin again blocks the myosin binding sites. Contraction ends and the muscle fiber relaxes.

Muscle fiber relaxation steps (6)

1. Acetylcholinesterase decomposes ACh, the muscle fiber membrane no longer stimulated. 2. Calcium ions are actively transported into the sarcoplasmic reticulum. 3. ATP breaks cross-bridge linkages between actin and myosin filaments without breakdown of the ATP itself. 4. Breakdown of ATP "cocks" the myosin heads. 5. Troponin and tropomyosin molecules block the interaction between myosin and actin filaments. 6. Muscle fiber remains relaxed, yet ready until stimulated again.

Excitation contraction coupling in cardiac muscle

1. Action potential enters from adjacent cell. 2. Voltage-gated Ca2+ channels open. Ca2+ enters cell. 3. Ca2+ induces Ca2+ release through ryanodine receptor-channels (RyR). 4. Local release causes Ca2+ spark. 5. Summed Ca2+ sparks create a Ca2+ signal. 6. Ca2+ ions bind to troponin to initiate contraction. 7. Relaxation occurs when Ca2+ unbinds from troponin. 8. Ca2+ is pumped back into the sarcoplasmic reticulum for storage. 9. Ca2+ is exchaned with Na+ by the NCX antiporter 10. Na+ gradient is maintained by the Na+/K+ ATPase.

Simple molecular mechanisms of skeletal muscle contraction

1. Activation of Neuromuscular Junction. 2. Excitation-Contraction-Coupling Electrical signal -> muscle contraction 3. Cross-Bridge Cycling (Sliding Filament Theory)

What are the 2 special cases of sympathetic innervation?

1. Adrenal medule: postganglionic is epinephrine released by chromaffin cells. 2. Sweat glands: Sympathetically innervated and release ach post ganglionic Sympathetic release norepinephrine postganglionic except in these two instances.

What are the 23 peptide/polypeptide hormones?

1. Adrenocoricotropin (ACTH) 2. Atrial natriuretic peptide (ANP) 3. Arginine vasopressin (AVP) (AKA: antidiuretic hormone [ADH]) 4. Calcitonin 5. Cholecystokinin (CCK) 6. Corticotropin-releasing hormone (CRH) 7. Follicle-stimulating hormone (FSH) 8. Glucagon 9. Gonadotropin-releasing hormone (GnRH) 10. Growth hormone (GH) 11. Growth hormone-releasing hormone (GHRH) 12. Inhibin 13. Insulin 14. Insulin-like growth factors I and II (IGFI and II) 15. Luteinizing hormone (LH) 16. Oxytocin (OT) 17. Parathyroid hormone (PTH) 18. Prolactin (PRL) 19. Secretin 20. Somatostatin 21. Thyrotropin (TSH) 22. Thyrotrophin-releasing hormone (TRH) 23. Vasoactive intestinal peptide (VIP) Know the amino acid and steroid hormones, everything else is peptide hormones.

What are the 5 steroid hormones and what are their signaling characteristics

1. Aldosterone 2. Cortisol 3. Estradiol (E2, estrogen) 4. Progesterone 5. Testosterone - Adrenal cortex, ovaries, testes, and placenta. - Mostly alter gene expression - Tend to be longer-lasting effects

3 main classes of hormones and the physical characteristics that lead to variations.

1. Amino Acid Hormones 2. Polypeptide/peptide hormones 3. Steroid and steroid-like hormones These physical characteristics lead to variations in: - Synthesis - Transport - Half-life - Receptors used on target tissues.

Neuromuscular junction (NMJ) steps.

1. An action potential in a motor neuron is propagated to the terminal button. 2. The presence of an action potential in the terminal button triggers the opening of voltage-gated Ca2+ channels and the subsequent entry of Ca2+ into the terminal button. 3. Ca2+ triggers the relase of acetylcholine by exocytosis from a portion of the vesicles. 4. Acetylcholine diffuses across the space separating the nerve and muscle cells and binds with receptor sites specific for it on the motor end plate of the muscle cell membrane. 5. This binding brings about the opening of cation channels, leading to a relatively large movement of Na+ into the muscle cell compared to a smaller movement of K+ outward. 6. The result is an end-plate potential. Local current flow occurs between the depolarized end plate and adjacent membrane. 7. This local current flow opens voltage-gated Na2+ channels in the adjacent membrane. 8. The resultant Na2+ entry reduces the potential to threshold, initiating an action potential, which is propagated throughout the muscle fiber. 9. Acetylcholine is subsequently destroyed by acetylcholinesterase, an enzyme located on the motor endplate membrane, terminating the muscle cell's response.

Action potentials characteristics

1. Are all-or-none 2. Are not graded by stimulus size 3. Cannot summate because of refractory periods - Absolute refractory period - Relative refractory period 4. Do not decrease with distance.

Neural reflex mechanisms in BP control (3)

1. Arterial baroreceptor reflex (high-pressure receptors) 2. Atrial baroreceptor reflex (low-pressure receptors) *volume receptors* 3. Arterial chemoreceptors - regulates respiration and also affects BP (sense decrease in O2, increase in CO2, decrease in pH).

Autonomic reflex in heart in response to increased blood pressure

1. Baroreceptors sense increased blood pressure (receptor). 2. Glossopharyngeal nerve transmits signals to medulla oblongata (afferent neuron) 3. Integration in medulla oblongata. 4. Vagus nerve transmits inhibitory signals to cardiac pacemaker (efferent neuron). 5. Heart rate decreases (effector).

Coupling electrical-mechanical signal in smooth muscle

1. Ca2+ enters via voltage gated channels, stretch, ligand-gated, or secondary messenger (from SR). 2. Ca2+ combines with calmodulin to form Ca-calmodulin complex. 3. Ca-calmodulin binds to MLCK (myosin light chain kinase) which activates it. 4. MLCK phosphorylates myosin light chain (activates it) which leads to myosin ATPase becoming active and undergoing crossbridge cycling for contraction.

Calcitriol actions in kidneys

1. Calcitriol (steroid hormone) acts synergistically with PTH to increase Ca2+ reabsorption - Activates Ca2+-ATPase and Na+/Ca2+ exchanger (basolateral) and Ca2+ channels (lumen) 2. Increases phosphate reabsorption. - Na+/PO4- cotransporter (lumen)

3 mechanisms of increased intracellular calcium in smooth muscle

1. Calcium entry through voltage-gated channels (in response to cell depolarization). 2. Calcium entry through voltage-independent channels (stretch or ligand-gated) 3. Calcium release from SR (G-protein coupled receptors (GPCR)- second messengers)

Hyponatremia- dehydration? 1. Cause: 2. Plasma Na+ concentration: 3. ECF volume: 4. ICF volume:

1. Cause: Adrenal insufficiency, overuse of diuretics. 2. Plasma Na+ concentration: Decrease. 3. ECF volume: Decrease 4. ICF volume: Increase

Hypernatremia-overhydration? 1. Cause: 2. Plasma Na+ concentration: 3. ECF volume: 4. ICF volume:

1. Cause: Cushing's disease; primary aldosteronism 2. Plasma Na+ concentration: Increase 3. ECF volume: Increase 4. ICF volume: Decrease

Hypernatremia-dehydration? 1. Cause: 2. Plasma Na+ concentration: 3. ECF volume: 4. ICF volume:

1. Cause: Diabetes insipidus; excessive sweating. 2. Plasma Na+ concentration: Increase 3. ECF volume: Decrease 4. ICF volume: Decrease

Hyponatremia-overhydration? 1. Cause: 2. Plasma Na+ concentration: 3. ECF volume: 4. ICF volume:

1. Cause: Excess ADH (SIADH); bronchogenic tumors. 2. Plasma Na+ concentration: Decrease 3. ECF volume: Increase 4. ICF volume: Increase

How does a goiter develop with hypothyroidism? What cell/tissue takes up iodide? What transporter is used? Levels of T3/T4? Levels of TRH? Levels of TSH? Why does the thyroid gland grow?

1. Cause: Lack of dietary iodine. 2. Follicular epithelial cells take up iodide. 3. Transporter for Iodide is NIS transporter (Na+/I- symporter) 4. Levels of T3/T4: Decreased. 5. Levels of TRH: Increased 6. Levels of TSH: Increased 7. Why does the thyroid gland grow? Due to increased TSH levels

3 major mechanisms to regulate pH

1. Chemical buffer systems 2. Respiratory system 3. Renal system CO2 + H2O -> H2CO3 -> HCO3- + H+

Movement of solutes: 2 solutions separated by membrane that is permeable by Na+, Cl-, and H20 but not permeable to protein (P-) solution 1: Na+ 100 mmol/L P- 100 mmol/L Solution 2: Na+ 100 mmol/L Cl- 100 mmol/L How would the solutes respond?

1. Cl- diffuses from solution 2 to solution 1 down its concentration gradient. 2. Solution 1 is now more electrically negative compared to solution 2. The membrane voltage drives the diffusion of Na+ from solution 2 to solution 1 (to the more electronegative solution). 3. The accumulation of additional Na+ and Cl- in solution 1 increases its osmolality and causes water to flow from solution 2 to solution 1.

Intracellular receptors

1. Cytosolic receptor: Steroid hormone -> hormone-receptor complex -> Dimer of hormone-receptor complex bound to DNA -> Genomic response. 2. Nuclear receptor: Steroid hormone -> Nuclear receptor -> hormone-receptor complex -> dimer of hormone-receptor complex bound to DNA -> Genomic response.

Changes in volume/osmolarity per factor. 1. Diarrhea. 2. Water deprivation 3. Adrenal insufficiency 4. Infusion of isotonic NaCL 5. High NaCl intake 6. SIADH (excess ADH)

1. Diarrhea: ECF volume decreased 2. Water deprivation: ECF and ICF osmolarity increased ECF and ICF volume decreased 3. Adrenal insufficiency: ECF and ICF osmolarity decreased ECF volume decreased ICF volume increased 4. Infusion of isotonic NaCL ECF volume increased 5. High NaCl intake ECF and ICF osmolarity increased ECF volume increased ICF volume decreased 6. SIADH (excess ADH) ECF and ICF osmolarity decreased. ECF and ICF volume increased.

Receptors in T tubule

1. Dihydropyridine (DHP) receptors- L-type calcium channels (voltage-gated). 2. Ryanodine receptors (RyR)- calcium release channel (ligand-gated) DHP opens RyR via mechanical coupling.

Tyrosin kinase and MAP-K pathway

1. Dimers bind to tyrosin and activate it. 2. Tyrosine kinase activates Ras-GEF. 3. Ras-GEF activates Raf (MAP kinase-kinase-kinase) 4. Raf activates MAP Kinase Kinase. 5. Map kinase Kinase activates Map Kinase. 6. Map Kinase begins gene transcription.

Intestinal phase inhibitory pathways (2)

1. Distension of duodenum; presence of fatty, acidic, hypertonic chyme, and/or irritants in the duodenum (inhibits)-> local reflexes, vagal nuclei in medulla, or pyloric sphincter -> enterogastric reflex -> inhibition of stomach secretory activity. 2. Distension; presence of fatty, acidic, partially digested food in the duodenum -> release of intestinal hormones (secretin, gastric inhibitory peptide, cholecystokinin, vasoactive intestinal peptide) -> inhibition of stomach secretory activity.

What are the 5 amino acid derived hormones?

1. Dopamine (DA) 2. Epinephrine (Epi) aka adrenaline 3. Norepinephrine (NE) aka noradrenaline 4. Thyroxine (T4) 5. Triiodothyronine (T3)

What are the breast feeding hormones and their response to the breastfeeding reflex.

1. Dopamine: inhibited by reflex. 2. Oxytocin: activated by reflex-> activates milk production. 3. Cortisol: Activated by reflex -> activates milk production.

Gastric phase inhibitory pathways (2)

1. Excessive acidity (<pH 2) in stomach (inhibits)-> G cells -> gastrin secretion declines -> inhibition of stomach secretory activity. 2. Emotional upset -> sympathetic nervous system activation -> overrides parasympathetic controls -> inhibition of stomach secretory activity.

2 Types of postsynaptic responses

1. Excitatory postsynaptic potential (EPSP) - depolarizing (mainly Na+ in, some Ca2+). Common receptors: -acetylcholine receptors (AChR), --Glutamate receptors (AMPA-R, NMDA-R, Kainate-R). 2. Inhibitory postsynaptic potential (IPSP). - Hyperpolarizing (K+ out or Cl- in) Common receptors: - Glycine receptors (GlyR) - y-Aminobutyric acid (GABA) receptors (GABAa-R) These are both types of graded potentials. Postsynaptic responses depend on the receptors.

Signal amplification via second-messenger pathway

1. Extracellular chemical messenger binds to membrane receptor. (1 molecule) -> 2. Activated adenylyl cyclase (10 molecules) -> 3. Cyclic AMP (1000 molecules)-> 4. Activated protein kinase (1000 molecules)-> 5. Phosphoylated (activated) protein (eg. an enzyme) (100,000 molecules)-> 6. Products of activated enzyme (10,000,000 molecules) Go from 1 molecule to 10,000,000 molecules via signal amplification.

Pathway of thyroid hormone in cell

1. Free extracellular hormone enters the cell. 2. Once inside, T4 is deiodinated so that T4 and T3 levels are equal 3. TRs (thyroid hormone receptors) bind to nuclear DNA on the TRE (Thyroid response element) in the promoter region of genes regulated by T3 and T4 4. Binding of hormone to the thyroid hormone receptor regulates transcription The thyroid hormone receptors have a 10 fold greater affinity for T3 than T4, making T3 more potent. Action of thyroid hormones on target cells. Free extracellular T4 and T3 enter the target cell. Once T4 is inside the cell, a cytoplasmic 5′/3′-monodeiodinase converts much of the T4 to T3, so cytoplasmic levels of T4 and T3 are about equal. TRs (thyroid hormone receptors) bind to nuclear DNA at thyroid response elements in the promoter region of genes regulated by thyroid hormones. The binding of T3 or T4 to the receptor regulates the transcription of these genes. Of the total thyroid hormone bound to receptor, ~90% is T3. The receptor that binds to the DNA is preferentially a heterodimer of the TR (thyroid hormone receptors) and RXR (retinoid X receptor).

Actin strand formation (thin filaments) steps

1. G-actin molecule activated via ATP. 2. ATP-bound G-actin form stable actin oligomer. 3. Nucleus formation 4. Assembly into F-actin filament with the removal of phosphate producing ADP actin.

Gaq pathway

1. Gaq binds to phospholipase C 2. Phospholipase C converts PIP2 (phosphatidylinositol 4,5- bisphosphate to IP3 (inositol triphosphate) and DAG (Diacylglycerol) 3. IP3 stimulates calcium release from the sarcoplasmic reticulum (in muscles, ER in non-muscle cells) 3. DAG activates protein kinase C

Intracellular receptors: genomic pathway vs non-genomic pathway

1. Genomic pathway: Sex steroid hormone -> sex hormone receptor (forms complex) -> nuclear translocation (goes through the nuclear membrane to the nucleolus) -> Dimerization with another sex hormone-receptor complex -> gene transcription. 2. Non-genomic pathway (MAPK pathway): Sex steroid hormone -> sex hormone receptor (forms complex) -> eNOS -> NO release or -> Kinase -> protein into nucleolus -> gene transcription.

Insulin release from beta cell pathway

1. Glucose enters through GLUT2 transporter of pancreatic beta cell via facilitated diffusion. 2. Glucose undergoes glycolysis which leads to an increase in ATP or increase in ATP/ADP ratio. 3. The increased ATP and/or ATP/ADP inhibits an ATP sensitive K+ channel 4. Inhibition of this K+ channel causes the membrane to depolarize. 5. Depolarization activates a voltage-gated Ca2+ channel in the plasma membrane. 6. Ca2+ influx into cell which in turn evokes Ca2+ induced Ca2+ release on the endoplasmic reticulum. 7. The elevated Ca2+ leads to exocytosis and release of insulin (within secretory granules) into the blood.

Nutrient handling terminology

1. Glycogenesis 2. Glycogenolysis 3. Glycolysis 4. Gluconeogenesis

Ga stimulatory pathway

1. Hormone stimulates receptor. 2. GDP (inactive) G protein moves to activated receptor. 3. GDP G protein turns to GTP G protein (activated). 4. G alpha separates from beta-gamma subunits. 5. G alpha stimulatory subunit binds with adenylyl cyclase. 6. Activated adenylyl cyclase converts ATP to cAMP. 7. cAMP activated protein kinase A. 8. Protein kinase A phosphorylates proteins. cAMP broken down to 5' AMP via phosphodiesterase.

Endocrine hormones secreted by: 1. Hypothalamus 2. Anterior pituitary 3. Posterior pituitary 4. Parathyroid 5. Thyroid 6. Liver 7. Heart 8. Adrenal 9. Kidney 10. Pancreas 11. GI tract 12. Fat 13. Testis 14. placenta 15. Ovary

1. Hypothalamus: Releasing/inhibiting hormones. 2. Anterior pituitary: ACTH, TSH, GH, LH, FSH, Prolactin 3. Posterior pituitary: ADH, oxytocin 4. Parathyroid: PTH 5. Thyroid: T3, T4, Calcitonin 6. Liver: IGF-1 7. Heart: ANP, BNP (won't test) 8. Adrenal: Corticosteroids, Epi, NE 9. Kidney: Renin; 1,25(OH)2-D3 (calcitriol) 10. Pancreas: Insulin, glucagon 11. GI tract: CCK, VIP 12. Fat: Leptin 13. Testis: Testosterone, inhibin 14. placenta: hCG, estrogen, progesterone 15. Ovary: Estrogen, Progesterone, inhibin, activin

What factors increase lymph flow (6)

1. Increase Pc 2. Decrease πc 3. Increase πi 4. Decrease Pi 5. Increase Capillary surface area 6. Increase Capillary permeability (histamine, burns) (Many of these factors are same as those that favor capillary filtration).

Factors that influence the magnitude of K+ secretion

1. Increase in Na+/K+ ATPase activity increases K+ secretion. (Can be stimulated by aldosterone, collecting duct principle cells) 2. High plasma K+ concentration in plasma increases K+ secretion. 3. Increased Na+ in collecting duct lumen (due to loop diuretic) -> increased Na+ entry into principle cells -> increased Na+/K+ ATPase activity -> increased K+ secretion. 4. High fluid flow rate through the collecting duct lumen favors K+ secretion. 5. Increase permeability of luminal cell membrane favors K+ secretion. 6. Lumen-negative transepithelial electrical potential promotes K+ secretion.

What stimulates/inhibits thirst.

1. Increase in plasma osmolality -> osmoreceptors -> increase thirst. 2. Decrease in blood volume -> baroreceptors -> Increase thirst. 3. Decrease in blood volume -> baroreceptors -> Increase Renin -> Increase Angiotensin II -> Increase thirst. 4. Dryness of mouth and throat -> increase in thirst. ---- 5. Monitoring of water intake by GI tract -> inhibit thirst.

Stretching of cardiomyocytes results in?

1. Increased sensitivity of troponin for binding Ca2+. 2. Increased Ca2+ release from SR 3. Decreased spacing between thin/thick filaments More cross-bridges = more force

Calcitriol actions in the intestine

1. Increases Ca2+ reabsorption 2. Increases phosphate reabsorption

Effects of cortisol on overall flow of fuels

1. Increases blood glucose (spares glucose for neural tissue) 2. Increases fatty acids in blood for energy. Muscle: decrease protein synthesis, increase protein degradation (late fasting state), and decreased glucose uptake (fasting state) Liver: Increased gluconeogenesis (fasting state) and Increased glycogen synthesis (late fasting state). Adipose: Decreased glucose uptake (fasting state) and increased lipolysis (late fasting).

4 phases of the cardiac cycle

1. Inflow phase: - Rapid ventricular filling - Reduced ventricular filling - Atrial contraction 2. Isovolumetric contraction 3. Outflow phase: - Rapid ejection - Reduced ejection 4. Isovolumetric relaxation

5 actions of glucagon

1. Inhibits glucose uptake into target tissues 2. Promotes glycogenolysis and gluconeogenesis. 3. Promotes protein degradation 4. Promotes lipolysis (via HSL) and ketogenesis 5. Inhibits glycolysis in liver.

Indirect measurements of fluids?

1. Intracellular fluid: ICF= Total Body Water - Extracellular Fluid 2. Interstitial fluid: ISF= Extracellular Fluid - Plasma 3. Blood volume: BV = Plasma/ (1 - Hematocrit)

Types of extracellular receptors (3)

1. Ionotropic receptors (ion channel-linked). Example: Nicotinic receptor, IP3 receptors (binds to sarcoplasmic reticulum and opens ion channel releasing Ca2+). 2. G protein-coupled receptors (GPCR, 7 transmembrane domains) 3. Catalytic receptors (enzyme-linked) Example: RTK (receptor tyrosine kinase)

Cell signaling types (6)

1. Juxtacrine signaling- Cell-to-cell through gap junctions. 2. Autocrine- Cell releases signaling molecule that directly impacts the same cell (itself). 3. Paracrine- An adjacent cell releases a signaling molecule that impacts a neighbor cell. (ex. Nitrous Oxide) 4. Nervous (form of paracrine signaling)- Neuron forms a synapse that releases a neurotransmitter directly onto a cell. 5. Endocrine- Cell produces a signaling molecule (hormone) that is released into the circulatory system. 6. Neuroendocrine (form of endocrine signaling)- A neuron releases a signaling molecule into the circulatory system.

Stepwise approach to diagnosing acid base disorders

1. Look at pH: determine if acidosis or alkalosis. 2. Look at HCO3- and determine if it would explain the pH. 3. Look at PCO2 and determine if it would explain the pH 4. Distinguish the initial change from the compensatory response - Is compensation explained by PCO2 or HCO3- value not associated with pH change?

Cephalic phase inhibitory pathway (1)

1. Loss of appetite, depression (inhibits) -> cerebral cortex (inhibits) -> lack of stimulatory impulses to parasympathetic center (inhibits)-> inhibits stomach secretory activity.

What is RAS activated by?

1. Loss of blood volume 2. Drop in blood pressure (detected by baroreceptors in carotid sinus) 3. Decreased filtrate flow rate in glomerulus

Endosome/lysosome formation steps

1. Lysosome hydrolase precursor enters cis-Golgi from the ER. 2. Phosphate is added, forming mannose-6-phasphate (M6P). 3. M6P then binds to M6P receptor in the golgi and enters a budding vesicle. 4. Receptor-dependent transport of vesicle from golgi to prelysosomal endosome occurs. 5. An H pump acidifies the endosome resulting in dissociation of M6P (removal of phosphate). 6. M6P receptors organize within a budding vesicle and leave the late endosome/prelysosomal endosome. 7. The receptors recycle back into the trans golgi

Renin-Angiotensin-Aldosterone system pathway

1. Macula densa in distal convoluted tubule senses *low fluid flow or low Na+ concentration* and signals the juxtaglomerular cells to secrete renin. 2. Renin (enzyme) released into blood stream, liver releases angiotensinogen into the blood stream. Renin converts angiotensinogen to angiotensin I. 3. Angiotensin I converted to angiotensin II by the angiotensin-converting enzyme (ACE) released by the lungs in pulmonary bloodstream. 4. Angiotensin II stimulates the adrenal cortex to secrete aldosterone. 5. Aldosterone stimulates Na+ uptake on the apical cell membrane (principle cells) in the distal convoluted tubule and collecting ducts. (ADH causes aquaporins to move to the collecting duct plasma membrane, which increases water reabsorption.)

Calcium release from SR steps (3)

1. Membrane depolarization opens the L-type Ca2+ channel. 2. Mechanical coupling between the L-type Ca2+ channel and the Ca2+ release channel causes the Ca2+ release channel to open. 3. Ca2+ exits the SR via the Ca2+ release channel and activates troponin C, leading to muscle contraction. Ca2+ entering the cell via L-type Ca2+ channels also can activate the Ca2+ release channels. However, this pathway is not essential is skeletal muscle.

Endocytosis steps

1. Molecule begins in solution outside cell. 2. Receptor-mediated endocytosis or not receptor-mediated endocytosis. 3. A clathrin-coated endocytotic vesicle carries the endocytosed material within cell. (small indentations in the plasma membrane called CAVEOLAE can mediate clathrin-independent endocytosis) 4. Acidification of the endosome dissociates the ligand and its receptors. 5. Receptors are recycled to the plasma membrane.

What do the intestines secrete, what is it secreted by, action, and stimulus for secretion?

1. Motilin (hormone) secreted by M cells (antrum and duodenum). Increases myoelectric activity of stomach and duodenum; acts on smooth muscle of gastric antrum and duodenum frequency of spike potential on slow waves -> helps coordinate interdigestive motility patterns (MMC) and increases rate of gastric emptying. Stimulus for secretion is acid in the duodenum but not a meal and vagal stimulation. 2. CCK (hormone) released by I cells of the duodenum and jejunum. Stimulates gallbladder contraction and relaxation of spincter of Oddi. Stimulates pancreatic enzymes (binds to receptor on acinar cells). Also potentiates effect of secretin on pancreatic bicarb secretion. Stimulates growth of pancreas/gallbladder. Inhibits gastric emptying. Stimulus for secretion is small peptides and amino acids and fatty acids. 3. Secretin (hormone) released by S cells of duodenum. Increase pancreatic bicarb. Increase biliary bicarb. Decrease HCl. Stimulus for secretion is HCl in duodenum and fatty acids in duodenum. Inhibited by alkaline pH in duodenum.

GI processes (5)

1. Motility 2. Secretion 3. Absorption 4. Circulation 5. Control (neutral and hormonal)

What are the 5 GI processes?

1. Motility 2. Secretion 3. Absorption 4. Circulation 5. Control (neural and hormonal)

Solute concentrations inside/outside cell: 1. Na+ 2. K+ 3. Ca2+ 4. Cl- 5. HCO3-

1. Na+ out: 135-145 mEq/L in: 10-15 mEq/L 2. K+ Out: 3.5-5.0 mEq/L In: 120-150 mEq/L 3. Ca2+ Out: 1.1 -1.4 (ionized) mmol/L (2.1-2.8 total) In: .0001 (ionized) mmol/L 4. Cl- Out: 95-105 mEq/L In: 20-30 mEq/L 5. HCO3- Out: 22-28 mEq/L In: 12-16 mEq/L

1. Na+ 2. K+ 3. Ca2+ 4. Mg2+ 5. Cl- 6. HCO3- 7. H2PO^3-/HPO4^2- 8. Proteins 9. Glucose 10. pH 11. Osmolality Following solutes outside and inside cell

1. Na+ ~145 outside 15 inside 2. K+ ~4.5 outside 120 inside 3. Ca2+ ~1.2 (ionized) outside .0001 (ionized) inside 4. Mg2+ ~.6 (ionized) outside 1 (ionized) inside 5. Cl- ~110 outside 20 inside 6. HCO3- ~24 outside 16 inside 7. H2PO^3-/HPO4^2- ~.75 (ionized) outside .7 (free) 8. Proteins 1 outside (interstitium) 30 inside 9. Glucose ~5.9 outside Very low inside 10. pH 7.4 outside ~7.2 inside 11. Osmolality 290 outside 290 inside

1. Driving force of Na+ when at +60 equilibrium with -70 mV inside cell? 2. Diving force of K+ when at -90 equilibrium with -70 mV inside cell?

1. Na+ driving force is 130 mV 60 - -70 = 60 + 70 = 130 2. K+ driving force is 20 mV. -90 - -70 = 20

Termination of contraction (3) ways Ca2+ removed

1. Na-Ca exchanged and Ca2+ pump in plasma membrane extrude Ca2+ from the cell. 2. SERCA (sarco/endoplasmic reticulum Ca2+- ATPase): Sequesters Ca2+ within the sarcoplasmic reticulum using ATP. 3. Ca2+ is bound in the sarcoplasmic reticulum by calreticulin and calsequestrin.

Feedback regulation

1. Negative feedback loop. 2. Positive feedback loop 3. Complex, multilevel feedback loop. RH: releasing hormone TH: trophic hormone EH: endocrine cell hormone

Endothelial influences (3) short-term intrinsic

1. Nitric Oxide (NO): Vasodilator 2. Prostacyclin (PGI2): Vasodilator 3. Endothelin-1 (ET-1): Vasoconstrictor.

Excitation-Contraction coupling of cardiomyocytes

1. Pacemaker cell creates current, current spreads through gap junctions to contractile cell. 2. Action potentials travel along plasma membrane and T tubules. 3. Action potential enters from adjacent cell. 4. Voltage-gated Ca2+ channels open. Ca2+ enters cell. 5. Ca2+ induces Ca2+ release through ryanodine receptor-channels (RyR). 6. Local release causes Ca2+ spark. 7. Summed Ca2+ sparks create a Ca2+ signal. 8. Ca2+ ions bind to troponin to initiate contraction. 9. Relaxation occurs when Ca2+ unbinds from troponin. 10. Ca2+ is pumped back into the sarcoplasmic reticulum for storage via SERCA (uses ATP) 11. Ca2+ is exchanged with Na+ by the NCX antiporter 12. Na+ gradient is maintained by the Na+/K+ ATPase.

Initiation of AP in cardiac muscle steps (8)

1. Pacemaker cell creates current, current spreads through gap junctions to contractile cell. 2. Action potentials travel along plasma membrane and T tubules. 3. Ca2+ channels open in plasma membrane and SR. 4. Ca2+ binds to troponin, exposing myosin-binding sites. 5. Crossbridge cycle begins (muscle fiber contracts).

What 3 factors regulate Stroke Volume?

1. Preload (sarcomere length)- Initial stretch of the ventricles at the end of diastole. Is related to EDV (the amount of volume of blood in the ventricles at the end of diastole) Sometimes related to EDP. (EDV and EDP in the left ventricle). - Increase in preload causes increase in stroke volume. 2. Afterload- related to the pressure the ventricle must generate in order to eject blood into the aorta during systole (frequently related to aortic pressure) - Increase in afterload causes decrease in stroke volume. 3. Inotropy- "Contractility". alteration of the force of muscle contractions (Positive inotropes strengthen force of contraction) - Increase in contractility causes an increase in stroke volume.

Sodium Balance and Sympathetic Stimulation

1. Produces a decline in GFR and renal blood flow, leading to a decreased filtered Na+ load and peritubular capillary hydrostatic pressure, both of which favor sodium reabsorption and thus diminished Na+. (much like macula densa cells and afferent arteriole constriction.). 2. Directly causes Na+ reabsorption in the renal tubules to increase by increasing diffusion of Na+ 3. Causes renin release, which results in increased plasma angiotensin II and aldosterone levels, both of which increase tubular Na+ reabsorption.

Insulin Actions on hepatocytes

1. Promotes glycogen synthesis 2. Enhances glycolysis (glucose used for ATP) 3. Inhibits glycogenolysis and gluconeogenesis 4. Promotes lipogenesis and protein synthesis.

List major reabsorption/secretion of each part of the nephron/collecting duct.

1. Proximal tubule: Reabsorption: >70% of filtered water and solutes including Na+ (70%), Cl-, HCO3-, and K+. All glucose and amino acids. Secretion: H+, Organic acids/anions (PAH and penicillin), organic bases/cations (amine and ammonium). 2. Descending loop of Henle: Reabsorption: H2O 3. Ascending loop of Henle: Reabsorption: Na+ (20%), Cl-, K+, Ca2+, HCO3-, Mg2+. Secretion: H+ (hypoosmotic, impermeable to water) (Loop diuretics = Furosemide, Ethacrynic acid, Bumetanide). 4. Distal convoluted tubule: Reabsorption: Na+, Cl-, Ca2+, Mg2+ (Impermeable to water) (thiazides loop diuretic) 5. Late distal tubule and collecting duct: Reabsorption: Na+, Cl-, (+ADH) H2O, HCO3-, K+. Secretion: H+ and *K+* Most Mg2+ reabsorption in ascending tubule (65%). Most K+ secretion in the cortical collecting duct via collecting duct principal cells.

Catalytic receptors types

1. Receptor guanylyl cyclases: Catayze GTP to cGMP. 2. Receptor serine/threonine kinsases: Phosphorylate serine or threonine residues on cellular proteins. 3. Receptor tyrosine kinases (RTKs): Phosphorylate tyrosine residues on themselves and other proteins. 4. Tyrosine kinase: Associated receptors interact with cytosolic (i.e. not membrane-bound) tyrosine kinases. 5. Receptor tyrosine phosphatases: cleave phosphate groups from tyrosine groups of cellular proteins.

Pathway through the nephron

1. Red - Glomerulus 2. Orange - Proximal convoluted tubule 3. Green - Proximal straight tubule 4. Blue - Thin descending limb 5. Blue - Thin ascending limb 6. Purple - Thick ascending limb 7. Yellow - Distal convoluted tubule

Functions of the kidneys (8)

1. Regulate osmotic pressure. 2. Regulate ion concentration. 3. Acid-Base balancing (H+/HCO3-) 4. Regulate extracellular fluid levels (Na+) 5. Regulate arterial pressure. 6. Eliminate waste products 7. Remove drugs (when people have multiple organ failure/kidney failure, prescription drugs become a problem because you can't excrete it) 8. Hormone production

Mechanisms to maintain venous return (4)

1. Regulate venous compliance (sympathetic nervous system): Increase in sympathetic nervous system = increase venoconstriction = decrease venous compliance = increases venous return. 2. Venous valves: Make sure that the blood goes one way back to the heart, defect in valves, like in vericose veins, causes blood to pool back and increases the size of the vein (that's why you can see them). Decrease venous return when not working. 3. Skeletal muscle pump: Contraction creates the pressure needed to open up the valve above it and allow the blood to move upwards (valve below stays closed). Increase in skeletal muscle pump = increase in venous pressure = increase in venous return. 4. Respiratory pump: When breathing in, the diaphragm moves down and the chest expands lowering the pressure in the chest as a result. The new pressure difference between chest (lower) and rest of body pushes blood in the veins towards the heart. Decrease pressure in chest = increase in pressure gradient = increase in venous return.

Inhibition of RAS (4)

1. Renin inhibitor 2. ACE inhibitor 3. Angiotensin II receptor blocker (ARB) (blocks angiotensin II to angiotensin II type 1 receptor) 4. Aldosteone inhibitor

Example of secondary active transporters

1. SGLT1:Na+/glucose cotransporter 2. Phosphate cotransporter 3. Na+/iodide symporter 4. Na+/K+/Cl- cotransporter 5. Na+/Cl- cotransporter 6. K+/Cl- cotransporter

Renal Control of pH 4 mechanisms

1. Secreting H+ (when pH acidic) 2. Conserving (reabsorbing) HCO3- (when pH acidic). 3. Generating new HCO3- when excess acid has been added 4. Excreting HCO3- when excess base has been added.

What are the 2 cells involved in salivary glandular secretions and what do they secrete?

1. Serous cells: amylase, antibodies, lysozymes 2. Mucous cells: glycoprotein mucins.

Expiration (2 phases)

1. Shortly after the abrupt termination of inspiratory signal, some accessory inspiratory muscles are activated to control outflow and dampen the speed of expiration. 2. All inspiratory muscles are turned off and passive recoil of the lung causes outflow of air more rapidly. (Expiratory muscles like the abs can be activated if necessary).

Cephalic phase stimulatory pathways (2)

1. Sight and thought of food -> cerebral cortex -> (conditioned reflex)-> hypothalamus and medulla oblongata -> Vagus nerve -> stomach secretory activity. 2. Stimulation of taste and smell receptors -> hypothalamus and medulla oblongata -> Vagus nerve -> stomach secretory activity.

What kind of transporters are these: 1. Sodium glucose cotransporter 2. Glucose transporter (Glut2) 3. Na-K-ATPase 4. Amino acid transporter 5. Na-Amino acid cotransporter

1. Sodium glucose cotransporter: secondary active transport 2. Glucose transporter (Glut2): Carrier mediated diffusion. 3. Na-K-ATPase: Primary active transport. 4. Amino acid transporter: Amino acid cotransporter 5. Na-Amino acid cotransporter: Secondary active transport

What does the stomach secrete, what is it secreted by, action, and stimulus for secretion?

1. Somatostatin (paracrine) secreted by D cells inhibits release of all GI hormones and inhibits H+ secretion. Stimulus for secretion is HCl in the lumen. Inhibited by vagal stimulation and by gastrin. 2. Histamine (paracrine) is secreted by mast (ECL) cells of gastric mucosa. Increases gastric H+ secretion directly and by potentiating effects of gastrin and vagal stimulation. Stimulus for secretion is gastrin. 3. GRP (gastrin releasing peptide) (neurocrine) secreted by vagus nerves that innervate G cells. Stimulates gastrin release from G cells. 4. HCl secreted by parietal cells (fundus). Stimulus for secretion is gastrin, vagal stimulation, and histamine. Inhibited by low stomach pH, chyme in duodenum (via secretin/GIP (gastric inhibitory polypeptide)), atropine, and omeprazole. 5. Intrinsic Factor secreted by parietal cells. 6. Pepsinogen secreted by chief cells. Active pepsin digests proteins. Stimulus for secretion is vagal stimulation. 7. Gastric lipase secreted by chief cells. Causes lipolysis (breakdown of triglycerides for absorption in intestines.) 8. Mucous secreted by surface mucous cells and mucous neck cells. Stimulus for secretion is vagal stimulation. 9. Gastrin (hormone) is secreted by G cells (antrum and duodenum). Stimulates HCl secretion, stimulates growth of gastric mucosa, stimulates gastric motility, and stimulates release of somatostatin. Stimulus for secretion is small peptides and amino acids, distention of the stomach, and vagus (via GRP). Inhibited by low pH in stomach and somatostatin.

Hormone characteristics

1. Specificity (cellular receptors): Endocrine cell will secrete hormone into the blood and only the cells with the receptor (target cell) will stimulate a response. 2. Amplification: A small stimulus -> hormone (1 mol.) 1st messenger -> target hormone 2nd messenger (10 mol.) -> 3rd messenger (1,000 mol) -> target proteins, e.g channel (10,000 mol) -> great effect. 3. Multiplication: The same hormones can have multiple effects on a target tissue. The same hormone can have effects on different tissues. Several hormones may regulate the same tissue. 4. Pulsatile/Rhythmic secretions: Example: cortisol peak secretion right before you wake up (6-8 am). Growth hormone peaks in the middle of the night while sleeping. 5. Feedback regulation

Function of titin in sarcomere (3)

1. Stabilize myosin (center between thin filament) 2. Prevent overstretching of sarcomere 3. Recoil sarcomere after it is stretched

Cross-bridge cycle steps

1. Starting here, the myosin head is bound to ATP and is in its low-energy configuration. 2. Myosin head hydrolyzes ATP to ADP and Pi and is in its high energy configuration 3. Myosin head binds to actin, forming a cross-bride with the thin filament. 4. Myosin couples release of ADP and Pi to a power stroke that slides the thin filament along the myosin and returns the myosin head to a low-energy state.

PTH actions at the kidney

1. Stimulates Ca2+ reabsorption at distal tubule and loop of henle. 2. Increases phosphate excretion 3. Activates renal 1-hydroxylase.

Gastric phase stimulatory pathways (2)

1. Stomach distention activates stretch receptors -> vagovagal reflexes -> medulla -> vagus nerve -> stomach secretory activity. or Stomach distension activates stretch receptors -> local reflexes -> stomach secretory activity. 2. Food chemicals (especially peptides and caffeine) and rising pH activate chemoreceptors -> G cells -> gastrin release to blood -> stomach secretory activity.

GI Motility: Stomach functions

1. Storage/Reservoir 2. Mixing 3. Emptying

What is the stimulus for CRH and ACTH release?

1. Stress (low blood glucose) 2. Circadian rhythm Cortisol peak release when waking up (6-8 am)

Percentage of water in the body per: 1. Total body water 2. ICF 3. ECF 4. Interstitial fluid 5. Plasma (PV) 6. Transcellular fluid 7. Blood (BV)

1. TBW= 60% of body weight in males, 50% in females. 2. ICF= 60% of TBW 3. ECF= 40% of TBW 4. Interstitial= 75% of ECF 5. Plasma = 20% of ECF 6. Transcellular fluid = 5% of ECF 7. Blood = Plasma volume/(1-Hematocrit)

Indicator Dilution Method to accessing fluid levels in body

1. Take indicator with known mass/volume/concentration and inject it into compartment where it will only diffuse in specific area. 2. Come back later and draw blood. 3. The known indicator Mass A= Indicator Mass B. Since Mass = volume x concentration. You know that Mass A and the sample taken from blood (Mass B) are equal (both plasma) and now you need to find the concentration of B and extrapolate it to figure out the Volume of B. Volume B= (Volume A x Concentration A)/ Concentration B Make sure to take into account loss.

Match the description with the phase of the cardiac cycle: 1. The QRS complex of the EKG coincides with this phase. 2. This phase begins when the mitral valve opens 3. This phase has a large rise in ventricular pressure 4. This phase begins with closure of the aortic valve.

1. The QRS complex of the EKG coincides with this phase. Isovolumetric contraction and Rapid ejection 2. This phage begins when the mitral valve opens Rapid ventricular filling 3. This phase has a large rise in ventricular pressure Isovolumetric contraction and Rapid ejection 4. This phase begins with closure of the aortic valve. Isovolumetric relaxation

Another countercurrent multiplication example

1. The solution moves into the tubule 2. Salt is pumped out of the ascending limb into the interstitial space 3.Water from the descending limb leaves to the interstitial space (tonicity) 4. Fluid continues moving forward in the tubule pushing the preceding solution 5. Repeat step 2 6. Repeat step 3 7. Repeat step 4

3 different patterns for distribution of sympathetic axons:

1. To the body wall 2. To viscera above the diaphragm 3. To viscera below the diaphragm

Indicators for different compartments: 1. Total body water 2. Extracellular fluid 3. Plasma volume 4. Blood volume

1. Total: isotopes of H20. Antipyrine. or calculated from Total= intracellular + Extracellular 2. Extracellular: Isotopes of Na and I-iothalamate. Thiosulfate, inulin. 3. Plasma: Isotope of I-albumin, Evans blue dye (T-1824). 4. Blood: Cr-labeled red blood cells or calculated as blood volume= plasma volume/ (1-hematocrit)

What are the 7 zymogenic secretion of the pancreas?

1. Trypsin 2. Chymotrypsin 3. Lipase 4. Carboxypeptidase 5. Elastases 6. Nucleases 7. Pancreatic amylase All are released as proenzymes and activated in the duodenal lumen. *First, trypsinogen gets activated to trypsin via enterokinase, then trypsin activates all the other zymogens*.

Thyroid hormone synthesis

1. Tyrosine + I2 (iodine) -> Monoiodotyrosine (MIT) and Di- iodotyrosine (DIT) 2. DIT + DIT -> Thyroxine (T4) 3. DIT + MIT -> 3, 5 ,3'-triiodothyronine (T3) and 3,3', 5'-triiodothyronine (reverse T3) Thyroxine (T4); calorigenic T3; calorigenic rT3; non-calorigenic

How are the parietal cells regulated?

1. Vagus -> ACh -> M3 receptor (blocked by atropine) -> Gq -> IP3/Ca2+ -> H+/K+ ATPase -> H+ secretion (blocked by omeprazole). 2. G cells -> gastrin -> CCKb receptor -> Gq -> IP3/Ca2+ -> H+ secretion (blocked by omeprazole). 3. ECL cells -> Histamine -> H2 receptor (blocked by Cimetidine) -> Gs -> cAMP -> H+ secretion (blocked by omeprazole). 4. Somatostatin or Prostaglandins -> Gi -> blocks cAMP

Systemic actions of angiotensin II (6)

1. Vasoconstriction 2. Retension of Na+/H2O 3. Release of ADH from posterior pituitary 4. Secretion of aldosterone from adrenal cortex (Na+/H2O retention in kidney) 5. Stimulation of SNS 6. Stimulation of thirst

G-protein coupled receptor (GPCR) mediated regulation of smooth muscle

1. Via epinephrine/norepinephrine binding B-androgenic receptor GPCR -> G alpha and adenylate cyclase -> cAMP -Causes: Smooth muscle relaxation, vasodilation. 2. Via epinephrine/norepinephrine binding A-adrenergic receptor GPCR -> Gaq -> phospholipase C -> IP3 -> Ca2+ released from sarcoplasmic reticulum -> form calcium calmodulin complex. -Causes: Smooth muscle contraction, vasocontriction. (also DAG -> PKC causes contraction, vasocontriction)

Different types of ion channel gates (4)

1. Voltage-gated: responds to membrane potential. 2. Ligand-gated (extracellular ligand): responds to neurotransmitters, drugs. 3. Ligand-gated (intracellular ligand): Responds to intracellular cAMP, cGMP, Ca2+, ATP, etc. 4. Mechanically gated: Responds to pressure, temperature.

A 27-year-old female presents to her primary care physician complaining of fatigue ever since she was in a motorcycle accident 3 months ago. History and physical examination: She had been in good health prior to the accident. She stated that during the accident, she sustained only minor injuries, but the impact was strong enough to break her helmet. She says she has just not felt right since the accident and is tired all the time. She has gained 10 lbs., and drinks and urinates more than before the accident. She also states that she has been constipated since the accident. On physical examination, her skin is cool, dry, and rough. There is also a milky discharge coming from her breast. With further questioning, the patient admits to not menstruating since the accident. Glucose at 54 (60-109) Cortisol at 2 (3-15) Thyroxine at 3 (5-12) Prolactin at 70 (<20) IGF-1 at 64 (90-360) FSH at 2 (4-30) LH at 2 (5-30) TSH at <1 (1-10) 1. What is a probable diagnosis for this patient based on presentation and clinical data? 2. What is the mostly likely cause of this patient's condition? 3. What would be the expected changes in the following hormones compared to before the accident based on your diagnosis? GH, Estrogen, ACTH, CRH, TRH, Somatostatin. 4. Why is the patient constantly drinking and urinating? 5. What do you predict is the patient's plasma osmolarity 6. Based on your diagnosis, explain the patient's weight gain. 7. Based on your diagnosis, explain the patient's fasting blood glucose 8. Why is she lactating?

1. What is a probable diagnosis for this patient based on presentation and clinical data? - Panhypopituitarism: Inadequate or absent production of hormones from anterior pituitary. -- Reduce/destroy anterior pituitary function -- Interfere with secretion of releasing hormones in hypothalamus. 2. What is the mostly likely cause of this patient's condition? - Accident severed the pituitary stalk. 3. What would be the expected changes in the following hormones compared to before the accident based on your diagnosis? GH, Estrogen, ACTH, CRH, TRH, Somatostatin. - GH goes down, Estrogen down, ACTH down, CRH up, TRH up, Somatostatin down (no GH, IGF-1, or TH to stimulate it). 4. Why is the patient constantly drinking and urinating? - Lack of ADH: central diabetes insipidus. Low blood volume could also explain the low blood pressure (95/60). 5. What do you predict is the patient's plasma osmolarity Higher than normal 6. Based on your diagnosis, explain the patient's weight gain. - Hypothyroidism: lack of thyroid hormones. 7. Based on your diagnosis, explain the patient's fasting blood glucose. - Low due to reduction of cortisol and IGF-1. 8. Why is she lactating? Dopamine from the hypothalamus is no longer reaching the AP- lactotropes will secrete prolactin.

GI Motility: Esophagus - Deglutition steps

1. When a person is not swallowing, the esophageal sphincter muscle is contracted, the epiglottis is up, and the glottis is open, allowing air to flow through the trachea to the lungs. 2. The swallowing reflex is triggered when a bolus of food reaches the pharynx. 3. The larynx, the upper part of the respiratory tract, moves upward and tips the epiglottis over the glottis preventing food from entering the trachea. 4. The esophageal sphincter relaxes, allowing the bolus to enter the esophagus. 5. After the food has entered the esophagus, the laryng moves downward and opens the breathing passage. 6. Waves of muscular contraction (peristalsis) move the bolus down the esophagus to the stomach.

What are the 2 pancreatic secretions?

1. Zymogens (inactive enzymes) for digestion. 2. Aqueous bicarbonate secretion to neutralize stomach acid.

synaptic transmission steps

1. action potential arrives at the axon terminal 2. voltage gated Ca 2+ channels open 3. Ca 2+ enters the cell 4. Ca 2+ signals to vesicles 5. vesicles move to the membrane 6. docked vesicles release neurotransmitter by exocytosis 7. neurotransmitter diffuses across the synapse and binds to receptors 8. Enzymes (ex. acetylcholinesterase) break down/inactivate the neurotransmitter and transport proteins transport them to cytoplasm to be recycled. The vesicle is transported by clathrin back into cytoplasm.

What is the function of each receptor matching with below? 1. α1 adrenergic 2. β1 adrenergic 3. Muscarinic 4. β2 adrenergic Gastric acid secretion Bronchodilation Increased heart rate Vascular smooth muscle contraction Increased salivation

1. α1 adrenergic: Vascular smooth muscle contraction. 2. β1 adrenergic: Increased heart rate 3. Muscarinic: Gastric acid secretion. Increased salivation. 4. β2 adrenergic: Bronchodilation.

Pleural space

10 micrometers thick space between the lungs and the chest wall that contains fluid that allows the lungs to slide against the chest wall.

Force-velocity relationship in muscle

2 factors: The less load the faster the muscle can contract. The higher the sarcomere length the faster the muscle can contract (has to do with heart/ventricles). The more load, the smaller the maximum velocity/shortening velocity. Increase initial length of muscle and it will shorten faster.

Renal blood supply

20% of cardiac output Afferent and Efferent arterioles Glomerulus Vasa Recta Peritubular capillaries The blood supply to the kidneys serves several functions. Primarily it brings blood to the kidneys to be filtered. It also acts to reabsorb solutes after they've been filtered, and to allow for the secretion of water and other solutes later on. It works to balance fluid levels. The kidney microvasculature graph: The left side (red) shows the arterial vessels, glomeruli, and capillaries. A cortical radial artery arises from an arcuate artery (white arrow) and gives rise to afferent arterioles, which supply the glomeruli (red balls). Efferent arterioles leave the glomeruli and give rise to the extensive peritubular capillary network that surrounds tubules in the cortex. The efferent arterioles of juxtamedullary glomeruli give rise to peritubular capillaries directly and indirectly via the vasa recta that descend into the medulla. The right side of the figure (blue), which may be superimposed on the left side, depicts the venous vessels. Ascending vasa recta drain into interlobular veins or arcuate veins. In the outer medulla, many of the ascending and descending vasa recta are grouped together in vascular bundles; this facilitates the exchange of substances between blood fl owing into and out of the medulla.

GI Motility: Small Intestine What are the 3 functional components of the small intestine? What are the 2 motility patterns?

3 Functional Components: 1. Duodenum 2. Jejunum 3. Ileum 2 motility patterns: 1. Segmentation 2. Propulsion.

Renin-Angiotensin-Aldosterone system 3 main stumuli

3 main stimuli: 1. A decrease in pressure in the afferent arteriole, sensed by stretch of granular cells. 2. Stimulation of sympathetic nerve fibers to the kidneys via b2-adrenergic receptors on granular cells. 3. A decrease in luminal NaCl concentration in the Macula Densa, typically as a result of lowered GFR. The basics of the Renin-angiotensin-Aldosterone (RAAS)in *regulation of water*: 1. Reduction in blood pressure at afferent arteriole of the kidney stimulates renin release from kidney. 2. Renin is released into blood, acts on angiotensinogen (secreted by liver) to form angiotensin I. 3. Vascular endothelium (primarily in lungs) secretes ACE which turns ANG I to ANG II. ANG II then: - Constricts resistance vessels (via AT1 receptors) which increases systemic vascular resistance and arterial pressure. - Acts on the adrenal cortex to release aldosterone which in turn acts on the kidneys to increase sodium and fluid retention. - Stimulates release of ADH from posterior pituitary which increases fluid retention in kidneys. - Stimulates thirst centers in hypothalamus. *Aldosterone* is a steroid hormone which combines with a cytoplasmic receptor to initiate the gene regulation of Na/K ATPase pumps and Na+ and K+ channels in the cells of the *distal-convoluted tubule to promote Na+, Cl-, and water reabsorption and K+ secretion.*

Lobes of the lung

3 on right, 2 on left. The lobes are comparmentalized, so damage to one lobe doesn't affect the whole lung, just that lobe.

GI Neural Control: Integration

3 reflexes in GI: 1. Long reflex with the CNS (vagovagal). 2. Short reflex within the ENS 3. Peptide-based reflex General reflexes in GI: named for where the input is coming from to where the response will be. A general rule of thumb is that when affect is ahead it is turned on, when affect is behind it is turned off.

Which of the following describes the state of the heart during atrial systole? 1. Both AV and semilunar valves are open 2. Both AV and semilunar valves are closed 3. The mitral valve is open and the aortic valve is closed 4. The tricuspid valve is closed and the pulmonic valve is open 5. The semilunar valves are open and the AV valves are closed.

3. The mitral valve is open and the aortic valve is closed.

What is the pH of the human body?

7.4 (ranges from 7.3-7.5)

Oxygen transport by blood

98% of O2 is carried to the tissue via bound hemoglobin in RBCs, 2% dissolved in blood *Oxyhemoglobin*- hemoglobin + oxygen *Deoxyhemoglobin* - hemoglobin - oxygen If hemoglobin was not present in order to get the same O2 consumption (250 mL/min) then we would need a cardiac output of 83.3 L/min which is impossible Without hemoglobin Oxygen would reach equilibrium with the partial pressure in alveolar air. This is inadequate for perfusion of our tissues unless cardiac output is insanely high. With hemoglobin, oxygen can still reach equilibrium with the partial pressure in alveolar air, AND on top of that hemoglobin can carry 4 oxygen molecules per unit. This dramatically increases the O2 content of blood without impacting the partial pressure (oxygen that binds to hemoglobin isn't counted as a partial pressure.)

Which component of bile is not primarily secreted by hepatocytes? a. Bicarbonate b. Bile salts c. Cholesterol d. Lecithin e. Bilirubin

A - Bicarbonate in the bile is secreted by the epithelial cells lining the bile ducts.

Which of the following changes occurs during defecation? a. Internal anal sphincter is relaxed b. External anal sphincter is contracted c. Rectal smooth muscle is relaxed d. Intra-abdominal pressure is lower than when at rest e. Segmentation contractions predominate

A - Both the internal and external anal sphincters must be relaxed to allow feces to be expelled from the body. Rectal smooth muscle contracts, and intra-abdominal pressure is elevated by expiring against a closed glottis (Valsalva maneuver). Segmentation contractions are predominant in the small intestine during digestion and absorption.

Which of the following does cholecystokinin (CCK) inhibit? a. Gastric emptying b. Pancreatic HCO3- secretion c. Pancreatic enzyme secretion d. Contraction of the gallbladder e. Relaxation of the sphincter of Oddi

A - Cholecystokinin (CCK) inhibits gastric emptying and therefore helps to slow the delivery of food from the stomach to the intestine during periods of high digestive activity. CCK stimulates both functions of the exocrine pancreas - HCO3- secretion and digestive enzyme secretion. It also stimulates the delivery of bile from the gallbladder to the small intestinal lumen by causing contraction of the gallbladder while relaxing the sphincter of Oddi.

It is inhibited by acid in the stomach and stimulates acid secretion from the stomach A. Gastrin B. CCK C. Secretin D. GIP

A - Gastrin is a major controller of acid secretion by the stomach. When the stomach becomes very acidic, gastrin release is inhibited, preventing continued acid production.

Which of the following is the site of secretion of gastrin? a. Gastric antrum b. Gastric fundus c. Duodenum d. Ileum e. Colon

A - Gastrin is secreted by the G cells of the gastric antrum. HCl and intrinsic factor are secreted by the fundus.

Which of the following is the primary absorptive process in the large intestine? a. Active transport of sodium from the lumen to the blood b. Absorption of water c. Active transport of potassium from the lumen to the blood d. Active absorption of bicarbonate into the blood e. Active secretion of chloride from the blood

A - The active transport of sodium in the large intestine is the driving force for the osmotic absorption of water.

What do parietal cells secrete when they are stimulated? a. HCl and intrinsic factor b. HCl and pepsinogen c. HCl and HCO3- d. HCO3- and intrinsic factor e. Mucus and Pepsinogen

A - The gastric parietal cells secrete HCl and intrinsic factor. The chief cells secrete pepsinogen.

Pulmonary function test and obstructive disease

A FEV1/FVC ratio of <70% implies an obstructive disease of the lungs After it has been concluded that the ratio is low, analysis of the FEV1 number (compared to the normal number for your population group) will tell you the severity of the obstruction: >80% FEV1 - minimal obstructive effect 65-80% FEV1 - mild obstructive effect 50-65% FEV1 - moderate obstructive effect <50% FEV1 - severe obstructive effect Restrictive disorders cannot be determined via a PFT.

Acid-Base Disturbances

A blood pH below 7.35 = acidemia. Blood pH above 7.45 indicates alkalemia. The range of pH valvues compatible with life is approximately 6.8-7.8. Four simple acid-base disturbances may lead to an abnormal pH: respiratory acidosis, respiratoray alkalosis, metabolic acidosis, and metabolic alkalosis. The word 'simple' indicates a single primary cause for the disturbance. Acidosis is an abnormal process that tends to produce acidemia while alkalosis is an abnormal process that tends to produce alkalemia. There is too much (acidosis) or too little (alkalosis) CO2, then a respiratory disturbance is present. If the problem is too much (alkalosis) or too little (acidosis) HCO3-, then a metabolic (or nonrespiratory) disturbance of acid-base balance is present. If the primary problem is a change in HCO3- concentration of Pco2, the pH can be brought closer to normal by changing the other member of the buffer pair in the same direction. NOTE: Acid-base data should always be interpreted in the context of other information about a patient. *Always look at the pH first*. To identify an acid-base disturbance from laboratory values, it is best to look first at the pH. A low blood pH indicates acidosis while a high blood pH indicates alkalosis. If acidosis is present, for example, it could be either respiratory or metabolic. *A low blood pH and elevated Pco2 point to respiratory acidosis while a low pH and low plasma HCO3- indicate metabolic acidosis*. Appropriate compensatory responses can be seen through changes in the other buffer-pair. However, inappropriate values suggest that more than one acid-base disturbance may be present. A mixed acid-base disturbance is not uncommon and involves some acidosis/alkalosis with respiratory and/or metabolic compensation.

Acid-Base buffering

A buffer is a combination of a weak acid and its conjugate base in solution. Its pH changes very little when a small amount of a strong acid or base is added to it. HCO3- + H+ -> H2CO3 H2O + CO2 -> H2CO3 HA + A- -> A- + HA Addition of a strong base (OH-) to the solution causes the weak acid (HA) to donate the hydrogen to create H2O and excess A-. A- is a weak base though and does not impact pH as much as the strong base. Addition of a strong acid (HCl) to the solution causes the weak base (A-) to bind with the excess H+ to form a weak acid (HA) HA is a weak acid though and does not impact pH as much as the strong acid.

Renin release via Macula Densa and granular/juxtaglomerular cells

A decrease in arterial pressure causes a drop in the glomerular hydrostatic pressure which decreases the glomerular filtration rate and also lowers the flow rate which both lowers the amount of Na+ that filters through (ends up in the tubules) and increases its reabsorption in the proximal tubule (because the flow rate is lower, it stays longer in the proximal tubular which allows time for more Na+ reabsorption). The decrease in NaCl and flow rate is sensed by the Macula Densa cells in the thick ascending limb. The Macula Densa cells in turn signal to the granular/juxtaglomerular cells which secrete Renin (-> angiotensin II -> Increase in efferent arteriolar resistance) and decrease afferent arteriolar resistance. Both pathways ultimately increasing glomerular hydrostatic pressure and operating under negative feedback inhibition. *Biggest effect is Macula densa -> granular/juxtaglomerular cells -> increase renin -> increase Angiotensin II -> increase Efferent arteriolar resistance (Efferent constriction) which increases glomerular hydrostatic pressure.* The constriction of the efferent arteriole via angiotensin II is more important than the dilatation of the afferent arteriole in increasing glomerular hydrostatic pressure -> increase GFR.

Describe oogenesis and its relationship to changes in the ovarian follicle (folliculogenesis). Explain the roles of FSH, LH, and estradiol in follicular maturation

A developing egg (oocyte) differentiates into a mature egg (ovum) through a series of steps called oogenesis.

Pneumothorax

A pneumothorax is a collapsed lung. A pneumothorax occurs when air leaks into the space between your lung and chest wall (pleural space). This air pushes on the outside of your lung and makes it collapse. In most cases, only a portion of the lung collapses thanks to the mediastinal membrane and lobes. The air into the pleural space causes the intrapleural pressure to equal atmospheric pressure and the lung collapses. To remedy, how to suck out the air from the pleural space to establish a negative pressure again. The transpulmonary pressure = 0 and the elastic recoil of the lungs causes it to collapse.

Sodium balance: Macula Densa

A portion of the distal convoluted tubule passes directly by the efferent and afferent arterioles in the area known as the Macula Densa. This allows for Granular cells to sense concentrations of Na+ and overall output pressures. The end result is for the cells to sense if the body needs to increase or decrease the rate of water conservation. The granular cells of the macula densa detect sodium levels in the distal convoluted tubule (96% reabsorbed) to determine if there is excess sodium (typically due to higher GFR and fluid volumes) or low sodium. Low sodium levels in the macula densa works to increase Na+ retention through the renin-angiotensin system. Renin release from the granular cells lining the afferent arteriole rely on NaCl sensing from the Macula Densa of the Distal Tubule. Low NaCl concentrations will lead to renin secretion and reabsorption of Na+. The granular cells adjacent to the macula densa can leak renin into the afferent arteriole. The release of renin eventually causes an increase in Angiotensin II (book says adenosine causes constriction of afferent) levels that will constrict the afferent arteriole (lowering GFR) and increase sodium reabsorption in the nephron.

Na+ transport in the Tubules

A. Early proximal convoluted tubule Na+ leaves via: - glucose/Na+ symport into cell - Na+/H+ antiport into cell - W/H2O via intercellular space into interstitial space. - Na+/HCO3- symport out of cell into interstitial space. - Na/K ATPase out of cell into interstitial space. B. Thick ascending limb Na+ leaves via: - *Na+/K+/2Cl- (NKCC) symport into cell* - Na+/H+ antiport into cell - Na/K ATPase out of cell into interstitial space. C. Distal convoluted tubule Na+ leaves via: - *Na+/Cl- symport into cell.* - Na/K ATPase out of cell into interstitial space. D. Principal cell of connecting tubule or cortical collecting tubule Na+ leaves via: - *Na+ channel into cell (K+ secreted into lumen via K+ channel)* - Na/K ATP out of cell into interstitial space.

A 50 Year old woman who has suffered from type 1 diabetes for almost 40 years comes to the doctor complaining of epigastric pain and the sensation that her meals are regurgitating into her mouth. Imperfect control of blood glucose levels over the long-standing course of her primary disease will most likely have resulted in injury to which of the following? A. Enteric neurons B. Gastric circular muscle C. Gastric longitudinal muscle D. Upper esophageal sphincter E. Parietal cells

A. Enteric neurons

A 50 year old woman has suffered from type 1 diabetes for almost 40 years comes to the doctor complaining of epigastric pain and the sensation that her meals are regurgitating into her mouth. Imperfect control of blood glucose levels over the long-standing course of her primary disease will most likely have resulted in injury to which of the following? A. Enteric neurons B. Gastric circular muscle C. Gastric longitudinal muscle D. Upper esophageal sphincter E. Parietal cells

A. Enteric neurons.

A 4 year old boy is brought to the pediatrician for an evaluation because of failure to thrive and frequent diarrhea characterized by pale, bulky, foul-smelling stools. Sweat chloride concentrations are measured and found to be elevated. Diminished secretion of which pancreatic product is most likely to be the primary cause of the patient's apparent fat malabsorption? A. Lipase B. Gastrin C. Cholecystokinin D. Bicarbonate

A. Lipase

Plasma glucose and insulin secretion following an oral (A) or intravenous (B) glucose challenge.

A. Oral glucose stimulates one high peak of insulin secretion. B. Intravenous glucose stimulates 2 peaks- *biphasic*. Peptide hormones are stored in vesicles and released ahead of time. Body senses high rate of glucose secretion and dumps all the presynthesized insulin that it already made (first peak). Second peak is the secondary insulin that is made.

In a study of the control of esophageal motility, a scientist instills a small amount of dilute hydrochloric acid into the upper third of the esophagus of a human volunteer using an endoscope. This treatment is most likely to produce which of the following responses? A. Peristalsis B. Retroperistalsis C. Esophageal spasm D. Relaxation of the upper esophageal sphincter E. Contraction of the lower esophageal sphincter

A. Peristalsis

Alveolar ventilation: Alteration

A. Taking a bunch of really shallow breaths. No alveolar ventilation because air doesn't make it past the dead space. (Decrease in tidal volume) B. Normal breathing C. Deep breaths, increases alveolar ventilation. (Increase in tidal volume)

A 40 year old man comes to his physician complaining of epigastric pain. An upper endoscopy reveals duodenal erosions and a test of gastric secretory function reveals markedly elevated levels of basal acid secretion that are increased only modestly by intravenous infusion of pentigastrin. What is the most likely diagnosis? A. Zollinger-Ellison syndrome B. H pylori infection C. Gastroesophageal reflux disease D. Dumping syndrome E. Hirschsprung disease

A. Zollinger-Ellison syndrome

Changes in water reabsorption

ADH acts on the initial and cortical collecting tubules, the outer medullary collecting duct, and the inner medullary collecting duct. No effect anywhere else including the distal convoluted tubule or distal tubule/connecting tubule. Water reabsorption can change the excreted load of water from 0.5% - 15% of the total amount that is filtered.

ADH: Collecting Duct and Water

ADH increases Aquaporin-2 expression and utilization on the luminal (tubular side) membrane of the collecting duct epithelium which allows water to be reabsorbed. This works to increase blood pressure and decrease plasma osmolality. Without ADH Urine: 70 mOsm and contains 15% of filtered H2O With ADH (possible) Urine: 1200 mOsm and contains 0.5% of filtered H2O Range: 0.5-15% of filtered water can be excreted

What increases the permeability of the medullary collecting duct to water by increasing the number of aquaporins on the luminal side allowing reabsorption of water?

ADH/AVP Anti-diuretic hormone/ Arginine Vasopressin

Conduction velocity slowest in heart in which cell type?

AV node

A 31 year old woman presents to the clinic with HR of 40 bpm. Examination of her EKG reveals broad QRS waves with no P waves. What could account for these findings?

AV node has taken over as the primary pacemaker.

Absolute vs relative refractory period

Absolute Refractory Period (1 ms): From threshold to beginning of hyperpolarization. No action potential can be generated in this period regardless of strength because either all Na+ channels are already open or the Na+ channels are inactivated. (Na+ channel inactivation and high permeability of K+) Relative refractory period (3 ms): Hyperpolarization phase. Action potential can be generated but requires a stimulus stronger than the original stimulus strength. Results from the partial inactivation of the Na+ channel and hyperpolarization (high K+ permeability)

What neurotransmitter is always released by preganglionic neurons?

Acetylcholine

The parasympathetic division uses only ________ as a neurotransmitter in the ganglionic neurons.

Acetylcholine (ach)

PNS regulation of pacemaker potential

Acetylcholine binds to Muscarinic cholinergic receptor (M2) -> G protein -> opens potassium channel causing hyperpolarization. Also inhibits T type calcium channels and Na+ funny channels. ====================== PNS - Vagus Nerve (Acetylcholine binds M2 muscarinic receptors) 1. Increases K+ permeability (hyperpolarization) 2. Decreases Na+ permeability (funny channel) (slows depolarization) 3. Decreases Ca2+ permeability (reduces steepness of depolarization. (Decrease in heart rate)

Acid excretion

Acid's are filtered in the kidney We typically excrete acids every day (metabolic byproducts) You can't load acids in the urine as actual acid, the pH would be damaging to the tissues. Instead acid has to be titrated or converted to be excreted. Renal net acid excretion (70 mEq/d) = Urinary titratable acid (mainly H2PO4- accounts for 1/3) (24 mEq/d) + urinary ammonia (NH4+ account for 2/3rds) (48 mEq/d) - urinary HCO3- (2 mEq/d).

Acid base disorder

Acidosis (acidemia): excessive addition of H+ to body fluids. - Conditions which could cause acidosis (based on physiology): - Not enough HCO3- - Too much acid (CO2) Alkalosis (alkalemia): excessive removal of H+ from body fluids - Conditions which could cause alkalosis (based on physiology): - Too much HCO3- - Not enough acid (CO2) Respiratory: changes in CO2 levels Metabolic: changes in HCO3- levels or gain/loss of nonvolatile acids or bases. *Key feature: the body will try compensate for these disorder*

Actin treadmilling

Actin is constantly treadmilling which means the (+) end is constantly added to (ATP actin) and the (-) end is constantly shrinking (ADP actin leaving)

Blood glucose levels

Acute hypoglycemia causes neurological problems, coma, and death. Therefore, fasting blood glucose levels must be maintained above 60 mg/100 ml. Chronic hyperglycemia (fasting blood glucose above 110 mg/100 ml) causes multiple problems, including increased oxidative stress within cells. Increased intracellular glucose also leads to increased intracellular lipids, and consequently lipotoxicity. Ultimately, these stresses induce insulin-resistance and beta cell dysfunction, which further compromise glucose tolerance and lead to Type 2 Diabetes Mellitus. High levels of blood glucose also create an osmotic burden on cells and the organism.

Pulmonary Edema

Additional fluid in the lungs typically ends up in the air space (alveolar air sacs) due to the limited interstitial space. This is called *Pulmonary Edema* The additional fluid creates a thick interface where it's difficult for O2 and CO2 to diffuse, decreased diffusion.

Structures of adrenal cortical steroids

Adrenal Cortex hormones: synthesized from cholesterol. Layers of the adrenal cortex have different enzymes Locations: mitochondria and endoplasmic reticulum *Zona glomerulosa*: Aldosterone (mineralcorticoid) *Zona fasciculata and Zona reticularis*: Cortisol (glucocorticoid), Corticosterone (glucocorticoid), and Dehydroeplandrosterone (androgen)

Ach released to adrenal medulla, what is the next?

Adrenal medulla secretes epi

Horner's syndrome

Affects sympathetic innervation to head and neck. Caused by pancoast tumors from smoking that impinges the sympathetic trunk decreasing sympathetic innervation. Symptoms: Flushing (vasodilation) Ptosis (right or left, depends which sympathetic trunk impinged) Anhidrosis (can't sweat) Miosis (constricted pupil) Only affects pattern 2 because tumor over upper chest.

ADH, plasma osmolality, and urine osmolality relationship

After plasma osmolality hits over 280, plasma ADH and Plasma Osmolality share a linear relationship. As the plasma osmolality goes up so does the plasma ADH. ADH also shares a linear relationship with urine osmolality.

What are the stimuli for GH release?

Age, time of day, low blood glucose and stress.

Dalton's Law

Air is predominantly Nitrogen (78%) and oxygen (21%). PO2 = .21 x 760 = 160 mmHg. This is at sea level PO2 = 0 x 760 = 0 mmHg PN2 = .78 x 760 = 592 mmHg It's just atmospheric pressure x the partial pressure of the gas.

What is the main contributor of capillary oncotic/osmotic pressure?

Albumin

Aldosterone

Aldosterone acts on *principal cells* to increase the number of *ENaC channels* that help move Na+ out of the tubular urine. It also increases the number of Na/K ATPases in the membrane that move K+ back into the principal cells while moving Na+ out into interstitial. It will also increase ATP production in the cells. Aldosterone binds with the mineralcorticoid receptor in the principle cells which then increases transcription of ENaC channels, ATP production, and Na/K ATPase activity. (Spironolactone blocks aldosterone binding to mineralcorticoid receptor. Amiloride blocks ENaC channels from allowing Na+ to cross into cell from tubular lumen.)

What increases Na+ permeability through the ENaC and upregulates the number of Na/K ATPases in the basolateral membrane?

Aldosterone.

Where can you find gray ramus

All 31 spinal cord levels, found in parasympathetic nervous system.

How much of glucose, bicarbonate, sodium, chloride, potassium, urea, and creatinine is filtered, reabsorbed, excreted, and %filtered load reabsorbed?

All glucose reabsorbed. All creatinine excreted. Half urea excreted. Potassium the most excreted of the solutes.

Adrenergic receptor A2

Alpha 2 (A2): Norepinephrine -> a2 g coupled protein receptor -> inhibits cAMP signaling causing: Decrease in GI motility Involved in GI motility.

What does the V/Q ratio tell us (Ventilation/Perfusion)

An area with perfusion but no ventilation (and thus a V/Q of zero) is termed "shunt". An area with ventilation but no perfusion (and thus a V/Q undefined though approaching infinity) is termed dead space A lower V/Q ratio (with respect to the expected value for a particular lung area in a defined position) impairs pulmonary gas exchange and is a cause of low arterial partial pressure of oxygen (paO2). *Seen in chronic bronchitis, asthma, hepatopulmonary syndrome, and acute pulmonary edema.* A high V/Q ratio decreases PACO2 and increases PAO2. Because of the increased dead space ventilation, the PaO2 is reduced and thus also the peripheral oxygen saturation is lower than normal, leading to tachypnea and dyspnea. This finding is typically associated with pulmonary embolism (Q = 0) (where blood circulation is impaired by an embolus). *A high V/Q can also be observed in emphysema* Normal V/Q ratio in healthy lung is .8

Inulin

An ideal substance to measure GFR is inulin, a fructose polymer with a molecular weight of about 5000 kD. Inulin is suitable for measuring GFR for the following reasons: - It is freely filtered by the glomeruli. - It is not reabsorbed or secreted by the kidney tubules - It is not synthesized, destroyed, or stored in the kidneys. - It is not toxic - Its concentration in the plasma and urine can be determined by simple analysis. The amount of Inulin filtered per unit time, the *filtered load*, is equal to the product of the plasma concentration of Inulin (Pin) multiplied by GFR. Because inulin is not reabsorbed, secreted, synthesized, destroyed, or stored by the kidney tubules, the filtered inulin load equals the rate of inulin excretion. *Therefore, inulin clearance = GFR*

Pericardium function

Anchors heart (heart would jump around as it beats without it) Prevent infection Reduces friction Normal pericardial fluid volume: up to 50 mL (not that much)

Anemia and carbon monoxide effect on hemoglobin curve

Anemia causes lower O2 content in the blood but doesn't effect the PO2 or oxygen saturation. Carbon monoxide bound to hemoglobin causes a similar effect.

Renin-Angiotensin system (RAS)

Angiotensinogen + renin -> Angiotensin I + angiotensin-converting enzyme (ACE) -> Angiotensin II -> Angiotensin II type 1 receptor (AT1R) Liver secretes angiotensinogen. Kidneys secrete renin in response to decrease in renal perfusion (juxtaglomerular apparatus) Lungs secrete ACE Angiotensin II stimulates: - sympathetic activity - tubular Na+, Cl- reabsorption and K+ excretion, H2O retension. - Adrenal cortex which stimulates aldosterone release which causes tubular Na+, Cl- reabsorption and K+ excretion, H2O retention. - Arteriolar vasoconstriction. Increase in blood pressure. - Posterior pituitary which stimulates ADH (Vasopresin) which causes H2O reabsorption in collecting duct and vasoconstriction. Overall causes water and salt retention. Effective circulating volume increases. Perfuson of the juxtaglomerular apparatus increases. Then negative feedback with increase in renal perfusion (juxtaglomerular apparatus) stops the kidneys from secreting renin.

Acid-base disorders and anion gap

Anion gap measurements are a method of determining the type of metabolic acidosis. *Anion gap*: used to diagnose specific form of metabolic acidosis *Equation* Anion gap = [Na+] - [Cl-] - [HCO3-] *Normal values = 8 to 14 mEq/L* Example of shift: Increased lactic acid. Lactic acid + HCO3- ⇌ Lactate- + H2O + CO2 Anion gap = [Na+] - [Cl-] - [HCO3-] : *increases* *Not all metabolic acidosis results in only decreased HCO3-.* - Diarrhea loss HCO3- but a rise in Cl- balances it out leading to a normal anion gap.

Hormones which regulate water levels in blood (3)

Antidiuretic Hormone (ADH) Renin and Angiotensin II

How does water cross the plasma membrane

Aquaporins are Transmembrane Proteins (channels) that facilitate the rapid transport of water through Plasma membranes.

What converts testosterone to estrogen?

Aromatase

Arteries vs veins

Arteries are thicker and have more smooth muscle, appear rounded. (vaso constriction/relaxation) Veins are thinner and have less smooth muscle, appear floppy. (veno constriction/relaxation)

ADH levels and decrease in blood volume

As ADH levels increase so does the osmolality of the urine as water is extracted in the collecting duct. (Increase in urine osmolality also increases ADH levels but to a lesser extent ~100x less potent). Although ADH is *primarily responding to osmolality changes*, changes in blood volume below 10% (0.5L) will also stimulate ADH release via *angiotensin II*. (> 10% loss in blood volume and you get a large sympathetic response which dramatically releases AVP/ADH.)

End result of this countercurrent multiplication

As fluid moves through the tubular sections it is modified by loss of solutes and water. Thanks to movement of salt out of the loop of Henle a gradient is established which encourages a net movement of salt and water out of the tubule and into the vasa recta. The end result is that of the filtered load of salt and water <0.5% of the Na+ and H2O remain by the time the urine leaves for the bladder.

CO2 equilibrium curve

As the partial pressure of CO2 increases so does it's content in the blood. Same as O2. The graph illustrates CO2 transported by hemoglobin. Haldane effect: Oxygenation of blood in the lungs displaces carbon dioxide from hemoglobin which increases the removal of carbon dioxide. Due to the haldane effect, when PO2 = 0 the curve moves up indicating more CO2 content in blood at the same PCO2.

Filtration pressures

As we filter more and more fluid, the remaining plasma becomes more concentrated therfore increasing the Glomerular colloid osmotic pressure. By modifying filtration and GFR we can change the plasma concentrations of solutes.

How would Cl- respond, with equilibrium Cl= -71 mV, to a Vm of -50 mV? -90 mV?

At -50 mV, Cl- (at -71) would go into the cell via chemical gradient. At -90 mV, Cl- would leave the cell via electrical gradient (against chemical gradient)

Where are cell junction found in the heart?

At intercalated discs. Mechanical junctions: -Fascia adherens - anchors actin (thin filaments) to plasma membrane -Desmosomes - bind intermediate filaments of adjoining cells Gap junctions- ion flow between cells, propagate APs to neighboring cells Allows for functional syncytium - coordinated contraction of the single nucleated muscle cells.

Airway resistance and volume

At low lung volumes there's high resistance. This is primarily due to airways being compressed at low volumes and some alveoli being completely closed off. At high volumes the resistance to air flow is much lower because the lungs are already inflated and less work needs to be accomplished to open the lungs to flow. Resistance = Delta P/ Volume

Where are graded potentials initiated?

At the axon terminals of the presynaptic cell

Where is ventilation highest in the lungs?

At the bases.

Chest pressures

Atmosphere is 0. Inside lungs is either +1 or -1. Paw - Airway Pressure Ppl - Pleural Pressure- negative at rest and becomes more negative during inhalation. PA - Alveolar Pressure- pressure in the alveolar sac Pta - Transairway Pressure: Pta = Paw - Ppl. PL - Transpulmonary Pressure (PL) The pressure that keeps the lungs inflated and prevents them from collapsing Both transairway pressure and transpulmonary pressure subtract pleural pressure.

Conduction velocity and cell type

Atrial Muscle (~.5 m/sec) Ventricular Muscle (~.5 m/sec) AV node (~0.05 m/sec) Bundle of His (~2 m/sec) Left and right bundle branches (~2 m/sec) Purkinje Fibers (~4 m/sec) (Purkinje Fibers- fastest fibers) (Atrial and ventricular muscle shows how fast SA node conduction velocity is)

Contribution of atrial contraction to ventricular filling

Atrial contraction is more important during exercise and aging.

What on EKG indicates atrial depolarization, ventricular depolarization, and ventricular repolarization

Atrial depolarization: P wave Ventricular depolarization: QRS complex Ventricular repolarization: T wave

Atrial diastole, atrial systole, ventricular diastole, ventricular systole and their location on EKG

Atrial systole: Half of P wave to peak of R in QRS complex. Ventricular diastole: From end of T wave to the next peak of R in QRS complex. Ventricular systole: From peak of R in QRS complex to end of T wave. Atrial diastole: From peak of R in QRS complex to half the next P wave.

Key elements of heart's pump function (3)

Automaticity that establishes duration of cycle Coordinated relaxation and contraction. Inlet and outlet one-way valves that open when upstream pressure exceeds downstream pressure.

Coronary circulation

Autoregulation to maintain coronary blood flow for needs of the heart. Affected by atherosclerosis/blockage.

Ovarian cycle

Average length: 28 days (range: 25-34 days) 2 phases: *Follicular phase* - Varies in length (11-20 days) *ovulation* *Luteal phase* - 14 days (constant)

Micelles increase the absorption of fat by which of the following mechanisms? a. Binding the lipase enzyme and holding it on the surface of the lipid emulsion droplet b. Keeping the insoluble products of fat digestion in small aggregates c. Promoting direct absorption across the intestinal epithelium d. Metabolizing triglyceride to monoglyceride e. Facilitating absorption into the lacteals

B - Because fat is insoluble in an aqueous environment, micelles keep fat droplets from re-aggregating and small enough to be absorbed.

It is stimulated by amino acids and fatty acids in the small intestine and stimulates pancreatic enzyme secretion. A. Gastrin B. CCK C. Secretin D. GIP

B - Cholecystokinin is the primary signal from the small intestine to the pancreas to increase digestive enzyme release into the small intestine.

Secretion of which of the following substances is inhibited by low pH? a. Secretin b. Gastrin c. Cholecystokinin (CCK) d. Vasoactive Intestinal Peptide (VIP) e. Gastric Inhibitory Peptide (GIP)

B - Gastrin's principal physiologic action is to increase H+ secretion. H+ secretion decreases the pH of the stomach contents. The decreased pH, in turn, inhibits further secretion of gastrin - a classic example of negative feedback.

Which of the following actions would result from an overproduction of PTH? A. Increased calcium excretion in kidney B. Increased phosphate excretion in kidney C. Decreased calcium resorption from bone D. Decreased phosphate reabsorption in intestine E. Decreased calcium reabsorption in intestine

B. Increased phosphate excretion in kidney

Sympathetic vs Parasympathetic stimulation and cardiovascular system flow chart

BP (MAP) = CO x TPR

Dopamine pathway and the suckling reflex

Baby suckling on breast triggers stimulus to hypothalamus to inhibit GnRH and dopamine secretion which allows the breasts to lactate milk. Lactational amenorrhea: temporary postnatal infertility that occurs when a woman is amenorrheic (not menstruating) and fully breastfeeding.

Open systems in the body

Because HCO3- can be regulated by the kidney or the lungs by conversion between CO2 and H2O we can also respirate out any excess acid. There are 2 points of exit for acids in the body (excrete or respirate). CO2 is a volatile acid and very strong, HCO3- is not.

Why are single-unit smooth muscles termed "single-unit"

Because they contract as a single unit because they have synchronized electrical mechanical activity thanks to gap junctions.

Why is Cl- concentrations modesty above equilibrium in most cells?

Because uptake via Cl-/HCO3 exchanger and Na/K/Cl- transporter balances passive Cl- efflux through channels.

Pulmonary resistance and cardiac output.

Because vessels higher in the lung have such low perfusion pressure the capillaries are typically closed. An increase in cardiac output which would typically cause an increase in pressure and/or resistance actually causes a reduction in resistance by opening more capillaries (Increase in cardiac output causes decrease in pulmonary resistance due to opening of more capillaries and pulmonary arteries being more stretchy than muscular arteries)

HCO3- An Open System

Because we can modify CO2, H+, and HCO3- levels through respiratory and renal mechanisms this allows us to have an open system where inputs and outputs are all controlled. We can readily add more HCO3- to the system in the kidneys which allows us to rapidly change our pH without needing input of extra molecules.

Acid base scale

Below 6.8 and above 8.0 plasma pH you die.

Adrenergic receptor B1

Beta 2 (B1): Norepinephrine -> B2 g coupled protein receptor -> Adenyl cyclase -> cAMP which causes: Contraction of cardiac muscle (increase rate, force, automaticity) Found in heart

Adrenergic receptor B2

Beta 2 (B2): Norepinephrine -> B2 g coupled protein receptor -> Adenyl cyclase -> cAMP which causes: Relaxation of of lungs Found in lungs Same mechanism as B1

Structure

Blood flow is designed to send plasma to the nephron to be filtered. Each medullary area works as a unit of nephrons Filtered blood exits through the ureter

Flow in the nephron

Blood in the vasa recta travels down and around the loop at the bottom in the same fashion as fluid in the loop of Henle. These flows that mimic each other create an environment where reabsorbed solutes and water in one location are quickly whisked away in the blood. This change in dynamic and creates different zones within the nephron.

What have no parasympathetic innervation?

Blood vessels. Controlled by A1 adenergic. Strong sympathetic tone = vasocontriction. Weaker sympathetic tone = vasodilation.

Central blood volume

Blood volume = intrathoracic + Extrathoracic. Central (Intrathoracic) blood volume = blood in the right atrium/ventricle, pulmonary circulation, left atrium, superior vena cava and intrathoracic portions of the inferior vena cava. (Basically all the blood in the heart). Central Blood Volume can be increased or decreased by shifts in blood to and from the extrathoracic blood volume (veins in extremities and abdominal cavity) Veins are primary factors of central blood volume: Venoconstrict = decrease in venous compliance = Increase central venous pressure -> blood goes to the heart. Venodilation = Increase in venous compliance = Decrease in central venous pressure -> blood goes to veins in extremities and abdominal cavity.

Renal blood flow vs Renal plasma flow

Blood: - Red blood cells - Proteins - Ions - Macromolecules Blood volumes: - 60% plasma - 40% hematocrit Renal plasma flow (RPF) = Renal blood flow (RBF) x (1 - hematocrit (Hct)) Renal plasma flow and renal blood flow refer to similar things. Blood includes red blood cless and other large macromolecules that make up about 40% of the volume. Plasma excludes that fraction. When spun down the fraction of blood that contains red blood cells is referred to as hematocrit (hct) moves to the bottom of the tube.

Temporal summation of muscle twitches

Body uses tetanus to hold position. Pathological situations where tetanus happens due to problems include cramps, spasms. Contraction lasts longer than action potential with temporal summation of muscle twitches. In a single muscle fiber, the force developed may be increased by summing multiple twitches in time.

Transport mechanisms

Both active and passive mechanisms help us to establish ion flow and reabsorption of essential ions. For a substance to be reabsorbed, it must first be transported (1) across the tubular epithelial membranes into the renal interstitial fluid and then (2) through the peritubular capillary membrane back into the blood. Thus, reabsorption of water and solutes includes a series of transport steps. Reabsorption across the tubular epithelium into the interstitial fluid includes active or passive transport mechanisms. For instance, water and solutes can be transported through the cell membranes themselves (transcellular route) or through the spaces between cell junctions (paracellular route). Then, after absorption across the tubular epithelial cells into the interstitial fluid, water and solutes are transported through the peritubular capillary walls into the blood by ultrafiltration (bulk flow) that is mediated by the hydrostatic and colloid osmotic pressures. The peritubular capillaries behave like the venous ends of most other capillaries because there is a net reabsorptive force that moves the fluid and solutes from the interstitium into the blood.

Active vs reactive hyperemia

Both caused by increase in metabolites. Active hyperemia = Increased blood flow to specific tissue due to increase of metabolism in that tissue. Reactive hyperemia = Increased blood flow to tissue following ischemia. The interruption in blood flow to the tissue causes buildup of metabolic waste leasing to vasodilation/increased blood flow to that area. Is transient- doesn't last very long.

Insulin Synthesis continued

Both insulin and C-peptide are present in secretory vesicles. When signaled, exocytosis of insulin and C-peptide (in equimolar amounts) 50-60% of the insulin produced by the pancreas is extracted by the liver before reaching systemic circulation. C-peptide is not extracted by the liver. Thus, it can be used to measure endogenous B cell function in an insulin-treated patient.

Respiratory acidosis can be caused by?

Brain damage, pneumonia, emphysema.

Blood flow and lung volumes

Breathing in = Increase in pulmonary vascular resistance. This is because the alveolus fills up with air and expands squeezing against the alveolar vessels causing an increase in capillary resistance (pulmonary vascular resistance) Breathing out = Increase in pulmonary vascular resistance. This is because the alveolus shrinks and the alveolar vessels below it expands causing the extra-alveolar vessels on either side to shrink which increases capillary resistance (pulmonary vascular resistance)

Capillary fluid exchange

Bulk flow- filtration (reabsorption) of fluids and their solutes. Capillary to tissue is called filtration. Tissue to capillary called re-absorption. Filtration = Kf x NFP *NFP = (Pc + πi) - (Pi + πc)* Kf = capillary permeability/surface area NFP = net filtration pressure Starling forces = Hydrostatic pressure and Osmotic/oncotic pressure ------------------------------ *Hydrostatic pressure (P)* = *Pushing* pressure (exerted by water in the blood) due to volume from fluid. *Pc* = hydrostatic pressure in capillary = push fluid out of capillary and into tissue *Pi* = hydrostatic pressure in interstitium = push fluid out of tissues and into capillary. ------------------------------ *Oncotic/Osmotic pressure*= *Pulling* pressure, due to solutes. *πc* = Oncotic pressure in capillary (plasma proteins- albumin) = pull fluid into capillary and out of tissues. *πi* = Oncotic pressure in interstitium = pull fluid into tissue and out of capillary. ------------------------------ High hydrostatic capillary pressure and high oncotic interstitial pressure favors filtration (capillaries -> tissues) High hydrostatic interstitial pressure and high oncotic capillary pressure favors reabsorption (tissues -> capillaries)

How does an increase in preload cause a higher stroke volume?

By stretching the cardiomyocytes, you increase the amount of cross bridges and increase their calcium sensitivity and therefore are able to generate more tension and eject more blood from the heart.

Which of the following substances inhibits gastric emptying? a. Secretin b. Gastrin c. Cholecystokinin (CCK) d. Vasoactive Intestinal Peptide (VIP) e. Gastric Inhibitory Peptide (GIP)

C - Cholecystokinin (CCK) is the most important hormone for digestion and absorption of dietary fat. In addition to causing contraction of the gallbladder, it inhibits gastric emptying. As a result, chime moves more slowly from the stomach to the small intestine, thus allowing more time for fat digestion and absorption.

Which of the following inhibits gastric HCL secretion during a meal? a. Stimulation of the parasympathetic nerves to the enteric nervous system b. The sight and smell of food c. Distension of the duodenum d. Presence of peptides in the stomach e. Distension of the stomach

C - Distention of the duodenum signals the stomach that the meal has moved on and continued acid secretion in the stomach is not necessary until the next meal.

A patient with severe Crohn's disease has been unresponsive to drug therapy and undergoes ileal resection. After the surgery, he will have steatorrhea due to which of the following mechanisms? a. The liver bile acid pool increases b. Chylomicrons do not form in the intestinal lumen c. Micelles do not form in the intestinal lumen d. Dietary triglycerides cannot be digested e. The pancreas does not secrete lipase

C - Ileal resection removes the portion of the small intestine that normally transports bile acids from the lumen of the gut and recirculates them to the liver. Because this process maintains the bile acid pool, new synthesis of bile acids is needed only to replace those bile acids that are lost in the feces. With ileal resection, most of the bile acids secreted are excreted in the feces, and the liver pool is significantly diminished. Bile acids are needed for micelle formation in the intestinal lumen to solubilize the products of lipid digestion so that they can be absorbed. Chylomicrons are formed within the intestinal epithelial cells and are transported to lymph vessels.

It is stimulated by the presence of acid in the small intestine and stimulates bicarbonate release from the pancreas and bile salts. A. Gastrin B. CCK C. Secretin D. GIP

C - When the stomach contents, which are very acidic, move into the small intestine, it stimulates the release of secretin, which circulates to the pancreas and stimulates the release of bicarbonate into the small intestine. This neutralizes the acid and protects the small intestine.

a 50 year old male is seen in nephrology with frequent urination and decreased urine osmolarity. He has hypernatremia and high plasma osmolarity. Loss of which of the following hormones would cause this condition? A. Oxytocin B. GH C. ADH D. ACTH E. TRH

C. ADH

Spatial summation of muscle fiber

CNS can control force by increasing/decreasing the number of muscle fibers stimulated.

What is the driving force for respiratory rate/ventilation?

CO2 Arterial PO2 changes do not substantially elicit responses in the carotid body until PO2 drops below 60 mm Hg. While changes in pH do lead to changes in ventilation rates, PCO2 is a stronger driver and has a much sharper curve. Extreme acidification (below 7.2) will lead to increasing responses in ventilation.

CRH (corticotropin-releasing hormone) and pituitary effect

CRH -> *ACTH (peptide)* -> Cortisol (glucocorticoid). CRH binds to the G-protein of the corticotroph cell in the anterior pituitary and activates *Gas* -> cAMP -> PKA -> protein transcription -> Secretory granules release ACTH and B-LPH (lipotropin, released with ACTH like how C-peptide is released with insulin).

CRH release and target cell effect.

CRH -> ACTH (peptide) -> *Cortisol (glucocorticoid)* ACTH -> G protein -> Gas -> cAMP -> PKA -> Nucleus transcription of steroidogenic enzymes -> binds to cholesterol -> Cortisol. ACTH bind to the receptor in the Zona Fasciculata of the adrenal cortex. *Cortisol Functions* - Controls blood sugar levels (stimulates gluconeogenesis) - Regulates metabolism

What are the 5 hypothalamus releasing hormones and their target cell, action on target cell, and the tropic hormone target

CRH has little control over the secretion of aldosterone.

Which pumps keep Intracellular Ca2+ four orders of magnitude lower than extracellular Ca2+?

Ca-H pump and Na-Ca exchanger (antiport, 2nd active transport) Ca-H pump (Primary active transport)

Calcium actions in sarcoplasm

Calcium binds to TnC which exposes the myosin binding sites

Average daily turnover of calcium

Calcium is mainly stored in the bones. Over 85% stored in bones. Absorbed by the small intestine - Active transport is regulated by calcitriol Kidneys only filter Ca2+ bound to small anions or free. Calcium and phosphate is handled by the GI, kidneys, and bone.

Electrolytes, minerals, and water digestion and absorption

Calcium: absorption regulated by vitamin D and parathyroid hormone Iron: levels regulated by amount of iron-binding proteins in intestinal cells Water: follows solutes osmotically Sodium: co-transported with amino acids, glucose/galactose; Exchanged for H+; dependent on Na/K-ATPase, Aldosterone stimulated Bicarbonate/Chloride: Absorption of Na+ in exchange for H+; Acid-Base regulation with HCO3-

Transport between extracellular and intracellular compartments takes place via what?

Capillary exchange. Plasma -> interstitial fluid -> intracellular fluid via capillary exchange. Mechanisms that control capillary fluid exchange are dynamic.

Transport between extracellular and intracellular compartments takes place via what?

Capillary exchange. Plasma -> interstitial via capillary exchange -> intracellular

What separates plasma and interstitial fluid

Capillary membrane.

AP/Contraction in Cardiac vs Skeletal Muscle

Cardiac muscle with a longer refractory period,refractory period lasts almost as long as the entire muscle twitch (prevents tetany of muscle). Cardiac muscle action potential graph has a plateau in it and is longer due to the influx/efflux of different ions.

Regulation of cardiac output

Cardiac output is increased by Heart Rate and Stroke Volume. Stroke volume: Increased by increase in contractility (Inotropy) and preload. Decreased by increase in afterload. (Afterload is the pressure against which the heart must work to eject blood during systole.)

Rate of flux of solutes depends on the mechanism of transport: compare carrier-mediated transport (facilitated diffusion) to simple diffusion.

Carrier-mediated transport is rapid at first but saturates due to the limited number of transporters in the membrane. Straight line going up at steep angle and then plateauing at Vmax. Simple diffusion: Only limited by the concentration gradient and the amount of time. Slope is smaller but line is linear. More solute= higher rate of entry, not as fast as carrier-mediated transport.

Hormonal influences of cardiovascular system

Catecholamine (adrenal medula, main hormone epinephrine): sympathetic pathway: cause the sympathetic stimulation flow chart in heart, arterioles, and veins increasing blood pressure.

Hypoadrenalism (adrenal insufficiency)

Causes hyponatremia, hypotension, and hyperkalemia with primary (adrenal) hypoadrenalism. Secondary hypoadrenalism has normal aldosterone levels, sodium levels, and potassium levels because the pituitary and/or hypothalamus doesn't really stimulate aldosterone, renin-angiotensin pathway does. Will have decreased ACTH levels because the problem is rooted in the pituitary and/or hypothalamus so the signal isn't being sent to stimulate ACTH release.

Central venous pressure

Central venous pressure (CVP) = pressure in thoracic vena cava near the right atrium (reflects right atrial pressure) - Major determinant of filling pressure (preload) of right ventricle. This regulates stroke volume through Frank-Starling mechanism. ΔCVP = ΔV/Cv ΔV = change in volume of blood within thoracic veins Cv = compliance of the thoracic veins (C=ΔV/ΔP) Increase in Central Venous Pressure = Increase in EDV = Increase in SV.

Na+ balance

Changes in Na+ balance are associated with circulating volume changes. Thus, altering Na+ is typically associated with changes to systemic blood pressure. The result is that the mechanisms we associate with blood pressure control become active. Components of the renin-angiotensin-aldosterone system. This system is activated by a decrease in the effective arterial blood volume (eg. following hemorrhage) and results in compensatory changes that help restore arterial blood pressure and blood volume to normal.

Graded potential

Changes in membrane potential that are confined to a relatively small region of the plasma membrane (of neurons) and occur in varying degrees of strength and magnitude. - Depolarizing or hyperpolarizing. - Decrease in magnitude with increasing distance from the site of origin. - Only function as signals over very short distances. Temporal summations (time) and spatial summations (space) allow potential past threshold to depolorize.

ADH levels in response to mOsm/kg H2O

Changes in plasma osmolality alter circulating ADH levels. Below 280 mOsm/kg, circulating levels of ADH are near 0 and water is freely excreted in the urine with little reabsorption. At physiological vales of plasma osmolality there will be circulating levels of ADH. 285 mOsm/kg H2O is normal plasma osmolality. 290 mOsm/kg H2O is when you get thirst. ADH/AVP levels are very responsive to plasma osmolality. A drop in osmolality will reduce AVP levels and stop water reabsorption in the collecting duct (so more water is excreted). An increase in plasma osmolality will increase AVP release and work to retain fluid via reabsoption in the collecting duct. ADH/AVP are also responsive to changes in urine osmolality but somewhat less responsive. *Changes of ~500 mOsm/kg in urine osmolality will cause a change of 2 pg/mL in Plasma AVP/ADH. Changes of 5 mOsm/kg in plasma osmolality with cause the same increase (2 pg/mL) in ADH/AVL.* This means that plasma osmolality is about 100x more effective in eliciting ADH levels than urine osmolality. Blood volume can also cause large shifts in AVP/ADH levels. *Loses of 5-10% (up to 0.5L)* can alter AVP/ADH release through *angiotensin II activity.* Large volume losses of up to *1 L of blood* (.05-1L) cause a dramatic release in AVP in an effort to maintain blood pressures. The stimulation is typically accompanied by *large sympathetic changes*.

Facilitated diffusion: carrier mediated

Characteristics: -Allow transport of polar molecules at rates much higher than that expected from the partition coefficient of these molecules. -They eventually reach saturation at high substrate concentration -Have structural specificity, meaning each carrier system recognizes and binds specific chemical structures. -They show competitive inhibition by molecules with similar chemical structure. Examples: Glut1- Glucose transporter AE1- Anion Exchange protein 1 which allows exchange of HCO3- and Cl- No ATP used

Chemical buffers

Chemical buffers in the ECF and ICF and in bone are the first line of defense of blood pH. Chemical buffering minimizes the change in pH but does not remove acid or base from the body. In the ECF, the main chemical buffer pair is HCO3-/CO2. Plasma proteins and inorganic phosphate are also ECF buffers. Cells have large buffer stores, particularly proteins and organic phosphate compounds. HCO3- is present in cells, although at lower concentrations than in the ECF. The best buffers have pKa's close to the pH that is desired. A pH buffer is defined as an agent that minimizes the change in pH produced when an acid or base is added. Note that a buffer does not prevent a pH change but simply minimizes the change. A chemical pH buffer is a mixture of a weak acid and its conjugate base (or a weak base and its conjugate acid). Although bicarbonate has a pKa at 6.1 which isn't as good as the 6.8 pKa of phosphate when compared to the physiological pKa of 7.4, the bicarbonate is way more controllable whereas phophates are just attained from the diet.

Hormone Transport: Receptor internalization

Clathrin-coated pits can endocytose membrane receptors into cytoplasm to be recycle when down-regulating. Opposite effect when up-regulating.

Clearance

Clearance is the measure of the kidney's excretion ability. Clearance = (urine concentration (mg/ml) * urine flow rate (ml/min))/ plasma concentration (mg/ml) Cx = Ux(V)/Px Excretion rate (mg/min): Urine concentration x urine flow rate. The renal clearance of a substance can be defined as the volume of plasma from which that substance is completely removed (cleared) per unit time. Renal clearance is one indication of kidney function.

Secondary active transport in the Tubules

Co-transporters symport: -SGLT: moves both glucose and Na+ from tubular lumen into tubular cells. -GLUT: moves glucose from tubular cell to interstitial fluid. - Amino acid/Na+ co-transporter: moves both Na+ and amino acids from tubular lumen into tubular cells. Co-transporters antiport: - NHE: moves Na+ into tubular cells from the tubular lumen, moves H+ out into the tubular lumen in the process. ----------- Na+/K+ pump keeps Na+ concentrations in check. (pumps Na+ out of the tubular cells into interstitial space/blood.

Which cell is primary involved in K+ secretion? Which cell is primary involved in acid-base homeostasis?

Collecting duct principle cell promotes K+ secretion via Aldosterone. Alpha-intercalated cells promote H+ secretion (also promotes K+ conservation when dietary intake of K+ is deficient). -Beta-intercalated cells promote HCO3- secretion.

Gravity and lung volume on regional compliance

Compliance is decreased at the apex and increased at the base of the lung when breathing from Functional Residual Capacity. Because the airways at the apex are already stretched open and filled with air like a fully inflated balloon while the base is like a partially inflated balloon. At low lung volumes, compliance at the apex is higher than compliance at the base- which is like a completely empty balloon

Lung compliance

Compliance is the ability of the lung to distend when addition volume is added. It is the siffness/stretchiness of an object. The lungs should stretch but not all lungs stretch the same. By looking at differences in compliance we are really analyzing the quality and content of our Elastin fibers. Compliance = C = Delta V/ Delta P. Change in volume/change in pressure. Compliance is a manner of evaluating the degree of volume change in response to pressure change. As a pressure pushes outward on an object, the compliance of that object determines how much stretch can take place to increase the volume. More compliant = can change volume more.

Distal tubule and collecting duct

Composed of 2 distinct cell types, the principal cells and intercalated cells, found in the late distal tubule (connecting/collecting tubule, water permeable) and the collecting duct. The principal cells reabsorb sodium and water from the lumen and secrete potassium ions into the lumen. The Type A intercalated cells reabsorb potassium ions and secrete hydrogen ions into the tubular lumen.

Golgi apparatus characteristics

Composed of four or more stacked layers of thin, flat, enclosed vesicles lying near one side of the nucleus. This apparatus is prominent in secretory cells, where it is located on the side of the cell from which the secretory substances are extruded. Processes substances transported from the ER to form: Lysosomes, secretory vesicles, and other cytoplasmic components.

Chord Conductance Equation

Conductance = electrical concept that relates the rate of charge movement (current = I) to the driving force on the charges. I (ion)= g(ion)(Vm-Eion) Example question on exam: We have conductance of sodium .1, chloride 0, potassium .1, and calcium at 10. If we took away potassium conductance what happens to resting membrane potential? It would move towards the sodium equilibrium potential and therefore would become more positive.

Splanchnic circulation

Connects GI tract, spleen, pancreas, and liver. - Portal vein connects GI tract to liver. - Quick and easy storage and processing of absorbed nutrients.

Extrinsic/Autonomic Nervous System

Contraction and relaxation within the GI muscular system involves excitatory junction potentials (EJPs) and inhibitory junction potentials (IJPs). Similar to EPSPs and IPSPs of the CNS, these graded potentials help influence the slow electrical waves within the GI system to trigger action potentials, or prevent the initiation of contraction. Inhibitory motor neurons, however, control the majority of movement within the GI system through prevention of contraction. Examples include: peristaltic movements, segmentations, control of sphincters.

Parts of the nephron found in cortex, outer medulla, and inner medulla

Cortex: - Juxtaglomerular apparatus - glomerulus - Bowman capsule - Proximal convoluted tubule - Proximal straight tubule - Connecting tubule Outer medulla: - Descending thin limb and ascending thin limb (superficial cortical nephron) - Ascending thick limb - Outer medullary collecting duct Inner medulla: - Ascending thin limb and Descending thin limb (long-looped juxtamedullary nephron) - Inner medullary collecting duct - Papillary duct

What do intestinal crypts do?

Crypts Small intestine: - Isotonic alkaline fluid - Paneth cells: immune response Large intestine: - HCO3- and K+ secretions, mucus-secretions. Purpose: - Protection from irritation, adherence for fecal matter, protects from bacterial acid production Stimuli for release: - Tactile, pressure (distension), hormonal, toxins, ENS reflex.

Which of the following is transported in intestinal epithelial cells by a Na+-dependent cotransport process? a. Fatty acids b. Triglycerides c. Fructose d. Alanine e. Oligopeptides

D - Fructose is the only monosacchride that is not absorbed by Na+-dependent cotransport; it is transported by facilitated diffusion. Amino acids are absorbed by Na+-dependent cotransport, but oligopeptides (larger peptide units) are not. Triglycerides are not absorbed without further digestion. The products of lipid digestion, such as fatty acids, are absorbed by simple diffusion.

It is stimulated by glucose and fat in the small intestine and increases insulin and amplifies the insulin response to glucose. A. Gastrin B. CCK C. Secretin D. GIP

D - GIP release is a feed-forward mechanism to signal the islet cells in the pancreas that the products of food digestion are on their way to the blood. This results in an augmented insulin release to meal.

Which of the following is true of peristalsis of the small intestine? a. It mixes the food bolus b. It is coordinated by the central nervous system (CNS) c. It involves contraction of smooth muscle behind and in front of the food bolus d. It involves contraction of smooth muscle behind the food bolus and relaxation of the smooth muscle in front of the bolus e. It involves relaxation of the smooth muscle simultaneously throughout the small intestine

D - Peristalsis is contractile activity that is coordinated by the enteric nervous system [not the central nervous system] and propels the intestinal contents forward. Normally, it takes place after sufficient mixing, digestion, and absorption have occurred. To propel the food bolus forward, the smooth muscle must simultaneously contract behind it and relax in front of it.

Slow waves in small intestinal smooth muscles are the example of which of the following? a. Action potentials b. Phasic contractions c. Tonic contractions d. Oscillating resting membrane potentials e. Oscillating release of cholecystokinin (CCK)

D - Slow waves are oscillating resting membrane potentials of the gastrointestinal (GI) muscle. The slow waves bring the membrane potential toward or to threshold, but are not themselves action potentials. If the membrane potential is brought to threshold by a slow wave, then a action potentials occur, followed by contraction.

Which of the following is true about pepsin? a. Most pepsin is released directly from chief cells b. Pepsin is most active at high pH c. Pepsin is essential for protein digestion d. Pepsin accelerates protein digestion e. Pepsin accelerates fat digestion

D - The enzyme pepsin is produced from pepsinogen in the presence of acid. This zymogen accelerates protein digestion.

Which of the following explains the reason why cholecystokinin (CCK) has some gastrin-like properties? a. Both CCK and gastrin are released from G-cells in the stomach b. Both CCK and gastrin are released from I-cells of the duodenum c. Both CCK and gastrin are members of the secretin-homologous family d. Both CCK and gastrin have five identical C-terminal amino acids e. Both CCK and gastrin have 90% homology of their amino acids

D - The two hormones have five identical amino acids at the C terminus. Biologic activity of cholecystokinin (CCK) is associated with the seven C-terminal amino acids, and biologic activity of gastrin is associated with the four C-terminal amino acids. Because this CCK heptapeptide contains the five common amino acids, it is logical that CCK should have some gastrin-like properties. G cells secrete gastrin. I cells secrete CCK. The secretin family includes glucagon.

Which of the following substances is released from neurons in the GI tract and produces smooth muscle relaxation? a. Secretin b. Gastrin c. Cholecystokinin (CCK) d. Vasoactive intestinal peptide (VIP) e. Gastric inhibiting peptide (GIP)

D - Vasoactive intestinal peptide (VIP) is a gastrointestinal (GI) neurocrine that causes relaxation of the GI smooth muscle. For example, VIP mediates the relaxation response of the lower esophageal sphincter when a bolus of food approaches it, allowing passage of the bolus into the stomach.

Factors that increase H+ secretion

Decrease in ICF pH/Increased ECF H+ concentration. Increased ECF Pco2 Increased Carbonic Anhydrase Activity Increased Na+ Reabsorption and Decreased plasma K+ concentration. Increased Aldosterone.

Metabolic influences (local influence)

Decrease in O2 Increase in CO2 Increase in adenosine Increase in H+ Increase in K+ All these factors are byproducts of tissue metabolism and act as vasodilators on blood vessels allowing them to leave tissues cells and go into the blood. Release of these factors is proportional to tissue metabolism. What causes metabolic dilation to stop or decrease? The metabolites can be washed away by the increase in blood flow. Metabolite production may decrease.

Effects of decreased afterload on SV

Decrease in afterload causes increase in SV and decrease in ESV. On subsequent cycle there is a decrease in EDV and therefore preload and subsequently stroke volume.

Ventilation and CO2

Decrease in alveolar ventilation = hypoventilating. (higher arterial CO2 concentration) Increase in alveolar ventilation = hyperventilating (lower arterial CO2 concentration)

Effects of decreased inotropy on SV

Decreased inotropy causes decrease in stroke volume and increase in ESV. Causes increased EDV on subsequent cycle.

PTH increases PO4- excretion by kidney how?

Decreases phosphate reabsorption by decreasing Na+/PO4- cotransporter activity.

Iv infusion of Hypotonic solution does what to Darrow-Yannet diagram

Decreases the osmolarity of both intracellular and extracellular fluid. Increases the volume of both intracellular and extracellular fluid. Osmolarity is decreased because you just added a solution with low amount of solutes which is basically diluting the intra and extra fluids. Volumes increase because the extracellular volume increases due to you adding fluid to it via IV, since this is a hypotonic solution- it will try to get into cells and therefore increase the volume intracellularly.

Parasympathetics only distribute to what?

Deep structures (except for the genitalia) Do not distribute to skin or blood vessels.

GI Motility: Esophagus- Deglutition

Deglutition - swallowing of food. 3 phases 1. *Voluntary phase* 2. *Pharyngeal phase* Control: - Trigeminal nerve - Glossopharyngeal nerve - Brainstem areas- swallowing center - Descending to cranial nerves 3. *Esophageal Phase* Control: - Swallowing reflex - Enteric reflexes - Two-part event: -- Primary Peristalsis: continuation of the initial wave. -- Secondary Peristalsis: distention of the esophagus stimulates another peristaltic contraction.

Guillain Barre Syndrome

Demyelination of peripheral nervous system (Schwann cells) progressive ascending paralysis (from lower to upper limbs), although it may come on rapidly and affect all four limbs simultaneously. The deep tendon reflexes are lost. The disease may also affect the face, trunk and diaphragm. The cause is also unknown; however it is often associated with viral infection 2 to 3 weeks before the infection and it involves autoimmune system

Multiple sclerosis

Demyelination of the central nervous system (Oligodendrocytes) disturbances in vision and speech, lack of coordination and muscle weakness. Although the cause of multiple sclerosis is unknown, it is associated with interplay of multiple genetic and environmental factors involving an immune mechanism.

What does the ionic composition of pancreatic secretions depend on?

Depends on flow rate. Bicarbonate secretion (HCO3-) increases with increasing flow rate and Cl- secretion decreases with increasing flow rate.

Parathyroid Hormone (PTH)

Derived from parathyroid glands Chief cells (of parathyroid glands) - synthesize and secrete PTH *Overall effect is to increase plasma Ca2+*

The descending and ascending loop of Henle

Descending loop: -Highly water permeable. -Lots of aquaporins. Ascending loop: -Impermeable to water because *no aquaporins* - Reabsorption of solutes.

Fick's law of diffusion

Describes the flux of a solute across a distance (delta X) Linear relationship where diffusion coefficient remains constant and driving force is the difference in solute concentration. Rate of diffusion, J (flux), is dependent on: -Concentration difference -Size of the molecule -Area of the diffusion surface -Distance of diffusion -Water solubility J= PA(C1-C2) or J= DA(C1-C2)/Delta X J- Flow of solute from region 1 to region 2 P- Permeability coefficient D- Diffusion coefficient A- Cross sectional area C1- Concentration in Region 1 C2- Concentration in Region 2

Renal blood flow

Despite changes in mean arterial blood pressure *(from 80-180 mm Hg)*, renal blood flow is kept relatively constant. GFR is also autoregulated when the blood pressure is raised or lowered, vessels upstream of the glomerulus constrict or dilate to modify kidney perfusion. Blood flow rates (in mL/min/g tissue) in different parts of the kidney: Cortex = 4-5 Outer medulla = .7-1 Inner medulla = .2-.25

Muscles involved in inspiration?

Diaphragm, sternocleidomastoid, scalene, and external intercostals

What is the primary determinant of resting Vm?

Diffusion of K+ outward along its concentration gradient.

Limits of diffusion

Diffusion of gases is really fast unless the membrane is thick, then it is diffusion-limited. (Slow Equilibration) Gases typically go by the perfusion limit - dictated by how much blood is available. (Fast Equilibration)

Capillary transport

Diffusion of solutes: Lipid-soluble substances pass through the endothelial cells. (O2, CO2) Bulk flow (Pores, Intercellular clefts): Small water soluble substances pass through the pores (Na+, K+, glucose, amino acids) Vesicular transport (exocytosis, endocytosis, pinocytosis): Exchangeable proteins are moved across by vesicular transport. Plasma proteins generally cannot cross the capillary wall.

Thick ascending limb transporters

Diffusion: Na+, Cl-, K+, Ca2+, Mg2+, and NH4+. Primary active transport: Na+/K+ ATPase. Secondary Active Transport: Na-K-2Cl cotransporter (NKCC), K-Cl cotransporter, Na-H Exchanger. Na+, K+, Cl-, and Cl- all symport from the tubular urine into the thick ascending limb cell via the NKCC cotransporter. First the Na+ goes downhill through the NKCC which then allows secondary active transport of 1 K+ and 2 Cl-. Na+ is then pumped out of the cell at the basolateral cell membrane by Na+/K+ pump. Cl- is pumped out at the basolateral cell membrane via either facilitated diffusion Cl- channel or by K+/Cl- symporter (basolateral side is predominantly permeable to Cl-). K+ is recycled back into the tubular urine via facilitated diffusion K+ channel (luminal cell membrane predominantely permeable to K+). The movement of these ions (Na+, K+, and Cl-) creates a transepithelial potential difference of +6 mV. This potential difference drives small *cations* Na+, K+, Ca2+, Mg2+, and NH4+ out of the lumen in the space between the tubular cells (intercellular space). Summary: Na+/K+/2Cl- symporter in luminal membrane (inhibited by furosemide, bumetanide, and Ethacrynic acid). Impermeable to water Diluting segment

Disease state in terms of physiology definition

Disease: A state of disrupted homeostasis. Once out of homeostasis the body won't get back in because the mechanisms that maintained it can't, example BP elevated, your body isn't going to bring it down to homeostatic levels.

pKa H2PO4-

Dissociation constant for H2PO4- The pKa of H2PO4 is 6.8 Phosphates are a better biological buffer However, we have far less phosphate buffers in the body though.

pKa HCO3-

Dissociation constant for HCO3- Ka = [H+] x [A-]/ [HA] The pKa of HCO3- is 6.1. This is not perfect, ideally the best buffers have a pKa near the pH you want to maintain.

Distal nephron

Distal nephron includes: distal convoluted tubule (early), collecting/connecting tubule (late, permeable to water), the cortical, outer medullary, and inner, medullary collecting ducts (collecting ducts aren't part of the nephron but functionally justified to include as part of distal nephron). Distal nephron vs proximal tubule: -Distal nephron reabsorbs much smaller amount of salt and water. - Distal nephron can establish steep gradients for salt and water. (For example, the concentration of Na+ in the final urine may be as low as 1 mEq/L (vs. 140 mEq/L in plasma) and the urine osmolality can be almost one tenth that of plasma.) - Distal nephron has a "tight" epithelium, where as the proximal tubule has a "leaky" epithelium. (Tight epithelium allows it to establish steep gradients). - Na+ and water reabsorption in the distal nephron are uncoupled because the water permeability is variable. In distal nephron: Na+ and Cl- are transported from the lumen into the cell by an Na-Cl cotransporter (symport) that is *inhibited by thiazide diuretics*. Na+ is pumped out of the basolateral side by the Na/K- ATPase. Water permeability of the *distal convoluted tubule is low and is not changed by arginine vasopressin (ADH).* *Na+/Cl-* cotransporter (inhibited by thiazide diuretics). (This is in the *distal convoluted tubule cells*) Absorbs salts and dilutes the tube Impermeable to water

What can block Na-K-2Cl transporter?

Diuretic drugs (loop diuretics), furosemide, ethacrynic acid, and bumetanide.

Metabolic alkalosis can be caused by?

Diuretics, increased aldosterone, vomiting *stomach content*, and Aklaline drugs.

Chart summary

Dopamine is an amino acid- the rest are all peptides.

Dorsal (posterior root) vs ventral (anterior root) roots

Dorsal = sensory neurons Ventral = motor

Airflow and pressure changes

During inspiration, the expansion of the chest cavity lowers the pleural pressure which causes the alveolar pressure to decrease. SInce Alveolar pressure is lower than atmospheric pressure, air enters the lung. During expiration the reduction in chest cavity space causes the alveolar pressure to increase. Since alveolar pressure is higher than atmospheric pressure, air leaves the lung.

Fluid balance

During out normal daily routines we gain and lose fluid through intake and output. Liquid consumption and Renal output are highly modifiable to balance any additional changes that are necessary. In a normal day, about 2.4 L of input and 2.4 L of output. Failure of water balance leads to edema or dehydration. Daily fluid balance works to maintain overall homeostasis by preventing edema or dehydration. Our bodies continually must adjust out fluid levels based on the environment around us and our perceived needs of water. (based on if we'll encounter water). This means our kidneys must alter their function throughout the day to compensate for different fluid levels. *Renal output is the main one controlling fluid balance.*

Which of the following is true about segmentation of the small intestine? a. It is a type of peristalsis b. It moves chyme only from the duodenum to the ileum c. Its frequency is the same in each intestinal segment d. It is unaffected by cephalic phase stimuli e. It produces a slow migration of chyme to the large intestine

E - Although the primary movement of cyme in segmentation is back and forth, the overall, net movement of chime is from the small intestine to the large intestine.

Glucose-galactose malabsorption is a rare disorder caused by mutations in SGLT-1. Infants with this disorder develop severe osmotic diarrhea if they consume certain carbohydrates. Of the following, which would NOT be expected to cause symptoms in these patients? A. Sucrose B. Glucose C. Amylopectin D. Lactose E. Fructose

E. Fructose

A 50 year old man who is markedly overweight comes to his primary care physician complaining that he suffers nightly from a burning sensation in his chest after retiring which is made worse if he has had a snack close to bedtime. Which of the following would be the most appropriate treatment for this patient if his symptoms are not resolved by weight loss and eliminating nighttime meals? A. Cholinergic agonist. B. Smooth muscle relaxation C. Nitric oxide donor D. Nicotinic agonist E. Proton pump inhibitor

E. Proton pump inhibitor

In an experiment, rabbits are administered a cholinergic agonist, pentagastrin, or histamine intravenously, and gastric acid secretion measured. Which treatment, when coadministered with each of these agents, would be expected to block gastric acid secretion produced by any of the stimuli? A. Histamine H2 antagonist B. Antibodies to gastrin C. Anticholinergic drug D. Histamine H1 antagonist E. Proton pump inhibitor

E. Proton pump inhibitor

A 77 year old woman was found to have a blockage in the blood vessel connecting the hypothalamus and anterior pituitary. As a result, which hormone will be elevated in the peripheral circulation? A. Oxytocin B. Growth hormone C. ADH D. ACTH E. TRH

E. TRH

What is used to indicate preload?

EDV (end diastolic volume) and EDP (end diastolic pressure) in the left ventricle. The more the ventricle stretches, the higher the preload = the higher the stroke volume = the higher the cardiac output. Stretches to ideal overlap of actin and myosin.

Sympathetic regulation of ventricular myocytes

Effects of sympathetic stimulation: B-adrenergic signaling (B1). 1. Lusitropy (increase lusitropy, increase relaxation). Catecholamines -> B-adrenergic receptor (B1) -> Gs -> adenylate cyclase -> cAMP -> PKA -> Phosphorylates PLN (phospholambin) which causes it to detach from SERCA and allow sequestering of calcium back from cytosol to sarcoplasmic reticulum to be reused in contraction. 2. Inotropy (increase inotropy, increase contractility). Catecholamines -> B-adrenergic receptor (B1) -> Gs -> Adenylate cyclase -> cAMP -> PKA -> phosphorylates L-type calcium channels and enhances its activity (more calcium comes in from t tubule/extracellularly). PKA also phosphorylates ranitidine channel (RyR2) and enhances it allowing more calcium to be released from sarcoplasmic reticulum. Ultimately, more calcium increases troponin sensitivity to Ca2+ and causes more cross- bridges = more force. Increases SV.

Creatinine Clearance

End-product of muscle metabolism Produced continuously in the body and is excreted in the urine. Does not have to be infused. C(creatinine) = U(creatinine) x V/ P(creatinine) Amount filtered (Pcr x GFR) = Amount excreted (Ucr x V) Cons: - Secreted by the kidneys (+20%) - The colorimetric method to detect plasma Creatinine also detects solutes such as glucose. (+20%). - *These cancel out.* One adds 20% to the numerator and the other to the denominator. ------------------------------------ Inulin clearance is the gold standard for measuring GFR and is used whenever highly accurate measurements of GFR are necessary. It is more common, however, to utilize an endogenous substance that is only filtered, is excreted in the urine, and normally has a stable plasma valvue that can be accurately measured. There is not a perfect substance for this but creatinine comes close. Creatinine is an end-product of muscle metabolism, a derivative of muscle creatine phosphate. It is produced continuously in the body and is excreted in the urine. Creatinine concentrations in the plasma are normally stable, so no outside infusions are necessary. Plasma and urine concentrations can be measured using a simple colorimetric method. (Watch out for people who supplement with creatine)

How can very large molecules be moved across membrane?

Endocytosis: Movement into cell. (Phagocytosis) and Exocytosis: Movement out of cell

Which reflex is the communication of the small intestine to the stomach to slow down gastric emptying.

Enterogastric reflex

Second law of thermodynamics

Entropy, or randomness, of a system is always increasing.

Equilibrium vs steady state

Equilibrium: When opposing forces are balanced (often in terms of concentrations of solutes). No net movement of solutes/energy Steady state: Is dynamic not static. Solutes can move around but the overall concentrations are balanced. Typically steady state requires energy.

Where are multi-unit smooth muscles found in GI?

Esophagus and Gallbladder

GI secretions: pathophysiology Zollinger-Ellison Syndrome

Etiology: Gastrin-secreting tumor; located in the pancreas, intestinal tissues, lymph nodes Pathophysiology: - Gastrin stimulates growth of stomach epithelium - Increased parietal cells - Additional secretion of HCl Signs/Symptoms: - Chronically elevated gastrin levels - Peptic ulcer signs/symptoms Verification Tests: - Secretin stimulation test Treatment: - Surgery - Pharmacology for acid

GI Digestion and Absorption: Pathophysiology Cystic Fibrosis

Etiology: Mutation in the apical membrane Cl- channel. Pathophysiology: - Loss of Cl- in the lumen - Decreased aqueous secretions - Decreased HCO3- due to loss of exchange Gastrointestinal symptoms - Meconium ileus - Lack of pancreatic enzymes - Failure to thrive

GI Secretions: Pathophysiology Peptic ulcer disease

Etiology: ulcers within the stomach and duodenum. Causes: - Imbalance between mucous production and acid secretion - Stress (increase in adrenergic stimulation decreased HCO3- secretion). - Zollinger-Ellison syndrome - Smoking and alcohol - Helicobacter pylori infection Signs/symptoms - Bloating - Heartburn - Nausea or vomiting - Gnawing or burning pain in the stomach Treatment: - Proton-pump inhibitor - Antibiotics - Endoscopy - Surgery

Nernst Equation

Ex = 60/Z * log(Co/Ci) Ex = Equilibrium potential Zx= charge (ex. Cl- = -1, Ca2+ = +2) Co= Concentration outside cell Ci= Concentration inside cell These calculations used to find the equilibrium potential inside the cell

Cell responses to extracellular receptors

Example: RTK receptors. Extracellular receptors are usually a fast response but can do both fast and slow responses. 1. Activation of enzymes/signaling pathways 2. Gene expression Signaling molecule -> receptor protein -> intracellular signaling protein -> target proteins -> multiple functions

Feedforward control

Example: Right before you begin to run your heart rate and respiratory rate goes up in anticipation of the run.

Extracellular receptor: ionotropic (ion channel linked)

Example: ligand-gated ion channels on post synaptic membranes. Neurotransmitters (acetylcholine, glutamate). Receptors that are directly coupled to ion channels. When a neurotransmitter binds, the ion channel opens and the ions enter.

Cations and anions extracellularly vs intracellularly

Extracellular: Na+ main cation also Ca2+ very small cation effect Cl- main anion also HCO3- anion small effect Intracellular: K+ main cation also Mg2+ very small cation effect PO4 and organic anions main anion also proteins anion small effect

Extracellular vs intracellular fluid

Extracellular: 1/3 of Total Body Water. Includes interstitial fluid and plasma. Whenever talking about serum concentration etc. its referring to extracellular. 1/5 of extracellular fluid is in constant motion. Intracellular: 2/3 of Total Body Water. Fluid inside the actual cells. Separated by extracellular fluid via cell membrane.

Pulmonary function tests

FEV1: Forced Expiration in 1 second. Most expiration occurs in 1 second so this test is important to know if you have an obstruction. FEF 25-75: Forced expiratory flow 25-75%. The steeper the slope the better the breathing. The wider the slope the worse the breathing. Shows how bad the obstruction is based on the steepness of the slope. FVC (VC): Functional Vital Capacity. Everything - RV. The slopes and timing of the lines from the pulmonary function test, spirometry exam... tell us information how well a patient is breathing.

Chloride Bicarb Exchanger

Facilitated diffusion via carrier: AE1 (Anion Exchange protein 1). Swaps 1 Cl- for 1 HCO3- Used in bicarb buffer found in blood

Vitamins digestion and absorption

Fat soluble (A, D, E, K): Absorbed with fat in micelles Water soluble: mainly absorbed in Na+ dependent transport. *Vitamin B12* (cobalamine): requires binding proteins (Intrinsic Factor).

Transpiration

Fluid moving through barriers it shouldn't have. Important in burn victims.

What are the *pyloric gland* cells and what functions do they have?

Found in the stomach and are involved in gastric secretions. *Mucous Neck Cells (surface cells)*: -viscid alkaline coating. *G-Cells*: - Secrete Gastrin - Stimulates Parietal Cells - Stimulates ECL cells *ECL Cell (Enterochromaffin Cells)*: - Secrete Histamine - Located in the gastric glands - Stimulated by Gastrin, acetylcholine, and other enteric hormones.

Frank-Starling Law

Frank-Starling Law: the strength of the heart's systolic contraction is directly proportional to its diastolic expansion with the result that under normal physiological conditions the heart pumps out of the right atrium all the blood returned to it without letting any back up in the veins. - This results in numerous possible curves, depending on the inotropic state, preload, and afterload. Important mechanism by which the heart keeps the blood moving and not backing up. Decrease in venous return = decrease in EDV.

Cardiac tissue (4)

From heart going out: Endocardium: Covers the inner surfaces of the heart. Covers the insides, the atria/ventricles, etc. Myocardium: Muscular wall of the heart consisting primarily of cardiac muscle cells. (has connective tissues) Epicardium: Covers the outer surface of the heart; also called the visceral pericardium. --Pericardial cavity (contains serous fluid) the fluid space between the epicardium and the parietal pericardium.--- Parietal pericardium: The serous membrane that forms the outer wall of the pericardial cavity; it and a dense fibrous layer form the pericardial sac surrounding the heart.

RR interval

From one R of QRS to the other R of QRS = x. Then take 60/x = HR.

GI Motility: Large Intestine

Functional Roles: - Storage - Movements Mixing Movements: - Haustrations Propulsive movements: - Mass movements *Gastrocolic Reflex*: Increases the frequency of mass movements.

Lung volume equations. TLC FRC RV IC ERV VC

Functional residual capacity calculated by knowing the other ones. IRV = IC - Tv

Endocrine Pancreas

Functional unit: *Islet of Langerhans* *Insulin*: Beta cells, released when blood glucose is high. *Glucagon*: Alpha cells, released when blood glucose is low. (Just know the alpha and beta cells.)

Thyroid gland anatomy

Functional unit: follicle Cuboidal epithelial cells surrounding an apical colloid gel containing thyroglobulin. Apical side faces colloid.

GFR and Starling Forces

GFR = Kf x UP = Kf x (Pgc - Pbs - COP) UP- net ultrafiltration pressure gradient. COP- colloid osmotic pressures (capillary osmotic pressure) Kf- glomerular ultrafiltration coefficient Pgc- glomerular capillary hydrostatic pressure Pbs- hydrostatic pressure in the space of the Bowman capsule. GFR depends on the balance of hydrostatic and colloid osmotic pressures (capillary osmotic pressure) acting across the glomerular filtration barrier. The high levels of fluid and solutes lost in the glomerulus necessitate a higher capillary pressure. This maintains positive pressure out into the tubules and collecting ducts. *Glomerular hydrostatic pressure* = 60 mmHg *Glomerular colloid osmotic pressure (capillary osmotic)* = 32 mmHg. *Bowman's capsule hydrostatic pressure* = 18 mmHg Glomerular hydrostatic pressure (60) - Bowman's hydrostatic pressure (18) - Glomerular oncotic pressure (32) = *Net filtration pressure (10 mmHg)*

Dysphagia

GI Motility: Pathophysiology Dysphagia: difficulty in swallowing Causes: - Failure of pharynx muscles - Failure of peristalsis in esophagus - Failure of LES (lower esophageal sphincter) to relax - Esophageal cancer.

GI anatomy: Wall Histology

GI wall histology is fairly consistent along the entire length of the GI tract, but some areas have deeper villi and glands. From inside GI to outside: - Epithelial lining with mucosal gland inside. - Mucosa - Mucosal muscle (muscularis mucosa) - Submucosa with Meissner's nerve complex and submucosal gland inside. - Circular muscle with mysenteric nerve plexus inside - Longitudinal muscle - Serosa (Mesentery covers outside)

Insulin Actions: Adipocyte

GLUT4 Enhances glycolysis (glucose used for ATP) *Lipogenic*: - Activates fatty acid synthase - Activates lipoprotein lipase (LPL) - Inhibits hormone-sensitive lipase (HSL).

Insulin Actions: Myocyte (muscle cell)

GLUT4 transporter Glucose used for ATP Promotes: - Uptake of amino acids - Uptake of fatty acids - Glycogen synthesis - Protein synthesis

Glucose transporters and locations

GLUT4: Muscle and adipose GLUT2: Pancreatic beta cells

Which G-protein subunit has impact on enzymatic activity

Ga (alpha) Extra: G-protein is 7 transmembrane protein with 3 subunits.

Juxtacrine signaling

Gap junctions allow for information from one cell to pass directly to the adjacent cell. Regulated by ligands and voltage. Connexons not connected to an adjacent cell as a gap junction may act as individual ion channels.

Junctional proteins

Gap junctions- connexins Desmosomes- cadherins

Ca2+ effects in cell, pathway.

Gaq-> phospholipase c -> IP3 (and DAG) -> Increased Ca2+ (released from ER/SR) -> Calmodulin -> CaM-Kinase or Ca2+ -> Protein Kinase C (PKC) Increase Ca2+ release into the cytosol can have numerous impacts from muscle contraction to activation of CaM-Kinases. Ca2+ can also bind/activate Protein Kinase C (PKC)

Extracellular: G protein-coupled receptors (GPCRs)

Gas = stimulates adenylyl cyclase 1. Signal molecule binds to receptor 2. The occupied receptor causes replacement of the GDP bound to Gs to GTP, activating Gs. 3. Gs moves to adenylyl cyclase and activates it. 4. Adenylyl cyclase catalyzes the formation of cAMP. 5. cAMP activates PKA 6. PKA -> phosphorylation of cellular proteins which causes the cellular response to the hormone. ( cAMP is degraded by cyclic nucleotide phosphodiesterase to 5'-AMP.) Gai = inhibit adenyly cyclase. Gaq= stimulates phospholipase C (PLC) -> IP3 and DAG. 1. Signal molecule activates receptor and associated G protein. 2. G protein activates phospholipase C (PLC), an amplifier enzyme 3. PLC converts membrane phospholipids into diacylglycerol (DAG), which remains in the membrane and IP3 which diffuses into the cytoplasm. 4. DAG activates protein kinase C (PKC) which phosphorylates proteins. 5. IP3 causes release of Ca2+ from organelles, creating a Ca2+ signal.

Gastric retention and rapid gastric emptying causes and treatment (GI Motility: Pathophysiology)

Gastric retention: Caused by: - Diabetic neuropathy - Vagal nerve damage - Idiopathic (arises spontaneously or cause unknown) Treatment: - Motility-stimulating drugs ------------------------------------ Rapid Gastric Emptying: Caused by: - Distal stomach resection - Surgical pyloroplasty - Dumping syndrome Treatment: Small meals of complex carbs and small volumes of liquid

Which reflex is responsible for the passing of feces by infants and occurs 10-20 minutes after eating by allowing the stomach to communicate to the colon to make room for the incoming digested food?

Gastrocolic reflex

Cystic fibrosis and effect throughout body

General: - Growth failure (malabsorption) - Vitamin deficiency states (vitamins A, D, E, K) Nose and sinuses: - Nasal polyps and sinusitis Liver: - Hepatic steatosis - Portal hypertension Gallbladder: - Biliary cirrhosis - Neonatal obstructive jaundice - Cholelithiasis Bone: - Hypertrophic osteoarthropathy - clubbing. - Arthritis - Osteoporosis Intestines: - Meconium ileus - Meconium peritonitis - Rectal prolapse - Intussusception - Volvulus - Fibrosing colonopathy (strictures) - Appendicitis - Intestinal atresia - Distal intestinal obstruction syndrome - Inguinal hernia Lungs: - Bronchiectasis - Bronchitis - Bronchiolitis - Pneumonia - Atelectasis - Hemoptysis - Pneumothorax - Reactive airway disease - Cor pulmonale - Respiratory failure - Mucoid impaction of the bronchi - Allergic bronchopulmonary aspergillosis Heart: - Right ventricular hypertrophy - Pulmonary artery dilation Spleen: - Hypersplenism Stomach: - GERD Pancreas: - Pancreatitis - Insulin deficiency - Symptomatic hyperglycemia - Diabetes Reproductive - Infertility (aspermia, absence of vas deferens) - Amenorrhea - Delayed puberty

What are the 3 glands involved in salivation? What are the characteristics of salivary secretions (pH, Na, Cl-, K+, and HCO3- content compared to plasma levels, other ions secreted and what organic contents are secreted?

Glands: 1. Parotid 2. Submandibular 3. Sublingual 1 L/day of salivary secretions Tonicity = hypotonic pH = 7.0-8.0 Na+ and Cl- are lower than in plasma K+ and HCO3- are higher than in plasma Other ions include Ca2+, Mg2+, and PO4(3-) Organic contents include proteins (e.g. amylase) and glycoproteins (e.g. mucin).

Reabsorption in the Proximal Tubule

Glomerular capillaries are all filtration. Peritubula capillaries are mostly all reabsorption. Normal GFR: - Hydrostatic capillary pressure of peritubular capillaries = 20. - Capillary Oncotic pressure of pertitubular capillaries = 35. Increased filtration fraction: - Hydrostatic capillary pressure of peritubular capillaries = 17 - Capillary Oncotic pressure of peritubular capillaries = 40. This is because with increased filtration there is more water and solutes being filtrated. So the efferent arteriole leaves with a higher oncotic pressure due to the increased concentration of albumin because it can't leave via filtration. The capillary pressure decreases a bit because there is less pressure due to less water.

Renal circulation pathway

Glomerular capillaries: blood entering/exiting the nephron. Peritubular capillaries: blood supply to tubular cells Vasa recta: blood supply to loops of juxtamedullary nephrons Blood entering and exiting the glomerulus travels in arterioles... but the Glomerulus contains capillaries (only place in the body). Aorta -> Renal artery -> Segmental artery -> Interlobar artery -> Arcuate artery -> Interlobar artery -> Afferent arteriole -> Glomerulus (glomerular capillaries) -> Efferent arteriole -> Peritubular Capillaries and/or Vasa recta -> Interlobar veins -> Arcuate veins -> Interlobar veins -> Renal vein -> Inferior vena cava.

Order within the nephron

Glomerulus -> Bowman's capsule -> Proximal convoluted tubule -> Loop of Henle: (Proximal straight tubule -> Thin descending limb -> Thin ascending limb -> Thick ascending limb) -> Distal convoluted tubule -> collecting duct

Peripheral chemoreceptors

Glomus cells located in the carotid body sense arterial PO2, PCO2, and pH and respond by activating the Glossopharyngeal efferent to signal to the medulla/pons that respiration needs to increase/decrease to maintain levels. CO2 levels are detected by converting CO2 into H+ and HCO3-. The end result is high internal H+ concentrations within the Glomus cells, which inactivate K+ channels and allow for calcium influx.

Glucagon characteristics

Glucagon- secreted when blood glucose concentration decreases. - Peptide hormone - Anti-insulin effects - Hormone of energy starvation (fasting state) - Promotes mobilization and utilization of metabolic fuels - Works primarily on the liver Binds GPCR -> Activates PKA (cAMP) *Raises concentration of glucose and fats in the blood*

Hypothalamic-Pituitary-Adrenal (HPA) Axis

Glucocorticoids (Cortisol): released and controls blood sugar levels (stimulates gluconeogenesis) and regulates metabolism.

What is the primary factor in regulation and synthesis of insulin?

Glucose

Describe the endocrine regulation of the female reproductive tract and ovarian steroidogenesis: the roles of GnRH, FSH, LH, estrogen, progesterone, inhibin (include feedback regulation).

GnRH -> LH and FSH -> Ovaries -> Estrogen + Progesterone -> stimulates reproductive tract and secondary sex characteristics. FSH involved in stimulating growth of ovarian follicles. LH is involved in inducing ovulation. Both LH and FSH regulate follicular steroidogenesis and androgen and estradiol secretion. LH regulates progesterone secretion from the corpus luteum. Ovarian steroids (estradiol and progesterone) inhibit LH and FSH secretion with one exception: Just prior to ovulation (at midcycle), estradiol has a positive-feedback effect on the hypothalamic-pituitary axis and induces significant increases in secretion of GnRH, LH, and FSH. Inhibin produced by the ovaries suppresses the secretion of FSH at the anterior pituitary. Function of Estrogen: - Maturation and maintenance of uterus, fallopian tubes, cervix, and vagina - Responsible at puberty for the development of female secondary sex characteristics - Required for development of the breasts - Responsible for proliferation and development of ovarian granulosa cells - Up-regulation of estrogen, progesterone, and LH receptors - Negative and positive feedback effects on FSH and LH secretion - Maintenance of pregnancy - Lowering uterine threshold to contractile stimuli - - Stimulation of prolactin secretion - Blocking the action of prolactin on the breast - Decreasing LDL cholesterol Anti-osteoporosis Function of Progesterone: - Maintenance of secretory activity of uterus during luteal phase - Development of the breasts - Negative feedback effects on FSH and LH secretion - Maintenance of pregnancy - Raising uterine threshold to contractile stimuli during pregnancy

Myogenic response (local influence)

Goal of autoregulation: To respond to fluctuations in perfusion pressure and maintain normal organ blood flow. - Independent of intact endothelium - Vessel constricts in response to an increase in pressure - Vessel dilates in response to a decrease in pressure Net effect: Maintenance of near constant blood flow for a particular metabolic level. (Makes sure that the blood going into each organ is the same, with high blood pressure there would be more blood going into an organ without constriction. At low blood pressure there would be less blood going to organ without dilation.) P = QR when pressure (P) decreases Q (flow) decreases and therefore R (resistance) must be decreased to maintain flow, therefore vasodilation.

How does a goiter develop in hyperthyroidism? What cell type/tissue does TSH act on? levels of T3/T4 increase or decrease? TRH? TSH? Why did the thyroid gland grow?

Goiter: enlargement of the thyroid gland. *Grave's disease*: autoimmune disease in which the body produces autoantibodies that mimics actions of TSH. 1. TSH acts on follicular epithelial cells. 2. Levels of T3/T4: Increase 3. Levels of TRH: Decrease (negative feedback via T3/T4) 4. Levels of TSH: Decrease (TRH decreased) 5. Why thyroid gland grew? Autoantibody mimics TSH which increases thyroid growth. (Goiter can be caused by hyper or hypo- thyroidism.)

Secretory vesicles characteristics.

Golgi apparatus packages proteins into vesicles to move to other cells or another cell structure. Allows signaling between cells via exocytosis, important in homeostasis. Vesicles form via clathrin-coated pits that form on the membrane (like right angled proteins that encircle and form the vesicles). Once the vesicles leave the golgi apparatus, they shed this clathrin coat. Uncoated vesicles then migrate. Docking of vesicles requires v-Snares and t-Snares to mingle at the membrane of choice. This allows the two membranes to fuse.

Graded potential vs action potential

Graded potentials are what lead to an action potential. Graded potentials require summation, action potentials are all or nothing. Graded potential works with K+, Cl-, Na+, and Ca2+. Action potentials due to Na+ and K+. Action potentials are not graded by stimulus size and do NOT decrease with distance.

What causes there to be high perfusion at the lung base and low perfusion at the apex?

Gravity.

GHRH (growth hormone releasing hormone) and Somatostatin

Growth hormone releasing hormone (GHRH): Somatotrope- stimulatory- growth hormone -> all tissues. Somatostatin (Growth Hormone inhibiting hormone - GHIH) - Somatotrope - inhibitory - growth hormone (GH) -> all tissues. *Growth hormone*: Exerts effects directly on tissues (almost all). - Peptide hormone - Causes growth of body tissues (childhood and adolescence) - Levels peak during puberty and decline with age. - Released in periodic bursts - Maximal levels during sleep SRIF (aka GHIH/somatostatin) binds to Gi which inhibits cAMP -> inhibits Growth Hormone. GHRH bind to Gs -> cAMP -> PKA which transcribed growth hormones which leave via secretory granules.

Proximal tubule reabsorption of HCO3-

H+ secretion (Na+ reabsorption) into the urine binds to filtered HCO3-. Carbonic Anhydrase cleaves H2CO3 into CO2 and H2O (later reabsorbed). CO2 rapidly enters the proximal tubular cell CO2 in the proximal tubule is bound to H2O via Carbonic Anhydrase to create H2CO3 which rapidly dissociates into H+ (to be secreted) and HCO3- which can be co-transported into the peritubular blood with Na+ Summary: H+ to the urine HCO3- to the blood Na+ to the blood too

What changes size during skeletal muscle contraction?

H-band gets smaller. I- band gets smaller. Z lines get closer together. A band doesn't change. M band, M line doesn't move.

HCO3 Control

HCO3- is controlled in the collecting duct. Alpha-intercalated cells have an HCO3-/Cl- antiport on the blood side (basolateral) that send HCO3- into the blood and Cl- into cell. H+ is secreted into the collecting duct urine either through ATP enabled transport or antiport with K+ and ATP. Beta intercalated cells have the HCO3-/Cl- antiport at the luminar side (tubule side) which secretes HCO3- into the collecting duct and Cl- goes into the cell. H+ is actively reabsorbed into blood via ATP or coupled with antiport with K+ and ATP. So Alpha-intercalated cells are responsible for reabsorbing/reclamation of HCO3- in the blood. Beta- intercalated cells are responsible for loss of HCO3- to the urine. If there is excess HCO3- present then it is lost to the urine via b-intercalated cells. If there is a decline in HCO3- then it is reabsorbed via the alpha-intercalated cells.

Channel states of sodium channel

Has 3 states closed, open, and inactivated. Must follow this route: Closed -> open -> inactivated -> closed.

Heart characteristics

Heart situated in mediastinum between lungs 2/3 of its mass is to the left of the midline Approximately the size of a person's closed fist Cone-shaped: pointed "apex" inferior; flat "base" is superior (like an inverted cone, apex on the bottom, base at the top) Heart protected by sternum (anterior) and vertebrae (posterior)

Hierarchy of GI neural control

Hierarchy of five levels of neural organization determines the moment-to-moment behavior of the GI tract. Level 1 is the Enteric Nervous System (ENS), which behaves like an independent integrative nervous system (minibrain) inside the walls of the gut. Level 2 consists of the prevertebral ganglia of the sympathetic nervous system. Levels 3, 4, and 5 are within the CNS. Sympathetic and parasympathetic signals to the digestive tract originate at levels 3 and 4 (central sympathetic and parasympathetic centers) in the medulla oblongata and represent the final common pathways for the outflow of information from the brain to the gut. Level 5 includes higher brain centers that provide input for integrative functions at levels 3 and 4. The frontal cortex (higher brain center (5)) accounts for the projection of an individual's emotional state to the gut. Underlies the symptoms of diarrhea and cramping lower abdominal pain sometimes reported by students during anticipation of a stressful life event.

Why might an individual with hyperparathyroidism develop osteoporosis and kidney stones?

High Ca2+ in plasma which involved breakdown of bone and kidney stone development.

Respiratory alkalosis can be caused by?

High altitude and psychoneurosis

Changes in ECF K+ levels

High levels of extracellular K+ typically cause an increase in intracellular K+. Over time, renal excretion of K+ will occur.

Absorption and secretion through the GI and amount

Homeostasis of GI system: 1200 ml/day water + 1500 salivary + 2000 gastric + 500 bile + 1500 pancreatic +1500 intestinal (primarily small intestine) = 8200 and then 6700 ml absorbed into blood in the small intestine. (1500 ml left to go into large intestine). 1500 in large intestine with 1400 reabsorbed in large intestine = 100 ml water left. 100 ml water exits via feces. 500-800 g solids ingested and 50 g solids excreted.

Chemical excitation of smooth muscle

Hormones/paracrine agents: -Angiotensin II, nitric oxide, prostacyclin, oxytocin, endothelin.

Hypercalcemia vs hypocalcemia

Hypercalcemia: >2.6 mEq/L Ca2+ Hypocalcemia: < 2.1 mEq/L Ca2+

Hyperglycemia and effects

Hyperglycemia -> insulin release from the beta cells of pancreas. -> insulin goes into splenic artery -> triggers body cells to take up glucose from the blood and utilize it in cellular respiration. Inhibits glycogenolysis in liver- glucose is removed from the blood and stored as glycogen in the liver. Inhibits gluconeogenesis - amino acids and free glycerol are NOT converted to glucose in the ER. -> Blood glucose concentration decreases.

Changes to K+ flow chart

Hyperkalemia will cause an increase in K+ secretion. Aldosterone levels will increase when hyperkalemia exists.

K+ uptake in cells

Hyperkalemia will result in several changes: - Increase in insulin release - Increase in Epinephrine release (via cAMP) - Increase in Aldosterone release (via mineralcorticoid receptor) The net impact is to increase the activity of Na+/K+ ATPases to transport K+ into cells and reduce hyperkalemia back to baseline levels.

Hyperkalemia vs hypokalemia

Hyperkalemia: >5.0 mEq/L K+ Hypokalemia: <3.5 mEq/L K+

Hypernatremia vs hyponatremia

Hypernatremia: > 145 mEq/L Na+ Hyponatremia: < 135 mEq/L Na+

What happens to cell in a hypertonic solution? Hypotonic solution?

Hypertonic solution = more solutes extracellularly = cell will shrink. Hypotonic solution = less solutes extracellularly/more solutes intracellularly = cell will swell.

D5NS

Hypertonic then isotonic once dextrose metabolized. Na+ 154 Cl- 154 Glucose 5000 mg/dl Hypertonic used in pulmonary edema and fluid overload

Hypoglycemia

Hypoglycemia -> glucagon release from the alpha cells of pnacreas -> enter splenic artery -> inhibits body cells from taking up glucose from the blood and utilizing it in cellular respiration. Stimulates glycogenolysis in liver - glycogen in the liver is broken down into glucose and released into the blood. Stimulates gluconeogenesis - amino acids and free glycerol are converted to glucose in the *ER* and released into the blood.

Antidiuretic hormone (ADH)/Vasopressin

Hypothalamus (therefore ADH) stimulated by (low blood pressure): Hyperosmolarity Decreased atrial receptor firing Angiotensin II Sympathetic stimulation. Hypothalamus then signals posterior pituitary which releases ADH/vasopressin. ADH/vasopressin causes: Blood vessels to constrict: increased systemic vascular resistance. Fluid reabsorption in the kidneys: Increased blood volume.

Growth hormone actions.

Hypothalamus -> GHRH -> Pituitary growth hormone -> liver and other tissues -> IGF-1 IGFBPs released. 1. Catabolic (short-term) 2. Anabolic (long-term) 3. Increases blood glucose (stress hormone) 4. Growth (height in children)

What are the feedback mechanisms of GHRH/Somatostatin?

Hypothalamus -> GHRH -> somatotroph -> GH -> GH target cells -> IGF-1 (insulin like growth factor 1) Inhibited by: - IGF-1 inhibits somatotroph and hypothalamus - GH inhibits hypothalamus. GH and IGF-1 stimulate the hypothalamus to produce somatostatin which then inhibits Somatotroph GH secretion.

Hypothalamus-Anterior pituitary negative feedback

Hypothalamus -> Releasing hormone (CRH, TRH, GnRH, GHRH) (hypophyseal portal system) -> Anterior lobe cell -> Anterior Pituitary hormone released (systemic circulation) -> Target endocrine gland -> Hormone secreted into systemic circulation -> Negative feedback to anterior lobe cell and to hypothalamus. Increase hormone from target = decrease hormone from hypothalamus/anterior pituitary.

Hypothalamus- Pituitary anatomy

Hypothalamus: - Median eminence- where hypothalamus and pituitary meet. Adenohypophysis: Anterior lobe of pituitary (anterior pituitary) Neurohypophysis: Posterior lobe of pituitary (posterior pituitary).

2/3 D5W & 1/3 NS

Hypotonic 51 Na+ 51 Na+ 3333 glucose mg/dl Change in ECF: 556 mL Change in ICF: 444 mL Hypotonic solutions are used when the cell is dehydrated and fluids need to be put back intracellularly. This happens when patients develop diabetic ketoacidosis (DKA) or hyperosmolar hyperglycemia.

1/2 normal saline

Hypotonic 77 Na+ 77 Cl- Change in ECF: 667 mL Change in ICF: 333 mL Hypotonic solutions are used when the cell is dehydrated and fluids need to be put back intracellularly. This happens when patients develop diabetic ketoacidosis (DKA) or hyperosmolar hyperglycemia.

What's one way you can get pulmonary hypertension

Hypoxia induced vasoconstriction, the capillary arteries constrict due to low concentration of oxygen (below 70 percent = 73 mmHg PO2) causing the adjacent blood vessels to constrict.

Coupling Pharmaco-Mechanical signal in smooth muscle

IP3 pathway and extracellular Ca2+ (depolarization) activate MLCK which leads to contraction. cAMP inhibits MLCK and causes relaxation. cGMP (from NO) stimulates MLCP (myosin light chain phosphatase) and causes relaxation.

Relationship between clearance and Glomerular Filtration Rate (GFR)

If a substance is filtered and not reabsorbed or secreted, such as *inulin*, all of the filtered plasma is cleared of the substance and clearance(Cx) = glomerular filtration rate (GFR). If a substance is filtered, not secreted, and completely reabsorbed, such as *glucose*, none of the filtered plasma is cleared of the substance and clearance (Cx) < glomerular filtration rate (GFR) If a substance is filtered, not secreted, and partially reabsorbed, such as *urea*, only a portion of the filtered plasma is cleared of the substance and clearance (Cx) < glomerular filtration rate (GFR) If a substance is filtered and secreted, but not reabsorbed, such as *hydrogen ions*, all of the filtered plasma is cleared of the substance and the peritubular plasma from which the substance is secreted is also cleared making clearance (Cx) > glomerular filtration rate (GFR) ----------------------------------- Clearance and GFR are not necessarily the same thing. Filtered objects aren't necessarily cleared. However, by determining clearance from known solutes we can determine what fraction was filtered.

How blood pressure can be heard

If blood pressure is 120/80 then: When the cuff is over 120 mmHg: you can't hear anything because no blood flows through the vessel. Between 120 -80: blood flow through the vessel is turbulent whenever the blood pressure exceeds cuff pressure. Intermittent sounds heard as blood pressure fluctuates through cardiac cycle. Below 80: Blood flows through the vessel in smooth laminar fashion, no sound is heard. (listent to korotkoff sounds in brachial artery)

Solute Movement Summary

If going up then it is secreted. If going down then reabsorbed.

Menstruation

If ovum is not fertilized, 2-3 days before the end of the monthly cycle, corpus luteum involutes, estrogen and progesterone decrease to low levels. Menstruation follows: - Caused by lack of estrogen and mainly progesterone. Decreased stimulation of endometrium - Prior to menstruation ( about 48 hrs.), blood vessels become vasospastic. Lack of blood flow, hormones and nutrients cause necrosis in the endometrium - Necrotic tissue, blood, etc cause uterine contraction - During menstruation, large numbers of leukocytes are released along with the necrotic material and blood. As a result of these leukocytes and possibly other factors, the uterus is highly resistant to infection during menstruation, even though the endometrial surfaces are denuded. This is of extreme protective value.

Pulmonary shunt

If the ventilation decreases in one bronchiole then the other alveoli gets more of the ventilation (5 L/min is split V1 = .5 and V2 = 4.5 between the two), if the flow (Q) from the capillaries remains Q = 2.5 L/min in each then the PaO2 will decrease (143 to 66) (arterial partial pressure of O2). (V/Q ratio here is .2) To compensate is hypoxia induced vasoconstriction. The arteries constrict to make the flow match the ventilation so now Q1 = 1.25 and Q2 = 3.75. This allows the PaO2 to increase (66 to 102) and the V/Q ratio increases from .2 to .4. If the alveoli becomes completely necrosed (V/Q = 0) then there will no ventillation to it (V1 = 0) but there will still be a small amount of flow to it Q = 1. The other alveoli would be V = 5 L/min and Q = 4 L/min. This cuases the PaO2 to decrease to 76.

GI Motility: Ileocecal Junction

Ileocecal Junction valve and sphincter: - Prevents backflow into the small intestine. - Sphincter constricted to slow the rate of movement. *Coloileal Relex*

Importance of sub-threshold depolorization

Important as a filtering mechanism, such that unimportant signals are not transmitted to higher centers.

Starling forces in a glomerular capillary

In a typical capillary, pressure is highest at the arteriole end and lowest at the venule end. This causes the standard capillary filtration according to starling forces. Glomerular capillaries have two arterial ends and a very interesting relationship. Their afferent end is higher in pressure than a standard capillary *(~40 mmHg)* and their efferent end is even higher due to constriction of the efferent arteriole keeping fluid in the capillary. This maintains pressure that is necessary to push fluid across the membrane and into the urine. Pressure profiles along a skeletal muscle capillary and a glomerular capillary. (A) In the "typical" skeletal muscle capillary, filtration occurs at the arterial end and absorption at the venous end of the capillary. Interstitial fluid hydrostatic and colloid osmotic pressures (COP) are neglected here because they are roughly equal and counterbalance each other. (B) In the glomerular capillary, glomerular hydrostatic pressure (PGC) (top line) is high and declines only slightly with distance. The bottom (dashed) line represents the hydrostatic pressure in the Bowman capsule (PBS). The middle line is the sum of PBS and the glomerular capillary COP. The difference between PGC and PBS + COP is equal to the net ultrafiltration pressure gradient (UP). Assuming that Kf is uniform along the length of the capillary, filtration rate would be highest at the afferent arteriolar end and lowest at the efferent arteriolar end of the glomerulus.

Differential processing of prohormones

In alpha cells of the pancreas (left), the major bioactive product formed from proglucagon is glucagon itself. It is not currently known whether the other peptides are processed to produce biologically active molecules. In intestinal cells (right), proglucagon is cleaved to produce the four peptides shown. Glicentin is the major glucagon-containing peptide in the intestine. GLP-1, glucagon-like peptide-1; GLP2, glucagon-like peptide-2; IP-1, intervening peptide-1; IP-2, intervening peptide-2.

Myosin characteristics

In muscle cells, the formation of myosin thick filaments into bands creates a motor to move along actin thin filaments. Myosin II has heavy meromyosin (includes head) and light meromyosin, light chains and heavy chains wrap around to form the contractile apparatus in muscle (myosin II). Antiparallel pairs of myosin II dimers assemble to form a myosin thick filament.

Importance of kidney vascularity

In order to filter large volumes of fluid the renal system has to be highly vascularized. Contains several specialized circulatory mechanisms and systems.

Reabsorption and creation of HCO3-

In order to maintain proper acid-base balance HCO3- must be reabsorbed in the nephron after being filtered. 99.9% of filtered HCO3- is reabsorbed. Above the plasma concentration (24 mEq/L), it is excreted to maintain a well regulated pH. *Plasma HCO3- = 24 mEq/L*

Series vs parallel in cardiovascular system

In series: -Right and Left heart are interdependent - Outputs of L/R must match - *Cardiac output = 5-6 L/min* In parallel: - Blood flow to organ system is identical composition (allows every organ system to get same oxygen and nutrient concentration) - Can be controlled independently (allows to decrease blood supply to some organs, ex. sympathetic innervation- decrease blood supply to gut)

What is secretion of K+ controlled by in the collecting duct?

In the collecting duct K+ secretion is largely controlled by the number of Na+/K+ ATPases in the membrane. The number of exchangers present can be modified by *aldosterone*.

A 50 year old male is found to have a tumor in his anterior pituitary which leads to overproduction of the hormone made in the corticotropes. How will this effect levels of the following hormones in the peripheral circulation? CRH, TRH, GH, Somatostatin, ACTH, Oxytocin, Cortisol, T3/T4, ADH, Dopamine, Prolactin

Increase ACTH and cortisol Decrease CRH

Factors affecting afterload.

Increase in *Systemic vascular resistance* causes increase in *aortic pressure* which causes an increase in *afterload.* Decrease in Aortic Compliance causes increase in *aortic pressure* which causes increase in *afterload.* (Both increase in systemic vascular resistance and decrease in aortic compliance causes increase in aortic pressure which increases afterload)

Effects of afterload/inotropy on Frank-Starling

Increase in Afterload: Decrease in SV. Increase in ESV. New curve down and to the right. Increase in Inotropy: Increase in SV. Decrease in ESV. New curve up and to the right. Increase in afterload or decrease in inotropy causes a subsequent increase in EDV which should activate Frank-Starling on next cardiac cycle to attempt to compensate. Moving up and to the left means either: Increase in Inotropy or decrease in Afterload. Moving down and to the right means either: Increase in Afterload or decrease in Inotropy

How does afterload affect length-tension relationship

Increase in afterload causes decrease in stroke volume, increase in ESV. (SV = EDV - ESV) Makes the line move up and to the left.

Effect of contractility on force-velocity curves

Increase in inotropy = increase in contractility. Can increase Fmax and Vmax (unlike preload)

Inotropy effects on Stoke Volume

Increase in inotropy causes the line to shift up and to the left. Increase in inotropy causes increase in SV and decrease in ESV. Causes decrease in EDV on subsequent cycle.

Factors regulating inotropy

Increase in sympathetic nervous system (norepinephrine) and increase in circulating catecholamines causes an increase in ionotropy. Decrease in parasympathetic nervous system (acetylcholine) causes increase in inotropy.

Factors that affect Central Venous pressure

Increase in venous blood volume or increased venous tone (decreased venous compliance) Increase venous blood volume and it increases on the line. Decrease in venous compliance (increased venous tone) and the line goes down and to the right.

Factors affecting preload.

Increase in ventricular compliance, atrial contractility, central venous pressure, thoracic venous blood volume, total blood volume, and venous return (respiration, muscle contraction, gravity) all increase ventricular filling (preload). Increase in heart rate or increase in venous compliance decreases Ventricular Filling (preload).

What does an increase in respiratory rate do to cardiac output?

Increase respiratory rate = Increase cardiac output.

Patient is given a drug that slows conduction through the AV node, what do you expect to happen?

Increased PR interval.

How does a loop diuretic work?

Increased amounts of Na+ in the collecting duct lumen (e.g., as a result of inhibition of Na+ reabsorption by a loop diuretic drug) result in increased entry of Na+ into principal cells, increased activity of the Na+/K+-ATPase, and increased K+ secretion.

ADH: Osmoreceptors

Increased solute concentration of interstitial fluid causes osmoreceptors in the hypothalamus to lose water (water goes to the high solutes found in interstitial fluid) and shrink in size. This causes the neurons to increase firing rate. When placed in a hypertonic solution the cells shrink and that causes *stretch receptors* on the surface of the neuron to become active and leads to increased action potential rates. When placed in a hypotonic solution the cells expand and this closes stretch receptors on the surface of the neuron leading to decreased action potential rates. The shift directly controls the release of vasopressin (ADH) from the terminals of these neurons in the pituitary. There is always at least a little bit of ADH being secreted because you do not want to lose water.

Factors affecting preload

Increases preload (ventricular filling)via increase in: Atrial contractility *Ventricular compliance (ventricles)* Central venous pressure -Thoracic venous blood volume - Total blood volume - Venous return -Respiration -Muscle contraction -Gravity Decreases preload (ventricular filling) via increase in: Heart rate *Venous compliance (Veins, venodilation decreases central venous pressure which decreases preload)

Iv infusion of Hypertonic solution does what to Darrow-Yannet diagram

Increases the osmolarity of both intracellular and extracellular fluid. Decreases the volume of intracellular fluid. Increases the volume of extracellular fluid. Increases the osmolarity of both because more solutes are now in the fluids. Decreases the intracellular fluid because the water leaves the cells. Increases the extracellular fluid because the you just added fluid into extracellular via IV and the fluid from intracellular is coming over to extracellular.

Iv infusion of Isotonic solution does what to Darrow-Yannet diagram

Increases the volume of extracellular fluid

O2 transfer to tissues by diffusion

Increasing blood flow increases interstitial fluid O2 content. Because O2 exchange is perfusion limited, the higher the blood flow the higher the rate of exchange. These curves represent different levels of O2 consumption by the tissues during varying degrees of activity.

cAMP effects

Increasing cAMP causes dissociation of the regulatory and catalytic domains of Protein Kinase A (PKA). PKA activates ion channels, enzymes, and transcription factors through phosphorylation.

Afferent vs. Efferent Resistance changes

Increasing efferent arteriole resistance increases glomerular pressure and increases filtration to a point. Increasing afferent resistance reduced blood flow into the glomerulus and therefore reduces filtration.

Effect of length (preload) on Force-Velocity curves

Increasing length increases the velocity with force is kept the same. Decrease afterload = increase in velocity Increase in preload = increase in velocity Increase in Inotropy = increase in velocity

Length-tension relationship: Cardiac

Increasing the sarcomere length (preload) generates higher tension. There is an optimal resting length for the heart.

Intercalated discs in cardiac muscle

Individual heart muscle cells connected by intercalated discs to work as a single functional organ or syncytium

Interior heart anatomy

Inferior & Superior Vena Cava ->Right atrium -> Tricuspid (right atrioventricular valve) -> Right ventricle -> Pulmonary semilunar valve -> Pulmonary trunk -> Left & right pulmonary arteries -> lungs (arteries->arterioles->capillaries->venules->veins) -> Left and right pulmonary veins -> Left atrium -> Bicuspid/mitral (left atrioventricular valve) -> Left ventricle -> Aortic semilunar valve -> Aorta -> Arteries of each organ -> arterioles of each organ -> capillaries of each organ -> venules of each organ -> veins of each organ -> inferior and superior vena cava. Intraventricular septum separates the left and right heart

Propagation of action potential.

Inputs: to dendrites and soma (cell body) Action potentials initiated in the axon hillock because neither soma nor dendrites contain voltage-gated Na+ or K+ channels. Action potential propagates down the axon since other regions of the axon (distal to initiation zone) do contain Na+ and K+ channels.

PTH increases Ca2+ reabsorption by kidney how?

Insertion of Ca2+ channels into luminal membrane Activation of Ca2+-ATPase on basolateral membrane Activation of Na+-Ca2+ exchanger on basolateral membrane

Inspiration

Inspiration is primarily driven by the Dorsal Respiratory Group. Inspiration begins by an abrupt release from inhibition of the Central Inspiratory Activity (CIA) Integrator neurons. It is not as simple as DRG = inspiration and VRG = expiration. Instead an integrator works to combine several neuronal signals to signal the DRG/VRG complex to increase inspiratory output and ramp up the signals to skeletal muscles. Expiration is the abrupt inhibition of that signal, which simultaneously resets the Integrator region to baseline activity. The PRG and apneustic center is important in maintaining length of inspiration. While the pneumotaxic center appears to turn off the integrator and DRG signals (but is not entirely necessary).

What 2 lung volumes make up the main difference between sexes?

Inspiratory reserve volume: Males: 3.1 L Females: 1.9 L and Expiratory reserve volume: Males: 1.2 L Females: .7 L

Insulin effects on various tissues

Insulin increases movement of GLUT4 into the membranes of adipose and muscle cells. (Moves GLUT4 from cytoplasm into plasma membrane, increases the number of these transporters on the membrane). Increases facilitated diffusion of glucose into cells.

Insulin synthesis

Insulin translated as *pre-proinsulin* and then goes to the rough endoplasmic reticulum and is converted to *proinsulin*. It then goes to the golgi apparatus and is cleaved into B-chain and A-chain dimerized insuline + C-peptide. Insulin: - Peptide hormone - Short half-life - Rapid removal - Stored in secretory granules - Secretion stimulated by elevated glucose

Primary active transport

Integral membrane proteins that directly use metabolic energy to transport ions against their concentration gradient or electrical potential. "Ion Pumps" or "ATPases" Most abundant: Na+/K+-ATPase Others: -Ca2+-ATPase -H+-K+-ATPase -H+-ATPase (proton pump) -ABCA1 (ATP-binding cassette) - Involved in lipid trafficking -OATP - Organic anion transporting polypeptides - transport anion and cation chemicals -F-type ATPase (proton pump) - Mitochondrial proton pump that creates ATP

What are intercalated cells important for and where are they found?

Intercalated cells are found in the collecting/connecting tubule and collecting duct and is important for acid-base balance They can alter H+ and HCO3- concentrations in the urine and blood. *Alpha intercalated cells* take K+ into cell and H+ secreted into collecting duct urine using ATP. Can also just secrete H+ into urine via ATP. Also involved in antiport of HCO3- into blood and Cl- into cell, the Cl- then goes back into blood via Cl- channel. *Acid secreting* *Beta intercalated cells* do almost the exact opposite as alpha intercalated cells. They push H+ into the blood via ATP or by ATP driven antiport with K+ going into the cell. They also secrete HCO3- into the collecting duct urine via antiport with Cl- coming into the cell, leaves via Cl- channel into the blood. *Bicarbonate secreting*

Muscles involved in expiration?

Internal intercostals, external oblique, internal oblique, rectus abdominis, and transversus abdominis.

Electrolyte and protein concentration interstitial fluid vs Intracellular fluid.

Interstitial: Na+= 145 mM K+= 4.5 mM Cl-= 116 mM Protein = 0 mM Intracellular: Na+= 15 mM K+= 120 mM Cl-= 20 mM Protein= 4 mM

What generates the slow waves in the stomach, small intestine, and large intestine?

Intestitial cells of Cajal (pacemaker cells).

Sarcoplasm

Intracellular fluid between myofibrils. Has tons of mitochondria that lay parallel to myofibrils.

Conduction cell type and rate of firing (bpm)

Intrinsic firing rates: SA Node: 60-100 bpm AV Node: 40-60 bpm Purkinje Fibers: 20-40 bpm Bundles of His: 1-20 bpm (SA node intrinsic firing rate is usually 100-110, but HR is usually around 80 bpm. This is due to vagal tone. At rest, vagus nerves innervating the SA node have high activity.)

Inuln and Glomerular Filtration Rate (GFR)

Inulin can be used to directly calculate GFR since it is filtered, not reabsorbed, not secreted, and eventually excreted. *The amount of cleared inulin is the GFR*. *Normal values* for inulin clearance or GFR (corrected to a body surface area of 1.73 m2) are *~110mL/min for young adult women* and *~125 mL/min for young adult men*.

Smooth muscle fiber characteristics (6)

Involuntary Non-striated Tapered/spindle single nucleus Non-branched Much smaller than skeletal muscle

Cardiac muscle cell characteristics

Involuntary Striated Single nucleus Branched Intercalated discs Much smaller than skeletal Numerous mitochondria Have large T-tubules, form a dyad with sarcoplasmic reticulum

Cardiac muscle cell characteristics (7)

Involuntary striated single nucleus branched intercalated discs Much smaller than skeletal muscle Numerous mitochondria

Endothelin-1 (Endothelium) pathway

Involved in pathogenesis of heart failure, coronary spasm, hypertension, pulmonary hypertension. Non-selective ET-1 receptor antagonists- used to treat pulmonary hypertension. Big Endothelin-1 -> Endothelin-1 -> Eta and ETb receptors on smooth muscle cells -> vasoconstriction.

Postsynaptic receptors 2 types.

Ionotropic (ligand gated ion channel) (is an ion channel and receptor): Fast, opens ion channel in response to ligand (typically acetylcholine neurotransmitter) which can depolarize or hyperpolarize the cell. Lasts milliseconds. Metabotropic (G protein coupled receptors): Slower, also open channels but do so in more steps hence it is slower. Lasts seconds. Acetylcholine is the neurotransmitter for both receptors.

ADH and blood volume loss relationship

Is a curved line. Severe blood volume loss also stimulates release of ADH. Despite diluting the blood the effect is to positively impact blood pressure.

Donnan-Gibbs effect

Is a name for the behaviour of charged particles near a semi-permeable membrane that sometimes fail to distribute evenly across the two sides of the membrane. High proportion of negatively charged proteins inside the cell plays a role in this.

Sarcoplasmic reticulum (SR) characteristics

Is considered a type of smooth endoplasmic reticulum. Typically found in myocytes (muscle cells) Different proteins are expressed, not used for protein production/storage. Stores Ca2+ (main function).

What is the tonicity of pancreatic secretions?

Isotonic

Normal saline

Isotonic 154 Na+ 154 Cl- Change in ECF: 1000 mL Change in ICF: 0 mL Isotonic solution used to increase the EXTRACELLULAR fluid volume due to blood loss, surgery, dehydration, fluid loss that has been loss extracellularly.

Ringer's lactate

Isotonic (slightly hypotonic) 130 Na+ 109 Cl- Change in ECF: 900 mL Change in ICF: 100 mL Isotonic solution used to increase the EXTRACELLULAR fluid volume due to blood loss, surgery, dehydration, fluid loss that has been loss extracellularly.

Isotonic vs isometric

Isotonic- Maintain constant tension as muscle changes length -Concentric contraction (force>resistance): muscle shortens -Eccentric contraction (Force <resistance): muscle lengthens. Isometric- Maintain constant length as tension changes. -Isometric contraction: muscle contracts but does not change length.

Phase 2 of cardiac cycle

Isovolumetric Contraction: The AV valves close and the ventricles contract without changing volume (all valves closed) This is the start of ventricular systole (like 1/3 of the whole ventricular systole phase). The Threshold pressure to open semilunar valves is reached here.

Phase 4 of cardiac cycle

Isovolumetric Relaxation: Start of ventricular distole (very small first part of ventricular diastole that start a quarter way before the end of t wave) Semilunar valves are closed in this phase. Pressure drops sharply here. ESV found in this phase.

What happens to the electrolyte concentrations during action potential?

It remains unchanged because action potential involved movement of only a few ions.

Where are potential differences found?

Just at the membrane surface. The bulk solution is electroneutral.

Which solutes more abundant inside of cell

K+ Mg2+ Proteins

Which ion dominates the membrane potential at the resting state? Why?

K+ because the resting membrane has greater permeability for K+ than other ions.

Potassium

K+ is predominately found in the inner cellular fluid. Most filtered K+ is reabsorbed, though not all. Daily intake is a small percentage of total body K+.

Collecting duct K+

K+ secretion/reabsorption is highly modifiable in the collecting duct ranging from 4-150%. This location works to actively maintain K+ levels in the serum. Sodium and most ions are heavily reabsorbed in the Proximal Tubule but K+ is secreted in the collecting duct and secretion can often be higher than the filtered load of K+.

What is a super important function of the kidneys?

Keeping blood in homeostasis by maintaining the acid/base balance via bicarb. Can reabsorb or create more CO2 therefore changing the blood chemistry. The respiratory system and the renal system offer a 2 way open ended system of keeping acid base balance of blood.

Why is control of blood pressure so important? What variables are manipulated to produce changes in arterial pressure?

Keeps BP relatively constant: - Maintenance of minimum BP ensures sufficient blood supply to organs. - Maintenance of maximum BP prevents excess cardiac work and hypertension-induced vascular/organ damage. Variables manipulated? MAP = CO x TPR CO is influenced by SV and HR TPR (also known as systemic vascular resistance SVR) is mainly influenced by radius.

Examples of intermediate filaments

Keratin Desmin Vimentin Neurofilaments Lamins

Metabolic acidosis can be caused by?

Kidney tubule disease, chronic renal failure, Addison's disease (too little aldosterone produced), diarrhea, vomiting *intestinal content*, diabetes mellitus, and ingestion of acids.

How much of cardiac output do the kidneys take up?

Kidneys take 20% of cardiac output. 20% of blood coming from the heart goes directly to the kidneys. Therefore, the kidneys offer the best way to control blood pressure via angiotensin.

Kidney anatomy

Know the *Cortex*, *Medulla*, and *Lobes/pyramids (and that nephrons are in these lobes/pyramids)*, all of this drains into the *ureter*. Capsule Cortex Medulla Lobes (pyramids) Calyxes Hilum Ureter

GnRH in Male Reproductive System

LH = Luteinizing hormone (glycoprotein) FSH = Follicle Stimulating Hormone (Glycoprotein) *LH stimulates Leydig Cells -> produce testosterone* *FSH stimulates Sertoli cells -> testis formation and spermatogenesis* GnRH -> Gonadotrope cells of anterior pituitary -> LH release -> Leydig cells of testis -> Produce testosterone. Negative feedback via testosterone (-) to anterior pituitary, hypothalamus, and CNS behavioral effects. GnRH -> Gonadotrope cells of anterior pituitary -> FSH -> Sertoli cells of testis -> testes formation and spermatogenesis. Negative feedback via inhibin (-) Anterior pituitary and hypothalamus.

Intermediolateral cell columns sit in the

Lateral horns. Lateral horn is only located between T1-L2

What is responsible for the negative resting membrane potentials of cells?

Leaky potassium channels.

Pressure difference inside heart.

Left side of heart: 0-150 mmHg, this is because it must provide enough pressure to squeeze blood to the body. Left ventricle with a lot of pressure. Right side of heart: 0-25 mmHg. Don't have to move it too far only to the lungs via pulmonary arteries, lungs are really close to heart.

Leydig and Sertoli cell physiology

Leydig cell: 1. LH binds to G protein receptor on leydig cell. 2. Activates Gas -> PKA pathway 3. New protein synthesis -> enzymes secreted. 4. Enzymes bind with cholesterol to form testosterone (increase testosterone) 5. Testosterone exits via diffusion to blood stream or through sertoli cell to the lumen. -------------------- Sertoli cell: 1. FSH binds to the G protein on sertoli cells 2. Activates Gas -> PKA pathway 3. New protein synthesis: produces - ABP (Androgen-binding protein) which exits to the lumen - Aromatase: Converts testosterone to estrogen. Estrogen then goes to leydig cell and blood stream and influences new protein synthesis. - Growth factors, other products -> goes to leydig cell and blood stream and influences new protein synthesis. - Inhibins - negative feedback to anterior pituitary and hypothalamus.

Renin-Angiotensin-Aldosterone pathway

Liver makes angiotensinogen, the granular/juxtaglomerular cells make and secrete renin. Renin binds angiotensinogen and converts it to angiotensin I. Angiotensin I is then converted to Angiotensin II via ACE enzyme in the lungs. Angiotensin II then: - Constricts the glomerular efferent arteriole and increases Na+/H+ exchanger activity in the kidneys - Stimulates ADH secretion in posterior pituitary. - Constricts vascular smooth muscle causing hypertension. - Stimulates thirst via the hypothalamus. - Stimulates aldosterone secretion in adrenal cortex. Factors that stimulate renin secretion: 1. Decreased blood pressure. 2. Decreased sodium delivery to the macula densa. 3. Increased sympathetic tone (sympathetic activity).

Atrial (Low-Pressure) Baroreceptors

Locations: 1. Vena Cava 2. Right Atrium 3. Pulmonary Artery Mechano/stretch receptors: - Respond to changes in blood volume

Arterial (high-pressure) baroreceptors

Locations: 1. Carotid sinus 2. Aortic arch Mechano/stretch receptors: Increase in BP causes the walls of the artery to expand which increases the firing rate of the baroreceptors. Decrease in BP decreases stretching of arterial walls and decreases firing rate of the baroreceptors. *Most important mechanism for minute-to-minute BP control*

Factors that influence venous return short term vs long term.

Long term = blood volume Short term = respiratory pump, skeletal muscle pump, sympathetic vasoconstrictor activity.

Filtration

Looking at how well solutes exit the glomerular capillary into Bowman's Space. Actively filtered by the basement membrane and Podocytes. Very leaky capillaries Filtration takes place in the glomerulus of the nephron. There, solutes pass through the basement membrane of the glomerulus and bypass podocytes with foot processes. The function is to passively filter solutes through a mesh like membrane and allow for small metabolites and ions to pass freely into Bowman's space and into the tubules and ducts of the nephron for excretion.

Loop Diuretics

Loop diuretics: Furosemide, Ethacrynic acid, and Bumetanide. NKCC co-transporter is blocked CounterCurrent Multiplication is altered Water reabsorption is reduced.

Na+ deprivation effect on K+ levels flowchart

Low Na+ = Na+ deprivation Low Na+ will lead to increase in K+ secretion via aldosterone but decrease in K+ secretion via other factors. Therefore, *K+ excretion remains unchanged.* This chart explains why sodium deprivation will not change K+ excretion.

Summary: Ca2+ regulation of PTH

Low calcium- stimulates PTH transcription and secretion. High calcium- Inhibits PTH transcription and promotes PTH degradation. If PTH is high for a long time- active vitamin D (calcitriol) will inhibit PTH.

Lymphatic system

Lymphatics help recover excess fluid and protein loss and return it to the central circulation. Not everything that gets filtered gets reabsorbed. That which is not reabsorbed picked up by the lymphatic system and put back into the veins later on. Filtration = reabsorption + lymph flow.

Calculate the MAP, SV, CO, EF, and TPR given: Systolic pressure (aorta) = 124 mmHg Diastolic pressure (aorta) = 82 mmHg R-R interval = 800 msec LV EDV = 140 mL LV ESV = 70 mL Mean pulmonary artery pressure = 15 mmHg Right atrial pressure = 2 mmHg Left atrial pressure = 5 mmHg

MAP = 2/3 diastolic + 1/3 systolic = 96 SV = EDV - ESV = 70 CO = SV x HR (HR = 60/ r-r interval (.8) = 75) = 5,250 EF = (EDV - ESV)/ EDV = 50% TPR = MAP/CO = .018

Mean Arterial Pressure (MAP)

MAP = 2/3 diastolic pressure + 1/3 systolic pressure. Measures the average blood pressure in the arteries. Heart spends 2/3 of its time in diastole (filling up) and only 1/3 of its time in systole.

Arteriolar tone: regulate organ blood flow

MAP is maintained centrally by adjusting CO and TPR. At the level of each organ, resistance to flow is locally regulated by adjusting the radius of the arterioles. (precapillary sphincters)

GI Motility: Stomach- Gastric Reservoir

Main functions: 1. Accommodate arrival of a meal. 2. Maintain constant compressive forces on the contents. Three types of Relaxation: 1. Receptive Relaxation: triggered by swallowing. 2. Adaptive Relaxation: triggered by distension. 3. Feedback Relaxation: triggered by small intestine. Vagotomy: loss of adaptive relaxation. - Patient's have higher intragastric pressures and lower threshold for fullness and epigastric pain. - Loss of inhibitory signals lead to more tonic contractions.

Carbohydrate digestion and absortion

Major Carbs: - Starch - Lactose - Sucrose Process: - Major breakdown by salivary and pancreatic enzymes - Brush Border enzymes finalize digestion - Transporters move simple sugars into the blood stream: GLUT and SGLT.

Lipids digestion and absorption

Major Fats: - Triglycerides - Phospholipids Process: - Emulsification - breakdown of large fat globules into small ones - Enzymes hydrolyze fats once emulsified - Absorbed via passive and co-transport mechanisms - Re-esterification of free fatty acids into chylomicrons (apoproteins and lipid carrying particles).

Potassium balance

Major osmotically active solute in cells. Determinant of cell excitability. Sets the resting membrane potential. - Low plasma K+ = hyperpolarization and reduced excitability. - High plasma K+ = depolarization and increased excitability. Sets acid-base status - K+ depletion = metabolic alkalosis - K+ excess = metabolic acidosis Factors that shift K+ to outside of cells: - Decrease in extracellular pH (antiport with H+ going into cell and K+ outside). - Digitalis. (Digoxen for heart failure and Afib, inhibits the Na/K ATPase causing more extracellular K+) - Lack of O2 - Hyperosmolality - Hemolysis - Infection - Ischemia - Trauma Factors that shift K+ to inside of cells: - Increase in extracellular pH. (H+ goes outside and K+ goes into cell) - Insulin - Epinephrine Potassium levels are vital to survival and therefore are tightly regulated to maintain normal cell excitability and cell volume. Changes to potassium levels can have very negative impacts on overall health. Hyperkalemia tends to depolarize cells away from the normal resting potential and can cause hyperexcitability and things like cardiac arrhythmia such as v-fib. Hypokalemia tends to hyperpolarize cells making action potentials much more difficult. It will lead to cell shrinking.

Protein digestion and absorption

Major proteins: - Meats - Eggs - Legumes - Dairy Protein absorption: - Amino Acids - transporters - Small Peptides - secondary active transport - Proteins - some endocytosed

Male vs female lung capacity and tidal volume

Male: Lung capacity = 5.8 L Female: Lung capacity = 4.2 L Both has same tidal volume = .5 L

What is the Expiratory reserve volume for males? Females?

Male: 1.2 L Female: .7 L

What is the Residual volume for males? Females?

Males: 1.2 L Females: 1.1 L Pretty much the same

What is the Inspiratory Reserve Volume for males? Females?

Males: 3.1 L Females: 1.9 L

What is the vital capacity for males? Females?

Males: 4.8 L Females: 3.1 L

What is the total lung capacity for males? Females?

Males: 5.8 L Females: 4.2 L

Brain (CNS) Ischemic Response

Massive ↓ BP -> ↓ Cerebral Blood Flow -> cerebral ischemia -> Activate vasomotor center (medulla oblongata (brain stem)) -> ↑ Sympathetic Activity -> ↑ Blood Pressure

GI Motility: Esophagus- Mastication

Mastication - chewing of food via movements of the jaws and teeth and muscle actions of the tongue and oral cavity. - Muscles are controlled by 5th cranial nerve and brainstem - Reflex stimulated by sight and tastes - Salivation aids chewing Chewing reflex- Involuntary - Mechanoreceptors in the mouth send sensory signals to coordinate muscle movements.

Mechanical junctions and gap junctions at intercalated discs of myocardium

Mechanical Junctions (made of cadherins) -Fascia adherens - anchors actin (thin filaments) to plasma membrane -Desmosomes - bind intermediate filaments of adjoining cells Gap Junctions (made of connexins) -ion flow between cells, propagate APs to neighboring cells

Aldosterone mechanism

Mechanism: - Receptor signaling increases number of Na+/K+ ATPase and ENaC channels (Epithelial sodium channels). Sodium channels (ENaC) on the luminal side, NA/K-ATPase on basolateral side. Physiological response: - Na+ and Cl- reabsorption - Water reabsorption (because it moves with Na+) - K+ excretion. 1. Aldosterone combines with a cytoplasmic receptor. 2. Hormone-receptor complex initiates transcription in the nucleus. 3. New protein channels and pumps are made 4. Aldosterone-induced proteins modify existing proteins 5. Result is increased Na+ reabsorption and K+ secretion

What provides the arterial blood supply for the gut

Mesenteric circulation which attaches to the villus of the GI and offers a massive supply of blood. Regulation of blood flow controlled by metabolic and neurological inputs.

Gastrointestinal circulation

Mesenteric circulation: arterial blood supply to the gut. Villus circulations offer massive supply of blood Metabolic and neurological regulation of flow.

Amino Acids

Metabolism produces by products Some amino acids are metabolized to produce proteins in the body The end result is reclamation of HCO3- (a-ketoglutarate turns into 2 HCO3- which is reabsorbed) into the circulatory system and loss of NH3 and NH4 in the urine. Urinary excretion of: NH4+ = 40 mmol/day Urea = 450 mmol/day

Mineralcorticoid vs Glucocorticoid vs Androgens vs Catecholamines

Mineralcorticoids: Aldosterone (steroid hormone in adrenal cortex) Glucocorticoid: Cortisol (steroid hormone in zona fasciculata of adrenal cortex). Androgens: Testosterone, Dihydrotestosterone, DHEA, etc. Catecholamines: Epi and Norepi (Amino acid (tyrosine) derived hormones in adrenal medulla)

Urine formation

Modifying blood flow into the Glomerulus can alter filtration. We can ultimately control excretion by modifying filtration *Filtration*: Ultrafiltration of plasma in the glomerulus. Goes from the glomerular capillaries into the Bowman's space. *Reabsorption*: Transport of substances out of tubular urine (ex: Na+, K+, Ca2+, Mg2+, Cl-, HCO3-, phosphate). Kidney tubule -> peritubular capillaries and/or vasa recta. *Secretion*: Transport of substances into the tubular urine (Ex: H+, ammonia) Peritubular capillaries -> kidney tubule. *Excretion*: elimination via the urine *Excreted* = Filtered - Reabsorbed + Secreted.

Factors that affect force of muscle

More calcium = more crossbridges = more force. More muscle fibers/motor units recruited = more force. (spatial/temporal summation)

HCO3- in the nephron

Most HCO3- is reabsorbed in the proximal tubule Some HCO3- is reabsorbed in the thick ascending limb and some in the collecting duct. We excrete very little HCO3- because it's such an important buffer. We can also generate new HCO3-

Na+ reabsorption

Most Na+ is reabsorbed in the proximal tubule (~70%). The overwhelming remainder is reabsorbed in the Ascending limb of the Loop of Henle (~25%). The net loss of filtered Na+ is around 0.4%.

Concentration of urea

Most urea becomes trapped in the inner medulla. Na+ and Cl- concentrations pretty much become saturated at the border of the outer and inner medulla (their concentrations don't change past it). The main driving force for the high osmolarity/concentration seen at the bottom of the loop of Henle is essentially urea.

Myosin II

Motor protein Two intertwined heavy chains (also regulatory and essential light chains)

Each section of GI and role (GI anatomy).

Mouth: Breaks up food particles. Assists in producing spoken language. Salivary glands: Saliva moistens and lubricates food. Amylase digests polysaccharides. Pharynx: swallows Esophagus: Transports food. Liver: Breaks down and builds up many biological molecules. Stores vitamins and iron. Destroys old blood cells. Destroys poisons. Bile aids in digestion (created by liver stored in gallbladder). Gallbladder: Stores and concentrates bile. Stomach: Stores and churns food. Pepsin digest protein. HCL activates enzymes, breaks up food, kills germs. Mucus protects stomach wall. Limited absorption. Pancreas: Hormones regulate blood glucose levels. Bicarbonates neutralize stomach acid. Trypsin and chymotrypsin digest proteins. AMylase digest polysaccharides. Lipase digests lipids. Small intestine: Completes digestion. Mucus protects gut wall. Absorbs nutrients, most water. Peptidase digests proteins. Sucrases digest sugars. Amylase digests polysaccharides. Large intestine: Reabsorbs some water and ions, forms and stores feces. Rectum: Stores and expels feces. Anus: Opening for elimination of feces.

Movement of air into the lungs

Movement is by bulk flow from branches 0 - 16 with velocity decreasing as you go down. When you get to the 17 branch- air is no longer moving due to bulk flow, starts moving through diffusion. Branching increases cross sectional area and decreases resistance- as you go further down resistance drops substantially.

What are the gastric secretions (4)

Mucus: protects stomach lining Acid (HCL): kills bacteria in food Hormones (gastrin)- regulate GI function Proteins - pepsinogen (protein digestion), intrinsic factor (vitamin B12 absorption).

Which type of smooth muscle of GI tract will only contract via nervous input?

Multi-unit smooth muscle Needs neural or endocrine influence to contract. Does not contract in response to stretching.

The two cholinergic receptor types are nicotinic and ________.

Muscarinic

GI Neural control

Muscle activity- under constant neural and hormonal regulation. Varying resting potential - mechanical, neural, and hormonal. *Slow waves*- slow changes in resting potential, vary along GI tract. - Ca2+ depolarization (causes contraction in the stomach). *Spike potentials*- generated by increased slow wave potentials. - Depolarization via Na+/Ca2+ channels. Interstitial cells of Cajal generate the electrical slow waves in the stomach, small intestine, and large intestines. These pacemaker networks are located surrounding the small intestinal circular muscle at the border with the longitudinal muscle layer (myenteric plexus) and at its border with the submucosa (submucosal plexus).

Passive tension

Muscle is being lengthened while in a passive state (not stimulated to contract, no ATP required) tension of muscle during relaxation. Property of structural proteins in muscle (titin), not part of the cross-bridge.

How do twitches vary among muscles

Muscles have an all or nothing character, so every twitch results in the same magnitude and shape (unless twitches follow one another closely). The difference is the size of the muscle fiber and differences in the speed of contraction among fibers.

GI Anatomy: Muscles

Muscles of the GI tracts: Smooth muscles: - Longitudinal - propulsion and mixing - Circular - Segementation and mixing - Muscularis mucosa - folding of the villi. *Muscularis mucosa can change size during digestion to optimize function of glands.*

Which smooth muscle of GI tract is responsible for the folding of the villi and can change size during digestion to optimize function of gland? Where is it located?

Muscularis mucosa Supports the mucosa, is located between the mucosa and the submucosa.

Nodes of ranvier and myelin

Myelin: formed in schwann cells in PNS and oligodendrocytes in CNS. Nodes of ranvier: Are unmylinated sections between the myelin sheaths. Have high density of voltage-gated Na+ channels, action potentials occur ONLY in the node of Ranvier in myelinated axons. Saltatory condution: the jumping of action potential from node to node. (myelinated neurons)

Where can you find white ramus

Myelinated axons- only present at spinal cord levels T1 through L2, therefore present in all preganglionic sympathetic axons.

What also acts as a motor protein to carry secretory vesicles around the cells?

Myosin

Myosin and ATP relationship. Crossbridge? Powerstroke?

Myosin + ATP = low energy position Myosin + ADP + Pi = high energy "cocked" position. Cross-bridge: Myosin + ADP + Pi binds to actin Powerstroke: ADP and Pi release from myosin allows stored energy to be expended as conformational change

Sympathetic responses generally are widespread because?

NE and epinephrine are secreted into the blood as part of the sympathetic response.

Net Filtration Pressure

NFP = (Pc + πi) - (Pi + πc) If (Pc + πi) > (Pi + πc): then positive number and filtration occurs (Capillaries -> tissues, seen at arterial end) If (Pc + πi) < (Pi + πc): then negative number and reabsorption occurs (Tissues -> capillaries, seen at venous end). Hydrostatic pressure drops along the length of the capillary, so hydrostatic pressure drops and explains why arteriolar end cause filtration while venous end causes reabsorption.

Excretion of acid flow chart in the early proximal tubule

NH3 can be produced to be secreted into the urine where it combines with H+ to form NH4.

Secreting H+

NH4+ acts as a mechanism to trap H+ in the urine. Na+/H+ exchangers cause H+ to enter the urine in the ascending limb. NH3 is secreted and allows for H+ binding. H+ also binds to titratable acids such as H2PO4- As the urine flows along the tubule, from the Bowman's capsule on through the collecting ducts, three processes occur: filtered HCO3- is reabsorbed, titratable acid is formed, and ammonia is added to the tubular urine. All three processes involve urinary acidification by the tubular epithelium. The nature of these processes differ in various part of the nephron. H+ is secreted in the *proximal tubule* through the production of ammonia, mainly from the amino acid glutamate. Ammonia is secreted into the tubular urine either by the diffusion of NH3+, which then combines with a secreted H+ to form NH4+, or via the Na/H exchanger on the tubular membrane which functions to exchange Na+ and NH4+. Similar secretion for H+ exists in the loop of Henle. The tubular fluid is acidified by the secretion of H+ along the ascending limb via a Na/H exchanger and NH4+ is also reabsorbed both passively and as a substitute for K+ in the Na/K/2Cl cotransporter. The distal nephron differs from the proximal tubule in its H+ transport properties. It secretes much less H+ and they are secreted primarily via an electrogenic H+ ATPase. The distal nephron is also lined with very tight tubular cells, thereby making steep urine-to-blood pH gradients possible; urine pH is typically around 6 but can be as low as 4.5. (pH at bottom of loop of Henle at 7.4 because water is allowed in.

Nitric Oxide (Endothelium) pathway

NO released in response to increased flow (shear stress), acetylcholine. NO pathway may be dysfunctional in atherosclerosis, coronary artery disease, hyperlipidemia, hypertension. (In response to increase in flow, increase in acetylcholine) L-arginine + eNOS (enzyme) -> NO (converts GTP to cGMP) -> cGMP -> Vasodilation. (Know L-arginine, eNOS, NO, cGMP, Vasodilation)

Note regarding single-unit vs multi-unit smooth muscles

NOTE: It must be emphasized that most smooth muscles do not show all the characteristics of either single-unit or multi-unit smooth muscles. These two prototypes represent the two extremes in smooth muscle characteristics, with many smooth muscles having overlapping characteristics.

Calculate the equilibrium potential given the following: Extracellular: Na+= 150 mM K+ = 5 mM Intracellular: Na+ = 15 mM K+ = 150 mM

Na = 60/1 * log (150/15) = log (10) = 60 * 1 = +60 mV. K+ = 60/1 * log (5/150) = 60 * log (1/30) = -88.

Which solutes more abundant outside of cell?

Na+ Ca2+ Cl- Glucose HCO3- (slightly)

Sodium

Na+ is predominantly found in the extracellular fluid. Nearly all Na+ that is filtered as part of the blood is reabsorbed in the nephron. Daily intake is a small percentage of total body Na+.

What keeps intracellular pH and HCO3- values above their equilibrium values?

Na-H exchanger and Na+ driven Cl-/HCO3-

HCO3- balance

Nearly all of bicarbonate is reabsorbed.

Baroreceptor reflex

Negative feedback Stimulus: change in BP -> Baroreceptors (pressure and volume receptors) ->(afferent pathway)-> control center: Nucleus Tractus Solitarius (NTS) in medulla (brain stem) -> (Efferent pathway)-> Effectors: Heart and vessels -> Response: restore BP towards normal.

What pressure inflates/deflates lungs

Negative pressure: During inhalation the diaphragm goes down and creates a negative pressure within the chest cavity which creates a pressure differential allowing the air to come into the lungs. Also, the negative pressure in the chest cavity moved the blood from the superior/inferior vena cava into the right atrium. Positive pressure: Positive pressure is when you are getting bagged/CPAP. This forces air into the lungs. Problem with positive pressure is we don't have the negative pressure system for the blood to move from the superior/inferior vena cava to the right atrium. This causes decrease in cardiac output and therefore decrease in blood pressure. (BP (MAP) = CO x TPR)

Osmosis

Net movement of water across semipermeable membrane from area of low solute concentration to area of higher solute concentration.

Control of cardiovascular system outline

Neural influences: - Autonomic Nervous System - Neural reflexes Hormonal influences: - Catecholamines - Renin-Angiotensin System (RAS) - Antidiuretic Hormone (ADH - Other Vasocontrictors/Vasodilators Local influence (intrinsic) - Myogenic response - Metabolic - Endothelium

Erectile dysfunction in the male

Neurological problems, such as trauma to the parasympathetic nerves from prostate surgery, deficient levels of testosterone, and some drugs (nicotine, alcohol, antidepressants) can also contribute to erectile dysfunction. Erectile dysfunction is most often caused by underlying vascular disease. Hypertension, diabetes, and atherosclerosis, reduces the ability of the body's blood vessels, including those in the penis, to dilate. Erectile dysfunction caused by vascular disease can often be successfully treated with phosphodiesterase-5 (PDE-5) inhibitors such as sildenafil (Viagra), vardenafil (Levitra) or tadalafil (Cialis). These drugs increase cyclic GMP levels in the erectile tissue by inhibiting the enzyme phosphodiesterase-5, which rapidly degrades cyclic GMP. Thus, by inhibiting the degradation of cyclic GMP, the PDE-5 inhibitors enhance and prolong the effect of cyclic GMP to cause erection.

Hypothalamus posterior pituitary anatomy

Neurosecretory neurons (magnocellular) originating in the hypothalamus. - Supraoptic nucleus (SON) - Paraventricular nucleus (PVN) Neural connection with axon terminals releasing hormones to capillaries. Unlike the blood stream connection in anterior pituitary. Peptide hormones with similar structures: - Oxytocin: secreted by supraoptic nucleus neurons (SON) - ADH/vasopressin: Secreted by paraventricular nucleus neurons (PVN)

What happens in muscle if there is no calcium? No ATP?

No calcium, then the myosin can't bind actin. No ATP, then the myosin can't unbind from actin (rigor mortis).

Example of how norepinephrine depolarizes smooth muscle of GI

Norepinephrine binds to a1-adrenergic receptors which stimulates the G-protein cascade which activates IP3 which releases Ca2+ from the sarcoplasmic reticulum.

Sympathetic nervous system regulation of pacemaker potential

Norepinephrine or epinephrine binds to B1 -> Gs -> Adenylate cyclase -> cAMP -> protein kinase -> opening of funny channels and T-type calcium channel -> depolarization. Protein kinase increases funny channels and T type calcium channels which increases pacemaker potential. (L type channels also increased by protein kinase but doesn't affect pacemaker potential.) --------------------------------- SNS -> Catecholamines -> Epinephrin to bind to B1 adrenergic receptors. 1. Increases Na+ (funny) permeability (increases steepness of depolarization) 2. Increases Ca2+ permeability in SA and AV node (steepens depolarization). (Increase in heart rate)

Relative levels of T3, T4, and rT3

Normal (euthyroid) situation: - 90% T4 - 10% T3 - < 0.1 % rT3 Iodide Deficiency: - Total T4 and T3 production/secretion *decreases* - Proportion of T3 synthesized/secreted *Increases* (I.e. 50% T3, 50% T4). Complete lack of iodide: - No T4 and T3 production/secretion.

Normal ventilation and normal blood flow levels vs mismatched V/Q

Normal V = 4 L/min Normal Q = 5 L/min Normal V/Q = .8 High V/Q: Ventilation exceeds perfusion: Wasted ventilation, unable to oxygenate any blood: COPD and Emphysema. Low V/Q: Poor ventilation: lack of O2 supply: Chronic bronchitis, asthma, acute pulmonary edema. Dead space: No blood supply for the lungs. (Pulmonary embolus) Shunt: No air enters the alveoli, unoxygenated blood continues to the systemic circulation. (Pneumonia, atelectasis).

Control of respiration

Normal quiet breathing is rhythmically controlled by the brain stem and DRG/VRG.

Obstructive vs restrictive PFTs

Normal: FEV1 (1 second) = 4.0 FVC: 5.0 % = 80 Obstructive: FEV1 (1 second) = 1.3 FVC: 3.1 % = 42 Restrictive: FEV1 (1 second) = 2.8 FVC: 3.1 % = 90 Obstructive Disease - Low FEV1/FVC ratio Restrictive Disease - High FEV1/FVC ratio Both obstructive and restrictive diseases see a reduction in FEV1 and FVC when compared to normal. The difference between the two is in FEV1/FVC ratios. Since Obstructive patients see a bigger impact on expiration, their FEV1 is much much lower than normal and much lower than a restrictive patient. Restrictive patients have a smaller FVC, but they can still expire relatively well. In fact they can push out most of their volume in 1 second (2.8 of the 3.1 FVC). That means they have a higher than normal FEV1/FVC ratio.

Explain how calcium and phosphate regulation would be affected in an individual with renal-1-hydroxylase mutation.

Not getting active vitamin D (calcitriol), both calcium and phosphate plasma levels would be lower.

Partial pressures of O2 and CO2 in ambient dry air, moist trachea, alveolar, systemic arterial blood, and mixed venous blood.

O2: Ambient dry air = 160 Moist tracheal air = 150 Alveolar air = 102 (14%) Systemic arterial blood = 95 Mixed venous blood = 40 CO2: Ambient dry air = 0 Moist tracheal air = 0 Alveolar air = 40 (5%) Systemic arterial blood = 40 Mixed venous blood = 46 Main ones Alveoli = 102 O2 and 40 CO2 Systemic Arterial Blood = 95 O2 and 40 CO2. Mixed venous blood = 40 O2 and 46 CO2. (Systemic arterial blood O2 goes down because some of the oxygen needs to be used to supply the lungs.) (Systemic to venous partial pressure of O2 goes down because the tissues use it)

Obstruction vs restrictive lung disease

Obstructive Lung Disease - Condition which causes an obstruction to the airway. Typically this causes air to become trapped in the lung (hyperinflated) and allows for very poor expiration. Restrictive Lung Disease - Condition where infiltration of particles or damage to the lung has caused macrophage infiltration into the interstitial space. This then leads to a fibrotic tissue that doesn't expand during inspiration (less elastic)

Obstructive lung disease vs restrictive lung disease examples

Obstructive: COPD/emphysema Chronic bronchitis Asthma Cystic fibrosis Restrictive: Pneumonia Tuberculosis

Normal vs obstructive vs restrictive

Obstructive: Bigger reserve volume Smaller FVC FEF(25-75) is less steep. FEV1/FVC ratio is low Restrictive: Normal reserve volume Smaller FVC Smaller TLC FEV1/FVC ratio is normal or higher. FEV1 and FEF(25-75) can only test for obstructive.

Describe the immediate consequences of a blood clot occluding the afferent arteriole or the efferent arteriole

Occluding afferent arteriole: Decreased filtration. Occluding efferent arteriole = Increased filtration. Juxtaglomerular Apparatus (macula densa cells and Juxtaglomerular cells) would be activated due to the lower pressure and would stimulate the renin-angiotensin pathway causing increased blood pressure.

What are the 4 cranial nerves of PNS

Oculomotor Facial Glossopharnygeal (salvation) Vagus (heart and most of gut)

Fluid movement equation

Ohm's Law: Delta P = QR or Q = delta P/R Delta P = Pi - Po (pressure, mmHg) (Pi is inlet pressure, Po is outlet pressure) Q = flow rate (Q, volume/time) R = resistance to flow (mmHg/volume/time) 2 ways blood flow through organ can be changed. - Heart generates Pi Increase in pressure means increase in flow. Resistance is inverserly proportional to flow, increase in resistance means decrease in flow.

Cardiac cycle definition, diastole vs systole

One complete sequence of contraction and relaxation. *Diastole*: Ventricular relaxation. During diastole, the ventricles are relaxed and filling with blood. *Systole*: Ventricular contraction. During systole, the ventricles are actively contracting and pumping blood out of the heart.

CO2 and transport

Only 20% of CO2 carried on hemoglobin. Most is converted to H2CO3 in the red blood cell. 10% dissolved in Plasma 60% bicarbonate in plasma and RBCs 30% as carbamino protein CO2 + H2O via Carbonic Anhydrase -> H2CO3 + H+. HCO3 can then go into the plasma via AE1 transporter (Anion Exchange, antiport) in a process called the "chloride shift": (chloride (Cl-) is pumped into RBC and Bicarbonate (HCO3) is pumped into the plasma) The H+ that is made during Co2 + H2O -> HCO3 + H+ can't leave the RBC wall due to its low permeability. *The H+ ion is responsible for the dissociation of O2*

Diffusion: Channel Proteins

Opening of channels allows for the opening of a pore. These channels typically exist to allow the passage of K+, Na+, Ca2+, Cl-, H2O, and other ions/small molecules. Different types of gates exist to control opening/closing: -Signal molecule (ligand) -Voltage -Pressure -Temperature Diffusion through channel proteins is considered simple diffusion. (Facilitated diffusion is through carrier proteins). No ATP used

Osmolarity vs Osmolality vs Tonicity

Osmolarity - osmoles per liter of solution (Osm/L) Osmolality - osmoles per kilogram of water (Osm/Kg) Tonicity - effective osmoles per volume of solution

Osmolarity vs Osmolality

Osmolarity = mOsm/L. The number of osmoles of solute in a liter of solution (Osmolarity is the concentration of an osmotic solution). Osmolality = mOsm/kg. The number of osmoles of solute in a kilogram of solvent (H20) (in general 1L = 1 Kg) Why? The volume of a solvent remains the same regardless of any changes in pressure or temperature, so it is relatively easier to determine the osmolality. Easier to express in terms of osmolarity in clinical cases. But since fluid volume in body is almost all H2O, terms are interchangeable

Osmolarity and the 3 types

Osmolarity takes into account all solutes including: -Membrane penetrating solutes - Non-penetrating solutes Solutions can be: Isosmotic- same osmotic pressure Hypoosmotic- lower osmotic pressure than surrounding Hyperosmotic- higher osmotic pressure than surrounding Refer to solutes in a solution, so high osmolarity means high solutes

Arginine Vasopressin (AVP) AKA Anti-Diuretic Hormone (ADH)

Osmoreceptors in the hypothalamus cause the release of ADH. These neurons are also sensitive to Angiotensin II and will increase their firing rate if Angiotensin II senses low volume. They are primarily responsive to *Osmolality levels in the plasma.* Changes in urine osmolality are normally brought about largely by changes in plasma levels of arginine vasopressin AKA antidiuretic hormone (ADH). ADH is released in response to a variety of factors including pain, trauma, emotional stress, nausea, fainting, nicotine, morphine, and angiotensin II. The main factors controlling ADH (vasopressin) release, however, is *a change in plasma osmolality*. When plasma osmolality or osmotic pressure increases (or blood pressure decreases), osmoreceptor cells located in the *anterior hypothalamus shrink stimulating the neurons in the paraventricular and supraoptic nuclei to release vasopressin (ADH) into the blood.* ADH then leads to vasoconstriction, behavioral responses of thirst, and increased water reabsorption from the collecting duct to increase blood volume and blood pressure. The release of ADH is also related to the detection of blood pressure by the baroreceptors in that a decrease in blood pressure causes the release of ADH (vasopressin) as well. So decrease in *plasma osmolality* stimulates hypothalamus, by the shrinking of the cells, to release ADH from the posterior pituitary. Decrease in blood pressure sensed by baroreceptors stimulates the release of ADH as well.

Osmotic pressure and equation

Osmotic pressure: Pressure needed to stop movement of water across a selectively permeable membrane that separates a solution from pure water. Osmotic pressure = n (# of particles) x RT (gas constant and temperature) x osmotic coefficient (closer to 1 the more impermeable the solute) x C (concentration of solute in mol/L Formula also known as osmolality of the solution

What would Ouabain drug due to cells?

Ouabain inhibits Na+/K+ ATPase pump, continued passive leakage of ions through the cell membrane causes intracellular osmolarity to increase and water to enter the cell causing cell swelling.

Endogenous Acid Production

Our body can also regulate acid production in response to changes in our acid-base status. This is a less common mechanism for control but adds another point where we can alter the pH of our body. Endogenous acid production via lactic acidosis and ketoacidosis is used when there is more base in the pH (alkalosis) or there is less acid in the blood (alkalosis).

Phase 3 of cardiac cycle

Outflow (Ejection) phase: Rapid ejection Reduced ejection Semilunar valves now open because left ventricle reaches pressure greater than Aorta. Marks the rest of ventricular systole (2/3). The end of this phase (ventricular systole) is = ESV- the small volume of blood remaining after the end of systole (contraction).

Normal nephron concentrations.

Ovals - osmolality Boxes - relative amount of H2O Solid arrow- active transport Dashed arrow - passive transport.

Menstrual cycle overview

Ovarian cycle: - Follicular phase (days 0-14) - Luteal phase (days 14-28) Endometrial cycle: - Menses (days 0-4) - Proliferative phase (days 4-14) - Secretory phase (days 14-28)

Ovarian follicles

Ovarian follicles are basic unit of female reproductive biology Roughly spherical aggregations of cells found in the ovary - Granulosa cells - Theca cells - Single oocyte (ovum or egg) Grow and develop, culminating in ovulation of usually a single competent oocyte. *Folliculogenesis*: Progression of primordial follicles towards large preovulatory follicles.

Which gastric gland secretes HCl, Intrinsic Factor, and Pepsinogen? Which gastric gland secretes gastrin and mucus?

Oxyntic gland: secretes HCl, Intrinsic Factor, and Pepsinogen. Pyloric gland: secretes gastrin and mucus.

Changes in P50

P50 is the partial pressure of O2 when hemoglobin is 50% saturated. A right shift of P50 means the hemoglobin can unload at higher partial pressures. This is important during exercise to unload O2 to tissues/muscles. - Caused by *Increase in temperature, DPG, PCO2, and a decrease in pH* A left shift of P50 means the hemoglobin loads at lower partial pressures?

Blood Flow in the lungs (Perfusion)

PA = alveolar pressure Pa = arteriole pressure Pv = venous pressure

Neural excitation of smooth muscle

PNS or SNS: - Nerve varicosities instead of neuromuscular junction. - receptors for neurotransmitters located throughout SMC membrane. - ACh, Epi/NEpi, nitric oxide

Location of PNS and SNS

PNS: Carniosacral- cranial nerves and sacral of spine ANS: T1-L2

Which part of the pancreas is responsible for secreting pancreatic juices and electrolytes?

Pancreatic duct cells Net result is secreting HCO3- into the lumen and H+ into the plasma. The cystic fibrosis transmembrane conductance regulator is responsible for the antiport of HCO3- into lumen and Cl- into cell which then exits via Cl- channel to continue the cycle.

What keeps the valves from flowing backwards?

Papillary muscles and Chordae tendineae. Chordae tendinae attached to valve and papillary muscle. Papillary muscle attached to heart and chordae tendinae. If rupture in these muscles then backflow of blood.

Parasympathetic ganglia vs sympathetic ganglia

Parasympathetic ganglia: longer preganglion and shorter post ganglion. Localized influence, innervate in a 1:1 ratio of preganglionic fibers to ganglionic cells. Sympathetic ganglia: shorter preganglion and longer post ganglion. Widespread influence, innervate in a 1:20 ratio. Sympathetic axons go everywhere, superficial and deep.

Salivation autonomic control

Parasympathetic: Increase secretion via acetylcholine->increased enzymatic output and VIP -> copius watery output. Sympathetic: Decreased secretion via Norepinephrine -> increased enzymatic output and vasocontriction, reduced water output. Example of cooperative response with the increased enzymatic output by both.

What are the parasympathetic fibers arising from S2, 3, 4 levels of the spinal cord called

Pelvic splanchnic nerves

Pelvic splanchnic nerves

Pelvic splanchnic nerves: preganglionic parasympathetic nerves. Preganglionic cell bodies in a IML-like area situated in the sacral regions of the spinal cord. WIll only use ventral rami.

Medullary collecting duct

Permeability of the medullary collecting duct to water is controlled by the level of *ADH*. When ADH/AVP is low, the collecting duct is practically impermeable to water. The collecting duct is also capable of secreting hydrogen ions against a large concentration gradient. H+ goes into the duct for secretion. Na+, Cl-, H2O (+ADH), Urea, and HCO3- are reabsorbed into blood.

Phase 1 (part 1) of cardiac cycle

Phase 1: Inflow (ventricular filling) Inflow phase: *Rapid ventricular filling* *Reduced ventricular filling* Atrial contraction (bold terms are highlighted area) During the rapid ventricular filling and reduced ventricular filling that starts at the end of the T wave (end of isovolumetric relaxation which takes up a tiny part of this phase of ventricular diastole) with AV valves open.

Phase 1 (Part 2) of cardiac cycle

Phase 1: Inflow (ventricular filling) Inflow phase: Rapid ventricular filling Reduced ventricular filling *Atrial contraction* (bold terms are highlighted area) During atrial systole (PR interval) the atria contracts, squeezing any remaining blood it has to the ventricles. AV valve still open at this time. EDV is found at the end of this phase (after atrial contraction) and is equal to the total amount of blood in the left ventricle when it is completely full.

Ventricular myocyte action potential

Phase 4: (Resting) Na+, Ca2+ channels closed, open K+ channel. Rectifier channels keep total membrane potential stable at -90mV. Phase 0: (Depolarization) Rapid Na+ influx through open fast Na+ channels. (-90mV to ~+10 mV) Phase 1: (Early repolarization) Transient K+ channels open and K+ efflux returns total membrane potential to 0. Phase 2: (Plateau) Influx of Ca2+ through L-type Ca2+ channels is electrically balanced by K+ efflux through delayed rectifier K+ channels. (Plateau) Phase 3: (Rapid repolarization) Ca2+ channels close but delayed rectifier K+ channels remain open and return total membrane potential to -90 mV. (the big difference is that the opening of the calcium channel caused a plateau which increased the refractory phase, longer absolute refractory phase than skeletal muscle-neuronal)

Average daily turnover of phosphate

Phosphate (Pi or PO4-): - Absorption by the small intestine is constant - Kidneys reabsorb 85% of PO4- in the proximal tubule by active transport.

What is an important intracellular buffer?

Phosphate.

How is PTH regulated?

Plasma Ca2+ regulates PTH secretion. Decrease in plasma Ca2+ levels increases PTH secretion. Calcium-sensing receptor (CaSR) = sensor in chief cells.

When referring ion concentrations from where do we refer to?

Plasma concentrations since we are drawing blood. Unless otherwise stated.

Which hormones interact with the *plasma membrane*/ cell surface receptors? Which hormones interact with the *Intracellular compartment*?

Plasma membrane: Water soluble/lipid insoluble receptors Intracellular: Lipid soluble/water insoluble

Pressure within pleural space

Pleural fluid is typically under negative pressure. It creates a suction on both the chest wall and the lung that holds them next to each other. This negative pressure arises because the lungs are in a vacuum within the thorax, separated from the other tissues. So the chest wall wants to go out (open up) and the lungs want to go in (collapse), the pleural pressure arises from this difference creating a negative pressure fluid in the pleural space. The chest wall and lung stick to one another due to the negative pressure of the pleural fluid. Also, diaphragm sticks to the lungs for the same reason. This pressure is between -3 mmHg to -5 mmHg at rest (negative in respect to atmospheric pressure). During inspiration, the intrapleural pressure gets more negative Elastin fibers are the ones responsible to pull back together after a certain expansion.

Structure of the Glomerulus

Podocytes act as a screen to keep large proteins in the blood but allow passage of small molecules and ions. (Won't allow passage of protein like albumin or RBCs. Will allow passage of electrolytes, water, and glucose. Restrictions: - Size of the fenestrations - Basement membrane: negatively charged. - Slits between podocytes. Type of constraints: - Size - Shape - Charge

Hypothalamus- anterior pituitary anatomy

Portal vein contains capillary bed of the median eminence to the capillary bed of the anterior pituitary. Tropic hormones are hormones released from the anterior pituitary. Neurons -> blood vessels (hypophyseal portal system) -> Anterior pituitary.

Potassium balance

Potassium is largely controlled through secretion in the *collecting duct.* By the end of the Distal Convoluted Tubule nearly 95% of filtered K+ has been reabsorbed. Modifications to K+ in the plasma are carried out by changing the secretion of K+ in the collecting duct. This allows the body to maintain normal K+ levels without compromising excitable cell function.

Where is the pre-ganglionic cell body and post-ganglionic cell body in the sympathetic pathway

Pre-ganglionic- lateral horn of spinal cord- grey matter Post ganglionic - sympathetic ganglion (AKA paraventebral chain ganglion) or prevertebral (preartic ganglia) in pattern 3

Peptide hormone synthesis and secretion

Pre-prohormone (ribosomes) -> Pro-hormone (Rough Endoplasmic reticulum) -> Hormone (golgi) -> Peptide goes into secretory/storage granules (Pep) -> Pep exits via exocytosis into blood stream. *Stimulus: Ca2+ and cAMP* The cleavage of prohormone into active hormone molecules typically takes place during transit through the Golgi, or perhaps, soon after entry into secretory vesicles. Secretory vesicles, therefore, contain not only active hormone but also the excised biologically inactive fragments. When active hormone is released into the blood, a quantitatively similar amount of inactive fragment is also released. In some instances, this forms the basis for an indirect assessment of hormone secretory activity. Other types of processing of peptide hormones that may occur during transit through the Golgi include glycosylation and coupling of subunits

Pattern 3: sympathetics to viscera below the diaphragm

Preganglionic 1. Pass through the anterior root 2. White ramus 3. Courses in the sympathetic chain and leaves without synapsing. Postganglionic 1. Has cell body in preaortic (aka prevertebral) ganglia 2. Distribute to gut on blood vessels Organs in the abdominal-pelvic cavity receive postganglionic sympathetic axons from preaortic ganglia (prevertebral ganglia)

Pattern 2: sympathetics to viscera above the diaphragm

Preganglionic axons: 1. Pass through the anterior root. 2. White ramus. 3. Synapse in the chain ganglia Post ganglionic axons: 1. Has cell body in chain ganglia 2. Leaves via a visceral nerve (e.g. cardiac nerve) *Organs in the head, neck, and thoracic cavity receive sympathetic axons directly from the sympathetic chain.

Pattern 1 of sympathetic pathway: sympathetics to the body wall.

Preganglionic axons: 1. Pass through the anterior root 2. White ramus 3. Synapse in the chain (sympathetic ganglion) Post ganglionic axons: 1. Has cell body in sympathetic chain ganglion (sympathetic ganglion). 2. Gray rami 3. Dorsal and ventral rami Dorsal and Ventral rami at all 31 levels contain sympathetics.

Peptide hormone synthesis

Preprohormone processing: example insulin. - Polypeptides are cleaved by enzymes within secretory vesicle - Multiple peptides released simultaneously.

Intestinal phase stimulatory pathway (1)

Presence of low pH and partially digested foods in duodenum when stomach begins to empty -> intestinal (enteric) gastrin release to blood -> stomach secretory activity.

Aortic pressure

Pressure changes during cardiac cycle: *Pulse pressure (pp)* = systolic pressure - diastolic pressure. Systolic pressure is the pressure coming out of aorta/pressure felt in arteries during contraction. Diastolic pressure is the pressure in arteries during ventricular filling.

Large arteries and pulse pressure

Pressure changes during cardiac cycle: Pulse pressure (PP) = systolic pressure - diastolic pressure It is dependent on the ratio: - SV/arterial compliance Therefore: - Increase SV = increase in pulse pressure - Increase arterial compliance = decrease in pulse pressure. Alter these factor and you change the pulse pressure.

Prostacyclin (Endothelium) pathway

Prevents vasoconstriction Prevents platelet aggregation *Cyclooxygenase (COX)*: Non-steroidal anti-inflammatory drugs (NSAIDs) are frequently COX inhibitors- treat inflammation/pain. - May increase adverse cardiovascular events (non-fatal myocardial infarction, non-fatal stroke, hypertension). (This is because COX is also responsible for making thromboxane) Arachidonic acid + COX (enzyme) -> Prostacyclin (PGI2) -> Adenylate cyclase (which converts ATP to cAMP) -> cAMP -> Vasodilation.

Erection of the penis or clitoris is primarily under the control of?

Primarily under parasympathetic control

Ventricular volumes

Primary function of the heart: Impart energy to blood in order to generate and sustain an arterial blood pressure necessary to provide adequate perfusion of organs. End diastolic volume (EDV)= Volume of blood in the ventricle at the end of diastole. End systolic volume (ESV)= Volume of blood in the ventricle at end of systole. Stroke volume (SV) = volume of blood ejected from heart each cycle.

Changes to K+ in acidosis, volume expansion, high water intake, and volume contraction

Principle Cell intracellular K+ levels can impact K+ secretion. Acidosis, volume expansion, high water intake, and volume contraction all have minimal net effect on K+ excretion due to opposing forces. Only thing that really increases K+ secretion in a healthy person is increased dietary K+.

Glucagon physiological effects

Promotes mobilization of stored nutrients - Glycogenolysis - Gluconeogenesis - Lipolysis - Ketogenesis Ketones are sources of fuel for muscle/heart cells during times of starvation.

Endosome and pH

Proton pumps embedded in endosome membrane ensure that, like the lysosome, the endosome maintains an acidic luminal pH. Most endocytic receptors bind their ligands tightly at neutral pH but release them rapidly at pH values <6.0

What is responsible for reabsorbing all of the filtered glucose and amino acids, reabsorbing the largest fraction of the filtered Na+, K+, Ca2+, Cl−, HCO3−, and water, and secreting various organic anions and organic cations.

Proximal tubule (mostly proximal convoluted tubule)

Na+/H+ exchanger

Proximal tubule cells: Na+ and H+ are exchanged at the luminal/apical basement pushing H+ into the lumen and Na+ into the tubular cell. H+ combines with HCO3- in the lumen and makes H2CO3 which dissociates via carbonic anhydrase into H2O and CO2. The CO2 made then rapidly moves into the tubular cell where it recombines with H2O to form H2CO3 which then dissociates to H+ and HCO3-. H+ then allows the cycle to repeat. The HCO3- is then transported out of the tubular cell via Na+/HCO3- symporter. This cycle allows to draw Na+ out of the lumen and then out of the tubular cell w/ HCO3- to ultimately end up in the peritubular capillaries.

Potassium transport

Proximal tubule: - K+ is only minimally secreted into the lumen. - K+ can exit the lumen through intercellular space alone or with H2O. - K+ mainly enters the cell from interstitial space via Na/K pump. - K+ can exit the cell to interstitial space via K+ channel or via symport with Cl. Thick ascending limb: - K+ can enter cell from lumen via NKCC. - K+ can exit lumen along with Na+ through the intercellular space. - K+ can enter cell from interstitial via Na/K pump. - K+ can leave cell to interstitial or lumen via K+ channel. Cortical collecting tubule principal cell (controls K+ secretion). - Main site of K+ secretion into the lumen via symport with Cl- or by the K+ channel. (Cl- then exits via intercellular route) - Na+ enters cell via ENaC channel - K+ enters cell from interstitial mainly by Na/K pump. - K+ can also leave cell to interstitial via K+ channel. Cortical collecting tubule (CCT) alpha intercalated cell (controls acid base balance) - K+/H+ antiport with H+ going into lumen and K+ going into cell, ATP driven. - H+ going into lumen from cell through ATP driven channel. - K+ mainly enters cell from interstitial via Na/K pump. - K+ leaves cell to interstitial via K+ channel. - HCO3- exits cell to interstitial via antiport with Cl- going into cell (Cl- can then leave via Cl- channel back out into interstitial space).

Where do the pseudo-unipolar sensory neuron, somatic multipolar motor neuron, and autonomic multipolar motor neurons of PNS sit within the spinal nerve?

Pseudo-unipolar sensory neuron: sits in posterior root of spinal nerve. Somatic and autonomic multipolar motor neurons sit in the Anterior roots of spinal nerve

Lung and chest wall receptors

Pulmonary Stretch Receptors - slowly adapting stretch receptors in the sensory terminals of myelinated afferent fibers within the smooth muscle layers of the airway. Firing leads to the excitation of the inspiration blocking switch. Irritant Receptors - rapid adapting receptors in the sensory terminals of myelinated afferent fibers found in large conducting airways. Respond to noxious stimulus such as dust, smoke, or touch. J Receptors - Juxtapulmonary capillary receptors (C-fiber endings). Situated near the alveoli and bronchial circulation, they are stimulated by lung injury, large inflation, acute pulmonary vascular congestion, and certain chemicals. They stimulate rapid shallow breathing, bronchoconstriction, and cardiovascular depression.

Vessel changes: recruitment and distention

Pulmonary arterioles and capillaries actually have a higher than normal compliance Recruitment tends to be the driving force and it's related to ventilation. Increase in cardiac output -> pulmonary arterial -> recruitment of more capillaries opening -> distention of the capillaries (if needed)

Pulmonary circulation

Pulmonary blood flow is equal to systemic blood flow. 5 L/min R = Delta P/Q Pulmonary resistance is really low due to the extensive amount of branching (more so than systemic circulation) that occurs almost instantly after the blood leaves the right ventricle. Despite the same cardiac output (5 L/min), Pulmonary resistant is far lower than systemic resistance from both the right and left ventricles. The lower resistance means that the right ventricular pressure is typically 1/5 of the left ventricular pressure and the pulmonary arterial pressure is 1/5 aortic pressure. You don't want a lot of blood pressure in the lungs (fine in the muscles) because you don't have rigid tissue and the blood will almost go into the air space and you get pulmonary edema.

Spirometry

Pulmonary function test. Determines lung volumes and the flow of air during inhalation and exhalation. Patient takes a deep breath and blows as hard as possible into a tube. Machine records the results of the spirometry test. Basically says how much air do you really breath in and out.

Pulmonary surfactant and babies

Pulmonary surfactant is especially important in infants because of their even smaller alveoli sizes. Premature babies must be given bovine surfactant until they can produce their own. Surfactant is produced by the Alveolar type II cells.

Surfactant

Pulmonary surfactant is secreted to line the alveolar sacs. This special solution allows for the reduction of surface tension at low volumes by altering the fluid-gas interface between inspired air and lung tissues. This prevents alveolar sacs from collapsing and the tissue of the sacs from sticking together.

Curve of lung volume vs pulmonary vascular resistance

Pulmonary vascular resistance is lowest at FRC. At high lung volumes, extra-alveolar vessels are actually distended because of the lower pleural pressure. However, alveolar vessels are compressed, causing a rise in pulmonary vascular resistance. At low lung volumes, alveolar vessels are distended, but the extra-alveolar vessels are compressed from the rise in pleural pressure, which results in a rise in pulmonary vascular resistance.

Pulmonary pressures

Pulmonary wedge pressure is an indirect measure of left arterial pressure. There's not really a pulse pressure in the lungs due to dampening from all the branching

In what phase of the cardiac cycle is both the mitral valve open and ventricular pressure falling?

Rapid ventricular filling (Before blood starts pooling into ventricle)

Cardiac cycle outline

Rapid ventricular filling -> reduced ventricular filling -> atrial contraction -> isovolumetric contraction -> rapid ejection -> reduced ejection -> isovolumetric relaxation.

Summary of proximal tubule transporters, absorption and secretion.

Reabsorption: Na+, H2O, and HCO3-. Secretion: organic acids (bile acids, oxalate, urate), Para-aminohippuric Acid (PAH), drugs, hydrogen. Basolateral membrane: Na+/K+- ATPase. Luminal membrane: Sympoters- Na+/glucose, Na+/amino acid, Na+/phosphate. Antiporter- Na+/H+ *Everything but Mg2+ is reabsorbed in the proximal tubule*

Location of transporters in proximal tubule cells

Reabsorption: sodium, water, bicarbonate Basolateral membrane: Na+/K+-ATPase Luminal/apical membrane: Co-transport: Na+/glucose, Na+/amino acid, Na+/phosphate Exchange: Na+/H+ exchanger Reabsorbs ~60-65% of necessary nutrients.

Extracellular: Catalytic (Enzyme-Linked)

Receptor Tyrosine Kinases (RTK) are one of the most common catalytic receptors. Transmembrane proteins 1. Ligand bind to extracellular domain of receptor protein. 2. Two receptors associate (dimerize) and phosphorylate each other (autophosphorylation) 3. Response protein bind to phosphotyrosine on receptor. Receptor can phosphorylate other response proteins. Example: Insulin receptor.

Electrocardiogram

Records electrical activity of the heart. EKG records the spread of action potential throughout the heart during depolarization and repolarizaiton (sum of all cells). Compares difference in voltages at two different electrodes on body surface. Tells us: -Electrical activity of the heart -Heart rate -Conduction abnormalities -Presence of myocardial ischemia, infarct, and cell death. P-Wave: Depolarization of atria in response to SA node triggering PR interval: Delay of AV node to allow filling of ventricles QRS Complex: Depolarization of ventricles, triggers main pumping contractions. ST segment: Beginning of ventricle repolarization, should be flat. T-wave: Ventricular repolarization.

Muscle recruitment and motor units

Recruitment- increasing number of individual contracting fibers to generate greater force. Motor unit = motor neuron + muscle fiber. Motor units vary in number of fibers stimulated and size of fibers within each unit. Precise movements created by small motor units Gross movements by large motor units

Actin-binding proteins

Regulate actin-myosin interactions

Main goal of the kidneys

Regulation and elimination is ultimately aimed at producing urine. Urine formation: Regulate osmotic pressure, regulate extracellular fluid levels (Na+), eliminate waste products, and remove drugs.

GFR: Renal Filtration Pressures

Relationship between selective changes in the resistance of either the afferent arteriole or the efferent arteriole on RBF and GFR. Constriction of either the afferent or efferent arteriole increases resistance, and according to Equation 32-11 (Q = ΔP/R), an increase in resistance (R) decreases flow (Q) (i.e., RBF). Dilation of either the afferent or efferent arteriole increases flow (i.e., RBF). Constriction of the afferent arteriole (A) decreases PGC because less of the arterial pressure is transmitted to the glomerulus, thereby reducing GFR. In contrast, constriction of the efferent arteriole (B) elevates PGC and thus increases GFR. Dilation of the efferent arteriole (C) decreases PGC and thus decreases GFR. Dilation of the afferent arteriole (D) increases PGC because more of the arterial pressure is transmitted to the glomerulus, thereby increasing GFR.

ADH and homeostasis

Release of ADH triggered when osmoreceptor cells in the hypothalamus detect an increase in the osmolarity of the blood. ADH acts on the collecting ducts of the kidneys to increase water retention. ADH (antidiuretic hormone) AKA Arginine Vasopressin (AVP).

Glucagon regulation

Release of glucagon: *Stimuli*: - Hypoglycemia/fasting - B-adrenergic agonists and acetylcholine - Exercise *Inhibitors* - Hyperglycemia (glucose) - Somatostatin - Insulin

PTH increases Renal 1a-hydroxylase activity

Renal 1-hydroxylase converts 25(OH)D3 (calcifediol) -> 1,25 (OH)2D3 (activated vitamin D/calcitriol) *Actions of calcitriol will increase plasma Ca2+ and phosphate*

Renin - Angiotensin II

Renin release is controlled by changes in Na+ concentration and blood pressure. *Decrease in Na+ concentration in the ascending tubule (sensed by macula densa) or decrease in blood pressure (sensed by the granular/juxtaglomerular baroreceptors) both cause renin release*. The impact is the creation of Angiotensin II which is a potent vasoconstrictor of arterioles. Angiotensin II preferentially constricts the *efferent arteriole*. In low pressure situations there's a reduction in GFR. Constriction of the efferent arteriole reinstates normal GFR levels. However, Renal Blood Flow is reduced due to the constriction and peritubular and Vasa Recta pressure drops causing an increase in Na+ and H2O reabsorption in the nephron.

Secondary active transport

Require ATP indirectly. Example: NaK+ pump is primary active transport using ATP to establish a gradient with more Na+ on the outside. Therefore, the Na+ coming back into the cell down its concentration gradient would be secondary active transport. Secondary active transport involves co-transport of molecules, like pairing up a molecule with Na+ to go down its concentration gradient. Use symport: 2 solutes in same direction. and Antiport: 2 solutes in opposite directions.

Which lung volumes cannot be determined directly by Spirometry?

Residual volume (RV), Functional residual capacity (FRC), and Total lung volume (TLC).

Arterioles

Resistance vessels (arteries main control over the blood pressure, not veins) - Regulate vessel diameter - High resistance to blood flow - Change Q *Metarterioles*- connect arterioles and capillaries *Precapillary Sphincters*- smooth muscle cells which regulate flow into capillary beds. Thoroughfare channel- Same as metarterioles but the venous side of it. Precapillary sphincters are basically smooth muscles lying at the beginning of the capillaries (at junction between metarterterioles and capillaries) that can constrict and stop blood flow to a certain region. Important in regulating blood flow. Instead, the blood would just collect in the thoroughfare channel and then leave through the venule.

pH, CO2, and HCO3- in every acid-base disturbance.

Respiratory acidosis: Increased CO2 Increased H+ Increased HCO3- Compensatory response (metabolic): Kidneys increase H+ excretion. Metabolic acidosis: Decreased HCO3- Increased H+ Decreased CO2 Compensatory response (respiratory and metabolic): Alveolar hyperventilation; kidneys increase H+ excretion. Respiratory alkalosis: Decreased CO2 Decreased H+ Decreased HCO3- Compensatory response (metabolic): Kidneys increase HCO3- excretion. Metabolic alkalosis: Increased HCO3- Decreased H+ Increase CO2 Compensatory response (respiratory and metabolic): Alveolar hypoventilation; kidney increase HCO3- excretion. Respiratory: Due to changes in Pco2. Metabolic: Due to changes in HCO3-.

CO2 and H+ in relation to ventilation rate

Respiratory and Ventilation rates are largely driven by chemoreceptor activation. H+ and CO2 are considered the main drivers of respiration, but it is the PCO2 level that mainly drives respiration because H+ only modestly changes total pH (log scale) CO2 causes the creation of H+ near the chemoreceptors (or even within) and this leads to activation of these receptors.

Vascular resistance.

Rsistance to flow depends on: r = radius inside the tube L = length of tube n = fluid viscosity (RBCs) Poiseuille's Law: R = (8Ln)/pi(r)^4 So the radius has the biggest effect on resistance. Increasing the radius by 2 means decreasing resistance by the 4th power and vice versa. (Resistance is proportional to the reciprocal of the 4th power of radius.) Increasing length or fluid viscosity is proportional with increasing resistance.

Heart sounds: turbulent flow and murmurs

S1 (1st)- Closing of the AV valves causes turbulent flow (lub) S2 (2nd)- Closing of the aortic, pulmonary valves (dub) Murmur: Heart murmurs are sounds during your heartbeat cycle — such as whooshing or swishing — made by turbulent blood in or near your heart. Diastolic murmur: Heard during diastole Systolic murmur: Heard during systole. (Won't be tested on S3)

What has the fastest firing rate and is usually the dominant pacemaker of the heart?

SA node

ANS regulation of vasculature

SNS: Vasocontriction: Increase vascular resistance Increase arterial pressure (Increases Total Peripheral Resistance (TPR)) Veno constriction: Decrease venous compliance Increase central venous pressure (CVP) (increases preload/EDV = increase SV = Increase CO) (MAP = CO x TPR, vasocontriction increases TPR, venoconstriction increases CO. Sympathetic nervous system increases MAP.) PNS: Little innervation of blood vessels ---------------------------------- Increase sympathetic -> norepinephrine which binds to adrenergic receptor A1 (mainly) -> Gq -> increase IP3 -> releases calcium from sarcoplasmic reticulum -> contraction. A1 is primary mechanism for vasocontraction. (norepinephrine activates a-adrenergic receptors a1 and a2, a1 is primary mechanism)

Goldman-Hodgkin-Katz voltage equation.

Same as Nernst equation but takes into account more than one ion ( K+, Na+, Cl-, and Ca2+).

APs in pacemakers vs ventricle myocytes

Same refractory period (picture very important)

Calcitonin

Secreted by C-cells of the *thyroid gland* follicle Peptide hormone Release stimulated by hypercalcemia Not involved in small changes in Ca2+ regulation (only involved with large changes in Ca2+)

What do salivary gland secrete, what is it secreted by, action, and stimulus for secretion?

Secretion: Saliva Secreted by: Parotid (serous), sublingual (mucous), and submandibular glands (mixed). Action: - begin to digest CHO via amylase - Protect mouth and esophagus Stimulus for secretion: - Parasympathetic and sympathetic stimulation

Properties of cardiac ion channels

Selectivity - Specific for ions Voltage-sensitive gating - Specific range of membrane potential required for open configuration Time-dependence - Some channels (fast Na+) close after fixed amount of time.

Summation of resistance: Parallel vs Series

Series: Rt = R1 + R2 + R3 + Rn Parallel: 1/Rt = 1/R1 + 1/R2 + 1/R3 + 1/Rn Adding similar-sized vessels in parallel reduces resistance

Peptide hormone signaling

Signals are often more transient (cytoplasmic response) May alter gene expression.

Gastroesophageal reflux disease (GI Motility: Pathophysiology). Signs/symptoms, causes, verification tests, and treatment

Signs/symptoms: - Heartburn - Regurgitation - Dysphagia - Sore throat Causes: - Respiratory pressure changes - Hernia of diaphragm - Bacterial infection - Bulemia Verification Tests: - Upper GI endoscopy - Manometry Treatment: - Diet/Exercise changes - Pharmaceutical - Surgical

Which type of smooth muscle found in the GI contract spontaneously in the absence of neural or endocrine influence and contract in response to stretch?

Single-unit smooth muscle Also contracts in presence of neural or endocrine influence

Smooth muscle units characteristics

Single-unit: Single unit smooth muscle cells are connected by gap junctions and the cells contract as a single unit. Multi-unit: multi unit smooth muscle cells are not electrically linked therefore each cell must be stimulated independently.

Cardiac conduction system

Sinoatrial (SA) node fires with signal going to left atrium via Bachman's bundle and ->anterior/middle/posterior internodal -> Atrioventricular (AV) node -> Atrioventricular (AV) bundle (bundle of HIS) -> Right and left bundle branches -> Purkinje fibers

Where is absorption greatest in GI tract?

Small intestine due to the large surface area.

Cardiac/skeletal vs smooth muscle fiber

Smooth muscle fiber: No T tubules Less SR than skeletal muscle No troponin Dense bodies/intermediate filaments. Different arrangement of actin/myosin (no sarcomere) Contracts in all directions.

Countercurrent Multiplication.

So lets say 300 mOsm fluid enters the descending limb and matches the interstitial space osmolarity (at 300). Once it starts going into the non-permeable ascending limb, NaCl moves into the interstitial space increasing its osmolarity. As more 300 mOsm fluid comes in, the water goes out into the interstitial fluid because the interstitial fluid is more concentrated due to the NaCl given off by the ascending limb. As this process continues and water starts going out, the top of the descending limb (with new 300 mOsm fluid coming in) has a lower concentration than the bottom of the loop. This is what makes the tops of the Loop of Henle have a lower osmolarity than the bottom of the Loop of Henle, and in turn the top of the interstitial space lower osmolarity than the bottom. Teachers explanation: Fluid enters the loop in A. -In B, the opening of channels in a water impermeable (ascending limb) section causes a gradient of 200 mOsm to develop. Water can then exit the water permeable (descending limb) section. This creates different concentrations for the static fluid. -In C, more fluid enters the loop and pushes the existing fluid a bit further along. The new fluid has the original 300 mOsm osmolality. -In D, the fluid that enters the next sections again tries to establish a 200 mOsm gradient, however, the new fluid in the ascending limb is now the 400 mOsm (in part C) solution that was pushed forward, and this creates a higher osmolality in their interstitial space. Water again exits from the water permeable portion (Descending limb) and solutes exit through channels in the water impermeable section (ascending limb). -In E, we again introduce more water into the loop at 300 mOsm and push the fluid further again..... We keep doing the same thing over and over until we have a really high osmolality at the bottom of the loop and a really low osmolality at the top of the loop. This is how countercurrent multiplication works. The ascending and descending limbs working together both have an impact and the continual movement of fluid is known as countercurrent multiplication.

A 44-year-old moderately dehydrated male was admitted with a two day history of acute severe diarrhea. Electrolyte results (mEq/L) Na+ 134 K+ 2.9 Cl- 108 (normal is 95-105) HCO3- 16 ABG pH: 7.31 PCO2: 33 mmHg PO2: 93 mmHg What is the acid base disorder?

So pH is low so its acidosis. PCO2 is low and so is HCO3-. This has to be metabolic acidosis. Now lets check anion gap. 134 - 108 - 16 = 10 (8-14 normal) so this is non-anion gap metabolic acidosis. The clinical presentation of the patient having acute severe diarrhea can be a clue.

A 22-year-old female with type I DM, presents to the emergency department with a 1 day history of nausea, vomiting, polyuria, polydypsia and vague abdominal pain. Physical Exam noted for deep sighing breathing, orthostatic hypotension, and dry mucous membranes. Electrolyte results (mEq/L) Na+ 132 K+ 6.0 Cl- 93 HCO3- 11 UA: pH 5 ABG: pH: 7.27 PCO2: 23 mm Hg What is the acid-base disorder?

So pH is very low so it is acidosis. PCO2 and HCO3- are both very low so this is metabolic acidosis. Anion gap is 132 - 93 - 11 = 28 so this is an Anion gap metabolic acidosis (ketoacidosis).

If we eliminated permeability to sodium, what would happen to membrane potential?

Sodium equilibrium potential is positive so then the membrane potential will get more negative without sodium there to tug the other way.

Sodium balance

Sodium is the most filtered substance and it is nearly completely reabsorbed through the kidneys.

Osmolarity

Solute particles per liter. Osmolarity intracellulary (290) = extracellulary (290)

What pathway would decrease insulin secretion?

Somatostatin and a-adrenergic agonist Bind to Gai and inhibit cAMP release. Inhibitors: 1. Somatostatin 2. a-adrenergic agonists.

Somatostatin source, target, and action.

Source: D cells of the stomach and duodenum, δ (delta) cells of pancreatic islets Target: 1. Stomach 2. Intestine 3. Pancreas 4. Liver Action: 1. Decrease gastrin release 2. Increase fluid absorption/ decrease secretion; increase smooth muscle contraction. 3. Decrease secretions 4. Decrease bile flow

Vasoactive intestinal peptide (VIP) source, target, and action.

Source: ENS neurons Target: 1. Small intestine 2. Pancreas Action: 1. Increase smooth muscle relaxation; increase secretion by small intestine. 2. Increase secretion by pancreas

Motilin source, target, and action.

Source: Endocrine cells in the upper GI tract (antrum and duodenum). Target: Esophageal sphincter, stomach, and duodenum. Action: Increase smooth-muscle contraction.

Gastrin source, target, and action.

Source: G cells, antrum of the stomach. Target: Parietal cells in the body of the stomach. Action: Increase H+ secretion. Stimulates stomach motility.

Cholecystokinin (CCK) source, target, and action.

Source: I cells in the duodenum and jejunum and neurons in the ileum and colon. Target: 1. Pancreas 2. Gallbladder Action: 1. Increase enzyme secretion. 2. Increase contraction. (gallbladder) inhibits stomach emptying.

Gastric inhibitory peptide (GIP) source, target, and action.

Source: K cells in the duodenum and jejunum. Target: Pancreas Action: Exocrine: Decrease fluid absorption Endocrine: Increase insulin release

Secretin source, target, and action.

Source: S cells of the small intestine. Target: 1. Pancreas 2. Stomach Action: 1. Increase HCO3- and fluid secretion by pancreatic ducts. 2. Decreases gastric-acid secretion; inhibits motility.

Gastrin releasing peptide (GRP) source, target, and action.

Source: Vagal nerve endings Target: G cells in the antrum of the stomach. Action: Increase gastrin release. Increases stomach motility.

Iodine sources and metabolism

Sources: bread, milk, iodized salt To maintain "normal" (euthyroid) function, we must ingest approximately 1 mg iodide per week. We actually ingest musch more (about 3 mg/week). Of the iodide ingested, 30% goes to the thyroid, 50-70% to the kidneys (excretion in urine), remainder to salivary glands/stomach. *Don't need to know percentages*

Transport mechanisms and location within tubule

Specific ions and molecules can use: - Passive transport - Primary active transport - Secondary active transport Transport mechanisms depending on the location within the tubule.

GI Motility: Sphincters

Sphincters prevent movement between specialized compartments. - Lower esophageal sphincter - Pyloric sphincter - Sphincter of Oddi - ileocolic sphincter (ileococcal valve) - Internal anal sphincter

Which circulation connects the GI tract, spleen, pancreas and liver?

Splanchnic circulation

Fibers that enter and leave the sympathetic chain without synapsing form structures called?

Splanchnic nerves

What do prevertebral (preaortic) synapse on?

Splanchnic nerves (greater, lesser, least) and lumbar splanchnic nerves Splanchnic nerves are preganglionic*

Capillary fluid exchange governed by what?

Starling forces = mixture of various pressures and filtration constant Fluid flow (Qf) = Kf ((Pc-Pi)-(Nc-Ni)) Qf = fluid movement Kf = filtration constant (measure of surface area + intrinsic permeability) Pc = capillary hydrostatic pressure Pi = interstitial fluid hydrostatic pressure πc = plasma oncotic pressure πi = interstitial fluid oncotic pressure.

Peripheral vascular system veins and arteries.

Start with elastic artery -> muscular artery -> arteriole -> continuous capillary -> fenestrated capillary -> venule -> medium-sized vein -> Large vein.

D5W (5% dextrose in water) solution effect

Starts out Isotonic then turns hypotonic once the dextrose metabolized by body. 5000 mg/dl glucose, no Na+ or Cl- Change in ECF: 333 mL Change in ICF: 667 mL Dextrose is slowly absorbed by cells making it's effect wear off over time, it also has the added benefit of adding energy to the system that can be used to accomplish the movement of solutes and water necessary to expand volume. Does not replace electrolytes. Isotonic solution used to increase the EXTRACELLULAR fluid volume due to blood loss, surgery, dehydration, fluid loss that has been loss extracellularly.

Steroid hormones characteristics

Steroid hormones (thyroxine, cortisol, testosterone, estrogen) can pass directly through the plasma membrane. Directly change transcription by binding to steroid receptors in the nucleus.

Oxytocin: stimulus for release and actions?

Stimulus for release: - Stretch activated sensory neurons in the cervix. - Breastfeeding stimulated sensory neurons Actions: - Contraction of uterine smooth muscle during labor. - Contraction of myoepithelial cells in breast mild leading to milk ejection. *POSITIVE FEEDBACK* 1. Baby moves deeper into mother's birth canal. 2. Cervix of uterus is stretched. 3. Nerve impulses are sent to hypothalamus 4. Hypothalamus sends impulses to posterior pituitary where oxytocin is stored. 5. Posterior pituitary releases oxytocin to blood; oxytocin travels to uterine muscle 6. Uterus responds to oxytocin by contracting more vigorously 7. At birth, stretching of cervix lessens and positive feedback cycle is broken.

Gastric phase pancreatic secretions: stimulus, mediators, and response

Stimulus: 1. Protein in food 2. Gastric distention Mediators: 1. Gastrin 2. ACh release by vagal stimulation Response: 1. Increased secretion, with greater effect on enzyme output. 2. Increased secretion, with greater effect on enzyme output.

Intestinal phase pancreatic secretions: stimulus, mediators, and response

Stimulus: 1. Acid in chyme 2. Long-chain fatty acids 3. Amino acids and peptides Mediators: 1. Secretin 2. CCK and vagovagal reflex 3. CCK and vagovagal reflex Response: 1. Increased H2O and HCO3- secretion 2. Increased secretion, with greater effect on enzyme output. 3. Increased secretion, with greater effect on enzyme output.

Cephalic phase pancreatic secretions: stimulus, mediators, and response

Stimulus: Thought of food, smell, taste chewing, and swallowing. Mediators: Release of ACh and gastrin by vagal stimulation. Response: Increased secretion with greater effect on enzyme output.

Where are single-unit smooth muscle typically found?

Stomach and intestines

How does change in length cause changes in tension? (3)

Stretching of myocytes results in: 1. Increased sensitivity of troponin for binding Ca2+ 2. Increased Ca2+ release from SR. 3. Decreased spacing between thin/thick filaments. More cross-bridges = more force

Cardiac function measurements

Stroke Volume (SV): volume of blood ejected from heart each cycle. SV = EDV - ESV. Cardiac Output (CO): amount of blood ejected from heart each minute. CO = SV x HR Ejection Fraction (EF): Fraction of EDV that is ejected during systole. EF = (EDV-ESV)/EDV Pulse Pressure (PP): Pressure changes during cardiac cycle. PP = Systolic BP - Diastolic BP

Effects of increased afterload on SV?

Stroke volume goes down while ESV goes up (volume in left ventricle after contraction). On the next cardiac cycle- heart again fills but now with the extra blood left over from last cycle means increase in EDV on this subsequent cycle which means increase in preload and activating Frank-Starling Mechanism, increase in SV also on this subsequent cycle.

GI Motility: Stomach: Antral Pump

Strong Peristaltic Phasic Contractions: - Gastric ICC's (interstitial cell of Cajal (pacemaker)) midway begin action potentials. - Leading contraction following by a trailing contraction. - ENS modifies plateau of action potential. - Pyloric contraction initiates retropulsion. Gastin (hormone) will increase activity of the antral pump.

Surface tension and lung compliance

Surface tension has to be overcome to inflate the lung. Therefore, surface tension decreases compliance. Surface tension is air-liquid interface in the alveolar sacs. Most work done in breathing air is work done fighting surface tension. A saline filled lung has increased compliance because you only have a liquid interface and there's no surface tension. Easier for the lung to expand when it doesn't have to fight against surface tension. Laplace's Law: P = 2T/r Where P is pressure, T is tension, and r is radius. A larger alveolar sac has less pressure as a result of surface tension because the larger the radius the smaller the pressure. Small airways could become more susceptible to collapse because of their smaller radius and effect of surface tension.

What are varicosities?

Swelled regions within the nerve axon which is the site of neurotransmitter release.

Shallow water blackout

Swimmer hyperventilates before diving -> decrease PaCO2 -> decrease respiration -> Increase O2 utilization by excercising tissue -> Decrease PaCO2 (hypoxia signal blunted by hypocapnia (reduced CO2 in blood)) -> Brain becomes hypoxic -> Blackout. The body uses CO2 as the main breathing factor, by decreasing CO2 when hyperventilating, the body thinks its good for a bit longer because the CO2 hasn't built up as much.

What innervates the sweat glands?

Sympathetic cholinergic fibers (use ach) causes contraction

How is adrenal medulla special in terms of neurotransmitters

Sympathetic nervous system -> splanchnic nerve (preganglionic) which synapses with Chromaffin cell (modified postganglionic sympathetic neuron in adrenal medulla) which releases epinephrine (instead of NE) This is pattern #3

ANS regulation of the heart SNS and PNS

Sympathetic nervous system: Increase chronotropy: Increase heart rate Increase slope of pacemaker potential (SA node) Increase inotropy: Increase contractility Elevation in intracellular Ca2+ Increase dromotropy: Increase conduction velocity (AV node) Increase lusitropy: Increase relaxation rate ======================== Parasympathetic nervous system: Decrease chronotropy: Decrease heart rate Decrease slope of pacemaker potential (SA node) Decrease inotropy: Decrease contractility Primarily in atria (PNS has one major effect- decrease in chronotropy. Decrease in inotropy mainly just affects the atria, not as big of a factor)

Autonomic regulation of the cardiovascular system.

Sympathetic nervous system: - Active under emergency/stressful situations, prepares for energy expenditure - Exercise, emotional excitement, pathological conditions. - Distributed to all parts of the heart. - Receptors on vascular smooth muscle. - Increases heart rate, contractility, blood vessel contraction, renin secretion by kidneys. (blood vessels, heart (all parts), adrenal medulla) Parasympathetic nervous system: - Active under restful conditions - Predominates during rest, sleep, emotional tranquility. - Primarily at the SA and AV nodes - Decreases heart rate. (salivary glands, mainly SA, AV nodes of heart, blood vessels of external genitalia only, GI system)

Neurotransmitters and receptors used by ANS

Sympathetic: Preganglionic = Ach which binds to nicotinic receptors. Postganglionic = Norepinephrin which binds to alpha or beta adrenergic receptors. Parasympathetic: Preganglionic = Ach which binds to nicotinic receptors. Postganglionic = Ach which binds to muscarinic receptors.

Thyroid hormone binding proteins

T3 and T4 secreted from the thyroid circulate in the blood almost entirely bound to proteins. 70% of circulatory T3 and T4 is bound to *Thyroxine binding globulin* (TBG) Only 0.03% - 0.3 % of total plasma T3 and T4 exists in free state Binding proteins maintain a large storage pool of circulating T3 and T4. - Prevent excretion of T3 and T4 in urine and metabolism by liver. *T3 is most biologically active thyroid hormone, but T4 has longer half-life*

Thyroid hormone metabolism at target cells

T3-TBG (thyroxine binding globulin) complex and T4-TBG complex separate and T3, T4 diffuse across plasma membrane of target cell. Or they can get in via carrier mediated transporter. Inside, T4 can deiodinase to rT3 or T3. rT3 deiodinases to T2 which deiodinases to T1. T3 can deiodinase to T2 then T1 or go to the nucleus and bind to the thyroid hormone receptor in the nucleus. *T3 is the physiologically active form* Target cell: Liver, kidney, muscle, etc. Iodide is either: - Excreted in urine - Recycled to the thyroid T1 and T2 are excreted in urine.

Total body water, extracellular, and intracellular fluid in liters of a standard 70 kg person

TBW: 42 L Intracellular: 28 L Extracellular: 14 L

TRH (thyrotropin releasing hormone)

TRH -> TSH (peptide) -> T3 and T4 (Amino acid-derived)) TRH -> binds to G protein on the Thyrotrophic cell -> activates Gaq -> PLC -> PI3 and DAG -> -> -> TSH release through secretory granules

Hypothalamic- Pituitary- Thyroid Axis

TRH is tonically relealsed from the hypothalamus. Other stimuli: cold exposure Thyroid hormone: Increase metabolic rate and Increase heat production.

TSH effects on thyroid hormone synthesis (4)

TSH: 1. Increases iodine uptake (NIS) 2. Activates thyroid peroxidase 3. Increases thyroglobulin synthesis 4. Increases *secretion* of T3/T4, NOT storage. - Enhanced proteolysis of thyroglobulin

Calcitonin effects

Targets kidney and bone. Kidney: Decrease phosphate reabsorption and decrease calcium reabsorption. Bone: Decreased Ca2+ resorption.

M1 muscarinic acetylcholine receptors

Targets: CNS, salivary glands, parietal cells. Increases: IP3/DAG Causes: Increase in CNS excitation, memory, locomotor activity, **gastric acid secretion**.

M2 muscarinic acetylcholine receptors

Targets: Heart Decreases: cAMP Causes: Decrease in heart rate, force, and AV conduction

M3 muscarinic acetylcholine receptors

Targets: smooth muscle and exocrine glands. Increases: IP3/DAG Causes: All smooth muscle contraction except blood vessels. Causes glandular secretions. Sphincter relaxation. Mainly GI tract activity

temporal vs spatial summation

Temporal summation: Multiple graded potentials from the same source over a short time period. Spatial summation: Multiple graded potentials from multiple sources. Is location dependent, synapses near the axon hillock have a stronger affect on summation because the graded potentials degrade less. These summations are how the actions of any one neuron is controlled.

What sets up gradients and movement in parietal cells?

The ATP exchange of H+ (going out) for K+ (coming into cell) on the luminal side. This is how parietal cells secrete HCl, H+ and Cl- exiting the luminal side recombine to for HCl in the lumen. Chlorine is exchanged for HCO3- going into the plasma which makes the blood leaving the stomach temporarily basic.

What are the 3 basic components of the endocrine system and what is its function?

The Endocrine System consists of 3 basic components: 1. Endocrine glands 2. Hormones 3. Target organs/cells *Function*: Regulate cellular and organ function and maintain homeostasis. This is achieved by the regulated secretion of hormones that influence the activity of target organs/cells.

Hypothalamus and ADH

The Hypothalamus is involved in thirst and water pressure sensing. A primary response is emotion, including the sensation of thirst. Release of ADH causes an increase in water retention in the collecting duct. These work to increase water and reduce osmolality changes.

Pulmonary circulation vs systemic circulation branching

The Pulmonary Artery immediately begins to divide into smaller vessels. This is one of the main causes of decreased pulmonary resistance. In Pulmonary circulation vessels immediately turn into parallel networks. In Systemic circulation the flow of blood starts out as a series of long tubes designed to transport blood long distances and to maintain blood pressure. Not as much branching in the aorta = higher pressure. The difference between the Series and Parallel networks leaving the heart is reflected in the lower pressure and resistance of the pulmonary circuit. Pulmonary circulation has the same waveform as systemic circulation, just with lower pressure.

Changes in the vasa recta

The Vasa Recta is also a countercurrent exchanger. The concentration of the blood inside these capillaries matches that of the interstitial space. As the Vasa Recta, *which is found at the boundary of the cortex and renal medulla*, descends solutes come into the blood while H2O flows out into the interstitial space (highly permeable to solutes). At the bottom, the mOsm is 1200 which matches the interstitial space and the bottom of the loop of Henle. As it goes back up, the Vasa Recta removes solutes back into the interstitium while simultaneously taking in water. Of note, the Vasa Recta started off with 300 mOsm/L and finishes with 350 mOsm/L which implies that it reabsorbs more solutes than water.

Tonicity

The ability of an extracellular solution to make water move into or out of a cell by osmosis. Related to osmolarity. Tonicity is influenced only by solutes that cannot cross the membrane, as only these exert an effective osmotic pressure. Determines cell volume Solutions can be: Isotonic Hypertonic Hypotonic

Valsalva Maneuver

The action of attempting to exhale with the nostrils and mouth, or the glottis, closed. This increases pressure in the middle ear and the chest, as when bracing to lift heavy objects, and is used as a means of equalizing pressure in the ears. This change in pressure occurs typically during defecation and can help move the feces forward.

Movement of Urea

The ascending *thick* convoluted tubule, the distal convoluted tubule (and late distal tubule), and the outer medullary collecting duct are all *impermeable* to urea. Urea is permeable practically everywhere else. 50% of urea is reabsorbed in the proximal tubule. 30% in reabsorbed in the inner medullary collecting duct. Of the filtered load of urea, only about ~20% is excreted. (Assuming 100 units to start with) 50 units of urea moves down the descending limb (50 units were already reabsorbed in the proximal tubule) and into the ascending *thin* limb where 50 more units comes into the ascending limb from the inner medullary collecting duct. Now you have 100 units of urea travelling through the upper thick limb, convoluted tubule, and outer medullary collecting duct (all of which are impermeable to urea). Once the 100 units is in the inner medullary collecting duct (where its permeable), 50 units are secreted back to the thin ascending limb (to repeat the cycle), 30 units are reabsorbed back into the vasa recta, and only 20 units are actually excreted. Urea becomes effectively trapped in the inner medulla (bottom of the Loop of Henle) especially in the interstitial space. Therefore, there becomes a really high osmolarity here of ~1200 thanks in part to Urea.

What happens to blood pressure if baroreceptors are denervated?

The blood pressure wouldn't be held within the normal range of 80-120. Instead, the blood pressure would fluctuate sporadically throughout the day. *Causes neurogenic hypertension*

Loop of Henle description

The descending portion is devoid of solutes, making water movement out of the tubule into the peritubular capillaries possible (this is because there is both less water and less solutes in the tubules making it isosmotic with the plasma). This dilutes the previously concentrated blood that left the efferent arteriole (efferent tubule had a bunch of solutes in it from reabsorption in the proximal tubule, the descending loop barely has any solutes in it because they were reabsorbed. Therefore, the water in the descending loop moves out and into the blood that has a bunch of solutes, diluting the blood in the process.) The thick ascending limb of the loop of Henle has transporters that allow further solute reabsorption. Thin descending loop of Henle: *Water goes out* Thick ascending loop of Henle: Out- Na+, Cl-, K+, Ca2+, HCO3-, Mg2+. In- H+

Blood flow distribution in the lungs

The effects of gravity on the distribution of blood flow in the lung are attributed to the hydrostatic pressure difference between the top and bottom of the pulmonary arterial system. At the uppermost parts of the lung, the pressure within the vessels may be less than the alveolar pressure. Therefore, these vessels collapse and the alveoli that these vessels traverse will receive little blood flow. This accounts for some 'wasted ventilation' or physiological dead space. In the gravitational middle zone, pulmonary arterial pressure is greater and pulmonary artery pressure exceeds the alveolar pressure, and, similarly, in the lower zone pulmonary venous pressure also exceeds alveolar pressure. PA = Alveolar Pressure Pa = Arterial Pressure Pv = Venous pressure

Membrane potential definition

The electrical potential difference across a plasma membrane; a difference in charges, in the form of ions, on one side of a plasma membrane verses another.

Glucose movement and concentrations in plasma and urine

The filtered load is proportional to the plasma concentration. As transporters (active) begin reabsorption, they move molecules back out of the filtered load and therefore excretion doesn't take place until threshold is reached. At a certain point the transport maximum will be reached, despite the filtered load increasing due to higher concentrations. All excess molecules will be excreted. Graph: As the concentration of glucose in the plasma increases, the filtered load increases. Since glucose passes through the basement membrane in the glomerulus, this isn't surprising. Through the nephron, glucose is reabsorbed, but it depends on a transporter to do the work. The result is the blue line (transporter/reabsorption) is dependent on the kinetics of the transporter. As this transporter becomes saturated, the excretion amount (green line) begins to increase. There is simply too much glucose in the filtered load (due to high plasma concentrations) for efficient reabsorption of all the molecules. After this point of transport maximum all additionally filtered glucose will be lost to excretion.

Saliva composition

The final saliva composition is hypotonic (lots of water compared to solutes). Primary secretion: isotonic (high Na+ and Cl-; low K+ and HCO3-) Duct modification: Increases K+ and HCO3-. So initially it is isotonic with a bunch of Na+ and Cl- secreted and some K+ and HCO3- secreted, all along with water. Then in the duct is it modified via Na+ and Cl- being reabsorbed and more K+ and HCO3- being secreted along with more water. So the final composition is hypotonic.

Filtration structure

The glomerulus has a distinct shape that makes it a natural filter. Blood flows to the kidneys to be filtered and screened. Not everything is passed into the tubules of the nephron, some solutes are passed into the efferent arteriole.

How does ionic composition of saliva vary with flow rate?

The higher the flow rate the more like normal plasma the ionic concentration of the saliva, still hypotonic though. The lower the flow rate the more hypotonic the saliva.

Renal blood flow- Autoregulation

The kidneys actively work to maintain perfusion pressure to the Glomerulus. As a result GFR stays relatively stable across wide ranges of systemic arterial blood pressure. This is accomplished by changing afferent and efferent arteriole diameters through vasoconstriction and vasorelaxation via *myogenic responses*- constrict when pressure increases and dilate when pressure decreases. In theautoregulatory range, renal blood flow and glomerular filtration rate (GFR) stay relatively constant despite changes in arterial blood pressure. This is accomplished by changes in the resistance (caliber) of preglomerular blood vessels. The circles indicate that vessel radius (r) is smaller when blood pressure is high and larger when blood pressure is low.

What determines the membrane potential

The magnitude and direction of the electrochemical gradient.

Main muscle that controls ventilation?

The main muscle that controls ventilation is the diaphragm The contraction of the muscle leads to inspiration The relaxation of the muscle leads to expiration In normal, relaxed breathing- just using the diaphragm mainly. Start using the other muscles during exercise, etc.

Airway resistance

The main site of airway resistance is the medium bronchi. (5-7th generation highest total resistance) Unlike the circulatory system, the terminal bronchioles and alveoli have very high compliance and are arranged in large segments of parallel flow. This increase in cross-sectional area means there's little resistance to flow at the alveoli.

Threshold potential

The membrane potential where 50% of the time an action potential will occur. This is usually ~ -40 to -55 mV.

What is the flux?

The net movement of the solute.

Hormone receptor regulation

The number and sensitivity of hormone receptors are regulated and not constant. *Up-regulation*: Increase in the number of active receptors - Tissue becomes more sensitive to hormone. *Down-regulation*: Decrease in the number of active receptors. - Tissue is less sensitive to hormone.

Example of osmolarity and tonicity: Cell has 300 mOsm/L inside and 200 mOsm/L NaCL along with 100 mOsm/L urea outside. What are the osmolarity and tonicity of this cell

The osmolarity is both the same 300 mOsm/L for both. However, since urea can't cross the membrane If the osmolarity higher extracellularly, the cell is in a hypertonic solution(higher solutes outside cell) and water will flow out of cell to extracellular area. If the osmolarity inside of a cell is higher, the cell is in hypotonic solution (lower solutes outside cell) and water will go into the cell.

Striated appears of muscle is due to?

The overlapping of thick and thin filaments.

Which cell is responsible for the pancreatic enzyme secretions and what stimulates pancreatic secretions?

The pancreatic acinar cell is responsible for enzyme secretions and is stimulated by GRP (gastrin-releasing peptide), ACh, CCK (cholecytokinin), and substance P which all activate enzyme release via Ca2+. Also activated by secretin and VIP (vasoactive intestinal peptide) which stimulate enzyme release via cAMP

What connects the GI tract to the liver

The portal vein

Proximal tubule transporters

The proximal convoluted tubule comprises the first 60% of the length of the proximal tubule. The proximal tubule is responsible for reabsorbing all of the filtered glucose and amino acids, reabsorbing the largest fraction of the filtered Na+, K+, Ca2+, Cl-, HCO3-, and water, and secreting various organic anions and organic cations. In addition, the active Na/K-ATPase pump is essential for sodium reabsorption, which serves as the major driving force for reabsorption of solutes and water in the proximal tubule. Na+ enters the cell from the lumen across the apical cell membrane and is pumped out across the basolateral cell membrane by the Na/K-ATPase. The blood surrounding the tubules then takes up the Na+, accompanying anions, and water. Filtered Na+ salts and water are thus returned to the circulation. The reabsorption of Na+ and accompanying solutes establishes an osmotic gradient across the proximal tubule epithelium that is the driving force for water reabsorption. Because the water permeability of the proximal tubule epithelium is extremely high, only a small gradient (just a few mOsmo/kg of H2O) is needed to account for the observed rate of water reabsorption. Some experimental evidence indicates that proximal tubular fluid is slightly hypo-osmotic to plasma. Because the osmolality difference is so small, it is still proper to consider the fluid as essentially *isosmotic to plasma*. Water crosses the proximal tubule epithelium through cells and between cells. *Diffusion*: Na+, Cl-, K+, H2O *Facilitated diffusion*: Glucose, Amino Acids, Phosphate *Primary active transport*: Na+, K+ *Secondary active transport*: Glucose, amino acids, phosphate, H+, Base, HCO3-, K+, Cl-

Proximal tubule characteristics

The proximal tubule epithelial cells are highly metabolic and have large numbers of mitochondria to support powerful active transport processes. In addition, the proximal tubular cells have an extensive brush border on the luminal (apical) side of the membrane, as well as an extensive labyrinth of intercellular and basal channels, all of which together provide an extensive membrane surface area on the luminal and basolateral sides of the epithelium for rapid transport of sodium ions and other substances.

Regional compliance

The relative pressures at the top of the lungs (pleural space) is higher at -10 than at the bottom of the lungs (pleural space) which is -2.5. This is because gravity pulls the fluid in interstitial spaces down towards the bottom of the lung and creates a large positive pressure on the alveoli at the base vs. the apex of the lung. Due to this pressure at the top, the alveoli at the top of the lungs are already almost maximally inflated while alveoli at the base are not inflated. Therefore, ventilation is highest at the base of the lungs because they have the smallest (not inflated) alveoli and can take in the most volume.

Sarcomere

The repeating unit of a myofibril, between 2 z lines.

Short bowel syndrome

The short bowel syndrome resulting in dehydration and malabsorption occurs as a result of massive intestinal resection, especially of the ileum with or without the colon. Resection of up to 100 cm of ileum causes diarrhea, because there are progressively greater degrees of bile salt malabsorption. Malabsorbed bile salts enter the colon where they cause water secretion by activating cAMP. When the resection exceeds 100 cm, there is progressively more fatty acid loss in the colon, which also adds to water secretion and diarrhea. There is also malabsorption of vitamin B12. In addition, there is loss of energy in the form of increased fat loss. However, as the length of the resection increases, there is malabsorption of all macronutrients, mainly, fat, carbohydrates, and protein. The malabsorbed carbohydrate entering the colon is fermented to produce flatulence and diarrhea. In addition, there is malabsorption of vitamins and trace elements such as zinc.

Gas exchange timing

The standard assumption is that when we inhale air, it directly goes down into the lungs and fills the air sacs. However, this isn't entirely accurate. Typically we inhale air and it fills the large airways. After this point the air mixes down into the lungs through diffusion. It turns out that we don't completely replace our entire volume of air in the air sacs with each breath. Instead we slowly mix gases over time. Takes about 20 breaths to exchange the air completely in an air sac. This is good because it protect us from poison etc, can smell it first before it completely saturates the air sacs. We (usually) have a constant pressure in the alveolar sac due to this principle.

How does mechanical stretch increase frequency of action potentials in single-unit smooth muscle of GI

The stretching of single-unit smooth muscle opens its stretch-sensitive ion channels which depolarizes the membrane and causes contraction which counteracts any additional stretching.

Mechanics of stetching

The thin tissue of the lungs (Type I and Type II) have very limited interstitial space. The interstitial space is filled with stretchy *Elastin fibers*. Elastin fibers allow the lungs to stretch or contract a little bit (*Elastic recoil*) but mainly keeps it in a set range to prevent collapse or overinflation. Emphysema is the destruction of these Elastin fibers.

Storage of thyroid hormone

The thyroglobulin T3, T4, rT3, MIT, DIT complex is endocytosed into the epithelial cell. MIT and DIT get deiodinased. Lysosome releases T3, T4, and rT3 from complex with hormones then exiting into the blood.

Iodide uptake by the thyroid

The thyroid gland actively accumulates iodide via a *sodium/iodide symporter (NIS)* The NIS allows iodide to be trapped within the thyroid gland against concentration and electrochemical gradients (Iodide Trapping). *Secondary active transport*

PV loop

The top of the curve is C- corresponds with systolic blood pressure. B corresponds with diastolic blood pressure. D corresponds to aortic valve closing and mitral valve opening. B corresponds with Mitral valve closing and aortic valve opening. D and E on X-axis correspond to ESV. A on X-axis corresponds to EDV. A to B = Isovolumetric contraction. B to C is rapid ejection. C to D is reduced ejection. D to E is isovolumetric relaxation. E to A is rapid ventricular filling, reduced ventricular filling, and atrial contraction. A to D is ventricular contraction. D to A is ventricular relaxation. (PICTURE IS FIXED NOW)

GnRH in female reproductive system

Theca cell stimulates cholesterol via cAMP to testosterone. Theca cells do not have aromatase, can't make estrogen. Granulosa cells (when small have FSH receptors, not making androgens but do have aromatase (A) can take androgens from theca cells and make estrogen). Later once the follicles develop a little more they get LH and then get to make progesterone. Environment, Age, Drugs -> stimulates/inhibits -> brain centers (if stimulated) -> Hypothalamus -> GnRH -> Anterior pituitary -> FSH/LH -> Ovary -> Estradiol, progesterone, androgens -> Stimulates Reproductive tract and secondary sex characteristics. LH causes ovulation and luteinization of the ovulated follicle in the ovary of the human female and stimulates the production of the female sex hormones estrogen and progesterone by the ovary. FSH stimulates the development of follicles in the ovaries (Just prior to ovulation, positive feedback effect by estradiol)

HCO3- creation

There are 2 different mechanisms for the formation of new HCO3- for the body. Both rely on utilizing CO2 found in the body (an acid) to create HCO3- and H+. The H+ is then secreted into the urine and paired with either NH3 or HPO4. In either case the excess H+ is lost to the urine and new HCO3- is added to the blood stream. Left shows excretion of ammonia, right shows formation of titratable acid.

What are the 2 types of acids in the body?

There are 2 types of acid in the body: 1. H2CO3 (carbonic acid) 2. Everything else ("nonvolatile acids") One of the most identifiable acids is CO2. To buffer CO2 and H2CO3 the body uses carbonic anhydrase to produce the conjugate base HCO3-. H2O + CO2 (+ carbonic anhydrase) -> H2CO3 -> HCO3- + H+. All arrows are reversible.

Overview of chemical buffers

There are several different chemical buffers in the body, each with different pKA values. HCO3- is the most abundant buffer of all.

Branching of the lungs

There is a lot of branching in the lungs with increasing total cross-sectional area and decreasing resistance at each branch. Airway generation: Trachea -> Bronchi-> Bronchiole -> Terminal bronchiole. This is 0-8th branching (trachea is 0 branch). This marks the conducting zone. From branches 9-16 there is no gas exchange. Respiratory bronchiole -> Alveolar duct -> alveolar sac. This is 17-23 branching. This marks the respiratory zone.

How does Multi-unit smooth muscle work?

These cells have little to no gap junctions. Each cell works individually. However, these cells are all connected to a parasympathetic or sympathetic nerve that rests alongside a bunch of these cells. Therefore, although the cells act individually, when the nerve is activated it releases neurotransmitters from its vericosities which then binds to a bunch of these cells at the same time and increases/decreases their membrane potential (depending on if excitory or inhibitory). However, a single action potential at the nerve fiber only releases enough neurotransmitters for only a small sub-threshold change in the membrane potential, multiple action potentials in the nerve are required to depolarize these smooth muscle cells to threshold. All of these cells depend on the frequency of nerve stimulation to depolarize. Also, certain circulating hormones can depolarize the cells by binding to their membrane surface and activating the G-protein cascade.

Where are the pacemaker cells (Interstitial cells of Cajal) located?

They are located in the small intestinal *circular muscle* at the border with the longitudinal muscle layer (myenteric border) and at its border with the submucosa

Excretion of Acid in the thick ascending limb and medullary collecting duct

Thick ascending: NH4+ can substitute K+ in the Na/K/2Cl In the thick ascending limb, NH4+ is transported from tubular into cell and then out to interstitial as NH3+ which recombines with H+ there. That NH4+ and NH3+ that was just put into the interstitial can be picked up in the medullary collecting duct. NH3 enters via RhBG and RhCG and NH4+ enters via antiport with 3 Na+ using ATP. While in the cell, NH3 and H+ dissociate and NH3 enters tubule lumen via RhCG and H+ enters via ATP coupled H+ channel.

Proteins of sarcomere (3)

Thick filaments (myosin) Thin filaments (actin, tropomyosin, troponin) Structural (titan)

Dense bodies

Thick filaments in smooth muscle (myosin) that are involved in contraction.

Tropomyosin

Thin filament 2 identical alpha helices that coil around each other and sit near grooves of actin strands

Relative concentrations of urea throughout the nephron

Throughout the nephron the changes to the interstitial space and the peritubular fluid can be seen. These are in alignment with changes to solute and water flux in the tubular urine. Water moves in response to *Total* solute concentration (so urea + NaCl). NaCl, on the other hand, moves down its own concentration gradient. As you move down the descending limb, water moves out into the peritubular fluid because of the high amount of solute (NaCl and urea) in it (due to the ascending limb pushing Na+ into the peritubular fluid). This in turn concentrates/ increases the osmolarity of NaCl and urea within the tubule. The descending limb and its peritubular fluid have the same exact osmolarity, the tubule is isosmotic at this point. Once you hit the bottom of the loop of Henle, you have your highest osmolarity within the tubule (and therefore also in the peritubular fluid). Once you start going up the ascending limb, water is *no longer permeable* but urea is permeable in the *thin ascending limb* and NaCl is permeable in the thin and thick ascending limb. As you go up the ascending limb, that *1120 NaCl* concentration is higher than the surrounding 600 and then 400 at the border of outer and inner zone of the medulla. Therefore, the NaCl leaves down its concentration gradient via NKCC until it becomes equal to the surrounding NaCl concentration. Because water can't move in to equalize the osmolarities, the ascending tubule becomes hypo-osmotic (because NaCl kept moving out). In the thin ascending tubule, urea moves into the thin ascending tubule down its concentration gradient. Not being permeable in the thick ascending tubule, the urea level stays at 100 as it moves up but the NaCl level continues to go down.

What are the components of the thyroid response element?

Thyroid hormone receptor (TR) and Retinoid X receptor (RXR) make a heterodimer.

Alveolar ventilation

Tidal Volume is the amount of air inhaled in each breath. This is typically 0.5L for a healthy individual at rest Anatomical Dead Space is comprised of airways that don't partake in respiration. Any air in this volume is considered "wasted" Alveolar Volume is the amount of the tidal volume that makes it to the alveolar space and takes part in gas exchange (Tv - dead space = 500 - 150 = 350) Alveolar ventilation = total ventilation - anatomic dead space = 5,250 ml/min if you breath 15x a minute. This number is same/similar to cardiac output which is 5 L/min

Lung Volumes

Tidal Volume: Normal respiratory range. The volume of air moved into or out of the lungs during quiet breathing (watching T.V. etc.) IRV (Inspiratory reserve volume): The maximal volume that can be inhaled from the end-inspiratory level. (Above Vt, doesn't include Vt) ERV (Expiratory reserve volume): The maximal volume of air that can be exhaled from the end-expiratory position. (Below Vt, doesn't include Vt) RV (residual volume) The volume of air remaining in the lungs after a maximal exhalation, can't breath it out. (Not determined directly by spirometry) FRC (Functional residual capacity): The volume in the lungs at the end-expiratory position. (ERV + RV) (Not determined directly by spirometry) IC (Inspiratory capacity) = IRV + Vt. VC (Vital capacity): The volume of air breathed out (with effort) after the deepest inhalation. = IRV + Vt + ERV. TLC (Total lung capacity): The volume in the lungs at maximal inflation. = VC + RV. (Not determined directly by spirometry).

Volumes

Tidal volume (Vt) is the amount of air inhaled in each breath. This is typically .5 L (500 mL) for a healthy individual at rest. Anatomical Dead Space (Vd) is comprised of airways that don't partake in respiration (9-16). Any air in this volume is considered "wasted". For most people this is about 150 mL. Alveolar Volume (Va) is the amount of the tidal volume that makes it to the alveolar space and takes part in gas exchange. About 350 mL are here.

Distal convoluted tubule

Tight membrane that's impermeable to water. (but the late distal tubule (connecting/collecting tubule) is permeable) Works against high gradients *Reabsorption of Na+ into the blood from the urine.* Na+/Cl- symporter transports Na+ and Cl- from the Tubular urine into the distal convoluted tubule cell. Na+ is then transported to blood via Na+/K+ ATPase. K+ and Cl- enter blood via respective channels, facilitated diffusion.

Troponin- heterotrimer

TnT- binds tropomyosin TnC- binds calcium TnI- binds actin and inhibits contraction.

Minute ventilation (total ventilation) and alveolar ventilation

Total ventilation (Minute ventilation): Total amount of air moved into (or out of) the lungs in volume per time (rate). It is determining the volume of air inspired every minute. It's a function of the breathing frequency (f) and tidal volume. VE = VT * f Where VE is minute ventilation. VT is tidal volume f is respiratory rate per minute. Alveolar ventilation: rate at which new air reaches the alveoli VA = (VT - VD) * f VA is alveolar ventilation VD is physiological dead space volume f is respiratory rate per minute ------ Minute ventilation = tidal volume x respiratory rate Alveolar ventilation = (tidal volume - dead space) x respiratory rate.

Blood functions

Transport (long distance)- dissolved or bound substrates. - O2, Co2, antibodies, acids/bases, ions, vitamins, hormones, nutrients, metabolites, minerals - Heat Hemostasis - arrest of bleeding after injury Homeostasis- Maintains optimal internal environment - pH, ion concentrations, osmolality, temperature, nutrient supply, vascular integrity. Immunity - Leukocytes in the blood fight infection by microorganisms.

Triad consists of?

Transverse tubule surrounded on both sides by sarcoplasmic reticulum cisterna

Isoosmotic to hypoosmotic in the loop of henle why?

Tubular fluid is isosmotic when entering the loop of henle and then in the descending limb water starts leaving because there is more solutes outside of the descending tubule. At the terminal portion/ascending tubule, water can't go in or out because there are no aquaporins. However, there is a +6 mV potential difference between the lumen and the interstitial space around it. This drives small cations such as Na+, K+, Ca2+, Mg2+, and NH4+ out of the lumen, between cells (not through the tubular cell because it's still at -72 mV). This causes the ascending limb to become hypo-osmotic compared to the plasma/interstitial fluid. 100 mOsm/kg H2O in the distal convoluted tubule compared to the 285 mOsm/kg H2O in the plasma. This is because more solute than water is reabsorbed by the loop of Henle. Loop of Henle reabsorbs about 20% filtered Na+, 25% of filtered K+, 30% of filtered Ca2+, 65% of filtered Mg2+, and 10% of filtered water. Descending limb *highly permeable to water, lots of aquaporins* (except for its terminal portion). Ascending limb is *impermeable to water, no aquaporins*.

Steroidogenesis (ovary)

Two-cell, two-gonadotropin hypothesis Theca cells acquire LH receptors at a relatively early stage, whereas FSH induces LH receptors on granulosa cells in the later stages of the maturing follicle. Theca cell has 17a hydroxylase so it can convert progesterone to testosterone. It doesn't have aromatase so it has to send its testosterone to the granulosa cell where the FSH has aromatase and convert the testosterone to estrogen. The granulosa cell lacks 17a hydroxylase so the furthest it can go is progesterone. Aromatase is regulated by FSH LH regulates androgen production (mainly testosterone) on theca cells.

Structure of alveolus

Type I Alveolar epithelial cell - barrier formation. Type II Alveolar epithelial cell- Surfactant secretion (*Surfactant = a glycoprotein that tends to reduce the surface tension of a liquid in which it is dissolved. Without surfactant you would go into respiratory distress (fluid fills alveolar air sacs)*) Macrophage (if foreign substance/invasive thing in lungs) - immune system component. Alveolar fluid lining is between the Type I (barrier) alveolar cell and the alveolus, contains pulmonary surfactant secreted by Type II alveolar cell

GI Digestion and Absorption: Pathophysiology Celiac Sprue

Types and causes: Non-tropical: - Celiac disease - Gluten destroys enterocytes - Mucosal inflammation - Immune-disorder associations Tropical: - Bacterial infection Signs and symptoms - Decreased absorption within the small intestine and throughout the bowel - Diarrhea - Flatulence - Severe abdominal pain

What regulates salivary flow?

Under Brainstem control. Stimuli: taste, tactile, smells +/-, nausea. Parasympathetic increases salivation and digestion. The parasympathetic nervous system stimulates bradykinin release which causes vasodilation and augments blood flow. Sympathetic: dry, sticky mouth.

Hypoxia induced vasoconstriction

Unlike in systemic circulation where local mediators Increase in O2 causes vasoconstriction, and Increase in CO2 causes vasodilation, in the pulmonary circulation decrease in O2 causes vasoconstriction and increase in CO2 causes vasoconstriction. When concentration of oxygen is below 70 percent of normal (below 73 mmHg PO2) the adjacent blood vessels constrict. This is a good thing, you don't want pulmonary arterial blood traveling to regions with low O2, its a waste of blood flow because less exchange can take place there. By diverting blood away from poorly ventilated regions, pulmonary circulation can maximize ventilation and perfusion. Generalized hypoxia will cause pulmonary hypertension due to persistent vasoconstriction and smooth muscle remodeling. High CO2 content in lungs is the main indicator/sensory behind this process.

AP propagation in unmyelinated axon vs myelinated axon

Unmyelinated: Passive depolarization spread. One region becomes depolarized which causes the neighboring region further down axon to become depolarized and hit their action potential. As this depolarization wave continues, the left side is becoming repolarized. Myelinated: Usually triggered at the axon hillock, just before start of myelin sheath. Depolarization spreads passively between nodes (nodes of ranvier) until threshold where a new action potential is generated. Refractory period prevents the signal from going backwards.

Einthoven's triangle

Upward deflection = recording is going in the direction of the positive electron. Lead I: Records from right arm to left arm. Lead II: Records from right arm to left leg. Lead III: Records from left arm to left leg.

Regional differences of V/Q ratios

V/Q Ratios: Apex = 3.6 Base = .6 Ventilation and perfusion are matched at different rates in different locations Pathologies can alter your V/Q ratios. Perfusion changes (decreases) more than ventilation as you go up the lung. Blood flow really reduced at the apex more so that the ventilation (both reduced) causing a high V/Q ratio. The partial pressure of oxygen in the apex alveolar is higher (130) but because of the lowered perfusion not a lot of the oxygen will diffuse to the capillaries. Lower partial pressure of carbon dioxide at the apex alveolar because of lower ventilation (carbon dioxide not diffusing from the capillaries to the alveolar)

How to calculate alveolar ventilation

VA = (VE(CO2) * 0.863)/ PA(CO2) Where VA is alveolar ventilation VE(CO2) - minute expiration of CO2 (how much co2 we are producing/breathing out) PA(CO2) - arterial partial pressure of CO2 Practically no CO2 in atmosphere so the CO2 we breath out is a good measure of gas exchange. From the CO2 expired and knowing the arterial partial pressure of CO2 we can calculate our alveolar ventilation. Arterial CO2 is inversely proportional to our ventilation rate. Therefore, if our arterial CO2 is high this means our alveolar ventilation rate is poor. (the .863 converts units to mmHg)

Regulation of gastric secretions.

Vagal stimulation of ACh and GRP (gastrin releasing peptide). ACh can go directly to the parietal cell and activate its HCl production or it can go and activate the ECL cell (Enterochromaffin-like cell) which then releases histamine which goes to the parietal cell and activates it stimulating HCl production. GRP inhibits the D cell and also goes to the G cell and activates it releasing gastrin which goes and activates the parietal cell directly and also activates the ECL cell which releases histamine which activates the parietal cell. The D cell is activated by secretin, glucagon, GIP (Gastric inhibitory polypeptide), and VIP (vasoactive intestinal peptide) and releases somatostatin when activates which inhibits the parietal cell directly and inhibits the G cell. PGE2 (prostaglandin) can inhibit the ECL cell, G cell, or directly inhibit the parietal cell.

Route of major parasympathetic outflow from the head is via the?

Vagus nerve.

Action of valves

Valves open passively due to pressure gradients. AV valves open when atrial pressure > ventricular pressure. Semilunar valves open when the ventricular pressure > arterial pressure.

What is the osmolarity and flow in the vasa recta compared to the loop of henle and collecting duct

Vasa recta: - Descending = 285 mosm/kg, 100 ml/min. - Ascending = 315 mosm/kg, 117 ml/min. Loop of Henle: - Descending = 285 mosm/kg, 36 ml/min - Ascending = 100 mosm/kg, 24 ml/min Collecting duct: - Enter = 285 mosm/kg, 6 ml/min - Exit = 1200 mosm/kg, 1 ml/min

Vascular compliance

Vascular Compliance: distensibility of a vessel (how much it can stretch). - Compliance (slope) decreases at higher P and V (hits max but before then High V low P huge slope, rise over run). - Veins are much more compliant that arterioles. The large compliance in veins allows them to accommodate high volumes with little change in pressure. C=ΔV/ΔP

T wave of EKG represents which electrical activity?

Ventricular repolarization

Diffusion of gaseous molecules

Vgas = (As x D x delta P)/ T Vgas = volume of gas diffusing per minute As = surface area D = diffusion coefficient delta P = change in partial pressure T = membrane thickness The diffusion coefficient is related to the solubility of the molecule divided by its molecular weight. That is why the diffusion coefficient of CO2 is 20 times greater than that of O2. It is far more soluble. *Respiratory Echange Ratio (R)*: At steady-state 250 ml O2 are transferred to pulmonary circulation and 200 mL of CO2 are removed. This is known as the respiratory exchange ratio, R. R = 200/250 = .8

Calcium removal from cytoplasm in cardiomyocyte

Via SERCA using ATP to transport Ca2+ back into the sarcoplasmic reticulum via ATP and binding it with calsequestrin. Via NCX (Na/Ca2+ exchanger) with Ca2+ transported out and Na+ pumped in (antiport).

How is T4 converted to T3?

Via a 5'/3' monodeiodinase activity which removes the 5' iodine, converting T4 to T3.

How is trypsinogen activated?

Via enterokinase located on the membrane of intestinal villi.

How are material taken in via pinocytosis or phagocytosis typically disolved?

Via lysosomes and then the digested material can be transported/released/excreted.

Ciliary movement

Via microtubules (tubulin) Found in the respiratory airway and other locations (mucocilliary escalator) Supported by 11 microtubules. Protein cross-linkages linke the 9 double tubules and 2 single tubules that make up the cilia. Require ATP, causes whip-like beating.

How is cAMP broken down?

Via phosphodiesterase

Atrial Natriuretic Peptide (ANP)

Volume expansion stretches the right atria and causes release of ANP. The impact is an attempt to reduce circulating fluid volumes. (promotes urination, increased Na+ excretion). *This causes a reduction in angiotensin II* Increased nocturnal urination in people with Afib and right sided heart failure because of activation of ANP via stretching the right atria while lying down.

Blood in the cardiovascular system

Volume of blood in adults: 4.5-5.5 L Small veins/venules: 60-68% of all blood. Large veins part of this but smaller pie size. Lungs: 10%- 12% Systemic arteries: 10%-12% Heart: 8%-11% Capillaries: 4%-5% Arterioles: 2% Most blood is in veins because they are more stretchy- can hold more blood.

Skeletal Muscle fiber characteristics (5)

Voluntary striated multinucleated non-branched extensive sarcoplasmic reticulum

Changes to fluid balance

Water deficit -> increase in extracellular osmolarity (sensed by osmoreceptors) -> increase in ADH secretion (hypothalamus to posterior pituitary) -> increased plasma ADH concentration -> Increased H2O permeability in distal tubules, collecting ducts -> increased H2O reabsorption -> decreased H2O excretion which then causes negative feedback via correcting the initial water deficit. When H2O levels change in the body the main impact is on extracellular osmolarity. The feedback mechanism for alteration in body H2O levels is the secretion of the hormone ADH. Anti-Diuretic Hormone (ADH) = Arginine Vasopressin (AVP).

Short term regulation of cell volume in hypotonic solution?

Water is entering the cell and therefore the cell responds via efflux of K+ by way of K+/Cl- symport, K+ channels, and Cl- channels. This increases solute concentration outside the cell and water leaves the cell via osmosis.

Short term regulation of cell volume in hypertonic solution?

Water is leaving the cell and therefore the cell responds via influx of Na+ by way of Na+/H+ antiport, Na+/K+/2Cl- symport, and Cl-/HCO3- antiport. This increases solute concentrate in the cell and water returns to cell via osmosis.

Fluid flows and Concentrations

Water is pulled out of the descending limb of the loop of Henle (-8 ml/min flow). That water enters the vasa recta (+7 ml/min flow) Salt is moving out of the thick ascending limb of the loop of Henle and into the vasa recta which causes an increase in concentration (in the vasa recta) (315 mosm/kg). The loss of salt in the thick ascending limb lowers the concentration at the distal tubule (100 mosm/kg).

Creatinine clearance and GFR

We can calculate the GFR from creatinine but we must make several assumptions about metabolic activity and kidney function. Since we produce and excrete creatinine the amount filtered should be proportional. Note the relationship between plasma concentration of creatinine and GFR. The graph shows the GFR and plasma concentrations of creatinine are in balance. Since creatinine is produced at a relatively steady state we can estimate the GFR based on the plasma concentration. If the plasma concentration increases, the GFR (or filtered load) must have decreased.

Tubuloglomerular feedback mechanism.

When single-nephron glomerular filtration rate (GFR) is increased - for example, because of an increase in arterial blood pressure - more NaCl is delivered to and reabsorbed by the macula densa, leading to constriction of the nearby afferent arteriole. This negative-feedback system plays a role in autoregulation of renal blood flow and GFR. Another method is through myogenic mechanism: Example- an increase in pressure stretches blood vessel walls and opens stretch activated cation channels in smooth muscle cells. Th e ensuing membrane depolarization opens voltage-dependent Ca2+ channels and intracellular Ca2+ rises, causing smooth muscle contraction

equilibrium potential for K+

When the chemical and electrical gradients are balanced and there is no net flux of K+

Macula Densa and Na+ levels

When there is low GFR due to low blood pressure then not as much Na+ with filter and the flow will be slower allowing more Na+ to be reabsorbed before it gets to the Macula Densa cells in the thick ascending tubule/distal convoluted tubule. Macula Densa cells sense this low Na+ and stimulate the granular/juxtaglomerular cells to secrete Renin (via cAMP pathway). Renin-Angiotensin system constricts the efferent arteriole which increases GFR allowing more Na+ filtration. The renal blood flow in the Vasa Recta and Peritubular is decreased because of the constriction leading to lower capillary hydrostatic pressure and therefore more Na+ reabsorption. When there is high GFR due to high blood pressure there is more filtration of Na+ and the flow increases thus it spends less time at the reabsorption part and more is able to pass through. Sensing higher Na+ levels in the macula densa, they instead constrict the afferent arteriole leading to decreased GFR. We want to keep Na+ because it is used in so many ways therefore even though we have a lot of Na+, the constriction of the afferent arteriole decreased Renal Blood Flow in the Vasa Recta and Peritubular which decreases their hydrostatic capillary pressure and allows more Na+ reabsorption. (basically sympathetic stimulation)

Baroreceptor reflex over time

When you have a sustained high blood pressure, baroreceptors become less sensitive and their firing rate at higher blood pressures decreases moving the curve down and to the right.

Diffusion: Gas Exchange

While CO2 has a 20 times greater Diffusion coefficient (D) than O2, in the pulmonary capillary the partial pressure gradient of O2 is 10 times greater. The two pressures rapidly reach equilibrium before the end of the capillary and are ready for circulation.

Proximal tubule

While reabsorption and secretion happen throughout the nephron, the proximal tubule is highly important in both. Immediately after filtration, nearly 70-80% of all filtered water and ions are reabsorbed by the proximal tubule. All glucose and amino acids are reabsorbed in the proximal tubule. The largest fraction of filtered: Na+, K+, Ca2+, Cl-, HCO3-, and H2O are reabsorbed in the proximal tubule H+, organic acids, bases are secreted.

Gray Rami vs White Rami

White rami (T1-L2) is presynaptic nerve that goes into sympathetic ganglion (the nodule of sympathetic trunk) and synapses with the Gray Rami (post-synaptic)

Composition of blood

Whole blood (approximately 5 L in an adult) - 55% liquid plasma (dissolved substances, plasma proteins) - 45% cellular/formed elements portion (RBCs, WBCs, platelets) Plasma = serum + clotting factors.

A patient has a heart attack that results in slowing of the electrical depolarization throughout the ventricles. What do you expect to see on the EKG?

Wider QRS complex.

How do secretory vesicles fuse with the plasma membrane?

With v-Snares and t-Snares mingling

How single-unit smooth muscle depolarization works

Within single-unit smooth muscles there are pacemaker cells, called the Interstitial cells of Cajal, that cause spontaneous depolarization once threshold is reached, these cells set the pace for how many times all the cells hit their action potentials. Some of the cells aren't pacemaker cells but get depolarized pretty much at the same time because of the fast movement of the depolarization wave through the gap junctions, this allows all the cells to basically contract at the same time and work as a "single-unit." The pacemaker cells are relatively slow to get going and reach threshold to fire because the Ca2+ conductance gradually increases (the rate at which Ca2+ crosses through the channel into cell). When the cell membranes are depolarized closer to threshold (but not at threshold) via nerve activity, hormones, mechanical stretching, or a variety of drugs, the frequency of spontaneous action potentials increases and thus the resulting mechanical activity of the muscle increases.

Collecting Duct and Water

Without ADH: - Urine: 70 mOsm and contains 15% of filtered H2O. With ADH (possible): - Urine: 1200 mOsm and contains 0.5% of filtered H2O. Range of water that can be excreted = 0.5%-15% of filtered water. ADH also known as AVP (Arginine Vasopressin). ADH/AVP is activated via high plasma osmolarity. Binds to V2 receptor of G coupled proteins Gs. Gs then binds adenylyl cyclase -> cAMP -> Increases aquaporin-2 synthesis -> PKA -> transports aquaporin-2 to the collecting duct epithelium.

Left sided heart failure

Would cause high pulmonary pressure because the blood would back up in the lungs due to improper functioning left ventricle. This creates high hydrostatic pressure in the pulmonary capillaries and causes pulmonary edema/flash edema.

Starling forces in pulmonary circulation capillaries

You don't want the capillaries pushing out too fluid. You just want the diffusion of O2 and Co2. Arterial end: The capillary hydrostatic pressure (35 mmHg) is very low but slightly higher than the interstitial-fluid hydrostatic pressure (25 mmHg) at the arterial end allowing for a +10 mmHg difference that pushes blood into the tissues (filtration) Venous end: The albumin within the capillary causes a pulling oncotic force (18 mmHg) that is slightly higher than the oncotic force coming from the elastin in the interstitial space causing fluid to enter the capillary (reabsorption)

Alpha and beta receptors are classes of ________ receptors.

adrenergic

Adrenergic receptor A1

alpha 1 (A1): Norepinephrine -> a1 g coupled protein receptor -> phospholipase C -> IP3 -> Ca2+ which causes: smooth muscle contraction Found in blood vessels, bladder, and skin (piloerection)

Filamentous Actin (F-actin)

double-stranded α-helical polymer Thin filament

What controls the ANS

hypothalamus. Hypothalamus also controls temperature, endocrine activity, and thirst

Transverse (T) tubules

invaginations of the sarcolemma that go into the muscle fiber. They run alongside the cisternae of the SR. Very important in transfer of action potential to the muscle cell.

What are the range values of pH, H+, Pco2, and HCO3-?

pH = 7.35-7.45 H+ = 35 - 45 Pco2 = 35 - 45 HCO3- = 22-26

Ventilation and the control of CO2

pH also control out ventilation rates. Excess CO2 in the body will combine with water and produce H2CO3 which will form H+ and HCO3-. In order to control excess H+ we can bind it to HCO3- and convert it to H2O and CO2. This can then be ventilated. As the pH of arterial blood goes up (more alkalosis) your ventillation rate goes down. If your pH is acidic, you hyperventilate to convert H+ + HCO3- -> H2CO3 -> CO2 + H2O and then the CO2 is breathed off. If you hyperventilate because of a panic attack etc (have normal initial pH) then you reduce the amount of H+ in your body below normal levels and your pH increases (alkalosis).

A 50-year-old female arrives at the emergency department with complaints of lightheadedness, confusion, dry mouth, and tingling sensations in her arms. She has been extremely busy at work recently and felt like she had a panic attack before the onset of her symptoms. Electrolyte results (mEq/L) Na+ 137 K+ 3.8 Cl- 97 HCO3- 19 ABG: pH: 7.47 PCO2: 18 mm Hg PO2: 95 mm Hg What is the acid base disorder?

pH is high so alkalosis. PCO2 and HCO3 is low meaning respiratory alkalosis. Anion gap doesn't matter in this patient. Only in metabolic acidosis patients.

A 60-year-old homeless male presents with nausea, vomiting and poor oral intake 2 days prior to admission. The patient reports a 3 day history of binge drinking prior to symptoms. Electrolyte results (mEq/L) Na+ 132 K+ 5.0 Cl- 104 HCO3- 16 Albumin 1.0 g/dL ABG: pH: 7.30 PCO2: 29 mm Hg PO2: 92 mm Hg What is the acid base disorder?

pH is low so this is acidosis. CO2 is low and so is HCO3- meaning metabolic acidosis. Anion gap is 132 - 104 - 16 = 10 which is normal. Non-anion gap metabolic acidosis. (end-stage liver failure).

Normal values of pH, H+, Pco2, and HCO3-

pH is tightly regulated and maintained at 7.4 *Things like oxyhemoglobin dissociation curves and protein function depend on a physiological pH of 7.4 to carry out proper homeostasis.*

Sarcolema

plasma membrane of muscle fiber, is fused to tendon.

Esophageal Diverticula

pouch of stretched tissue along the esophagus, pushing outward through the muscular wall Types: - Immediately above the upper esophageal sphincter (Zenker) - Near the midpoint of the esophagus (Traction) - Immediately above the lower esophageal sphincter (epiphrenic) Treatment: - Surgery if symptomatic.

Even in the absence of working potassium channels (pathology/pharmacology), the closure of sodium channels and always-present potassium pores lead to a __________ but it is slower than the usual state described above.

repolarization

Sarcoplasmic reticulum

specialized endoplasmic reticulum for skeletal muscle (cardiac muscle has similar). Lies in sarcoplasm and surrounds myofibrils. Especially important for controlling muscle contraction. Terminal Cisternae of SR = enlarged parts of SR that lie near T tubules

G-at pathway

t stands for transducin. This G alpha subunit is common in the visual pathways of the photoreceptor cells. 1. Activated by light, G-at binds to phosphodiesterase and activates it. 2. Phosphodiesterase converts cGMP to GMP. 3. The breakdown of cGMP leads to the closure of cGMP-dependent channels.

Total peripheral resistance

the resistance of the entire cardiovascular system, also known as systemic vascular resistance. MAP = CO x TPR so TPR = MAP/CO Pressure in the arteries is equal to the amount of blood pushes out of the arteries every minute multiplied by the resistance of those arteries. (P = QR) so (R = P/Q)

Loop of Henle reabsorption summary

~20-25% of sodium is reabsorbed in the thick ascending limb. (70% was already reabsorbed in the proximal tubule). Reabsorbs: ~20% of filtered Na+ ~25% of filtered K+ ~30% of filtered Ca2+ ~65% of filtered Mg2+ ~10% of filtered water Sodium reabsorption throughout nephron: Proximal tubule = 70% Thick ascending limb = 20% Distal convoluted tubule = 6% Collecting duct = 3% Urine = 1%


Conjuntos de estudio relacionados

Lesson 8: Listing Agreements: Pop Quiz

View Set

Air Methods Part 135 Study Guide

View Set

Reasoning in Algebra and Geometry

View Set

Oracle Study Guide Sections 1-10

View Set

Lewis Chapter 37 Vascular Disorders

View Set